Wikipedia Reference Desk – All recent questions
 
Shortcut:
WP:RD/ALL
WP:RD/ALL redirects here. You may also be looking for Wikipedia:Resolving disputes, Wikipedia:Redirect or Wikipedia:Deletion review.

This page lists all the recent questions asked on the Wikipedia reference desk by category. To ask a new question, please select one of the categories below. To answer a question, click on the "edit" link beside the question.

For information on any topic, choose a category for your question:

Computing reference desk
Science reference desk
Mathematics reference desk
Humanities reference desk
Computers and IT Science Mathematics Humanities
Computing, information technology, electronics, software and hardware Biology, chemistry, physics, medicine, geology, engineering and technology Mathematics, geometry, probability, and statistics History, politics, literature, religion, philosophy, law, finance, economics, art, and society
Language reference desk
Entertainment reference desk
Miscellaneous reference desk
Reference desk archives
Language Entertainment Miscellaneous Archives
Spelling, grammar, word etymology, linguistics, language usage, and requesting translations Sports, popular culture, movies, music, video games, and TV shows Subjects that don't fit in any of the other categories Old questions are archived daily
Help desk
Village pump
Help desk Village pump
Ask general questions about using Wikipedia Ask about specific policies and operations of Wikipedia
Help manual MediaWiki handbook Citing Wikipedia Resolving disputes Virtual classroom
Information and instructions on every aspect of Wikipedia Information about the software that runs Wikipedia How to cite Wikipedia as a reference For resolving issues between users An advanced guide on everything Wikipedia

Computing

edit

July 17

edit

Coding a tip calculator in TI Basic for a TI 84 Plus Silver Edition calculator

edit

I'm coding a tip calculator in TI Basic for a TI 84 Plus Silver Edition calculator. The code is as follows:

:ClrHome
:DISP "TIP CALCULATOR"
:Input "COST:", M
:M*0.15->Y
:Disp "15 PRCT:"
:Output (3, 12, Y)
:M*0.20->Z
:Disp "20 PRCT:"
:Output (4, 12, Z)
:M*0.25->X
:Disp "25 PRCT:"
:Output (5, 12, X)
:M*0.33->W
:Disp "33 PRCT:"
:Output (6, 12, W)


However, when I run this program, two things happen I don't want to happen. If I'm on just "Float" on the mode, all numbers goes to three digits (ie 3.829 for a 15% tip on 25.53). Also the last number runs 4 digits, so it's breaking onto the "Done" line. I'm aware if I change Float to 2 in the "Mode" menu, this solves both problems. However, it is tedious to change this back and forth just to run this program. I asked ChatGPT and it told me to use "toString(int", but this isn't available on the TI 84 Plus Silver Edition. Any help? Therapyisgood (talk) 13:27, 17 July 2024 (UTC)[reply]

Does your device have the round function, like: Output (6, 12, Round(W, 2)) 75.136.148.8 (talk) 14:42, 17 July 2024 (UTC)[reply]
ChatGPT was way off, the TI-84 Plus Silver Edition does have a round function and I was able to find and use it, thank you. I implemented it and it works. Therapyisgood (talk) 16:37, 17 July 2024 (UTC)[reply]

Upgrading from AMD to NVIDIA: Driver issues?

edit

I'm about to upgrade from an AMD GPU to an NVIDIA one, not for gaming, but for the sake of AI ML and video editing. Will there be any driver issues (including with DirectML, GPUOpen, or OpenCL), so that I'll need to de-install those? 2003:DA:CF11:CF56:691A:42A:6552:8061 (talk) 19:53, 17 July 2024 (UTC)[reply]

I'm not sure but please mention your operating system it may help get a better answer. Commander Keane (talk) 00:00, 18 July 2024 (UTC)[reply]
It's Windows 10. --2003:DA:CF11:CF56:691A:42A:6552:8061 (talk) 02:20, 18 July 2024 (UTC)[reply]
My procedure for systems running Windows is to 1. uninstall the GPU drivers normally through the windows control panel (or settings app), then 2. run Display Driver Uninstaller (DDU) to remove leftover bits and traces of the old GPU drivers before changing out the graphics card for a different model, especially if it's one from another brand. DDU works best if you run it while Windows is in safe mode. There's multiple ways to enter safe mode in Windows 10 but probably the easiest way to do it is going through the "advanced startup" menu accessed by clicking "Restart" in the start menu while holding down the shift key - see here for full instructions on that. — AP 499D25 (talk) 06:14, 18 July 2024 (UTC)[reply]
According to GPUOpen, it's a partially open-source middleware competitor to Nvidia GameWorks (proprietary and closed) and I would be prepared for changes. Do you rely on GPUOpen for some specific functionality? Does GameWorks claim to do the same thing? There may be work-arounds. Similarly, OpenCL (I'm not familiar with it) is an open standard maintained by both AMD and Nivida, among others in the Khronos Group. It seems at a quick glance that each manufacturer provides its own implementation. If you have been using OpenCL with AMD's implementation you may need to review Nvidia's offering to see what is possible: varying levels of compatibility with the latest standards are implemented with driver releases from individual manufacturers, for specific bits of hardware (see OpenCL#Vendor implementations). MinorProphet (talk) 17:08, 20 July 2024 (UTC)[reply]
The DirectML (aka Direct3D) API is part of DirectX. Although my brain exploded while trying to make sense of it all, there should be few problems for you (hah!) since this is M$'s own API, and individual manufacturers should interface with it correctly... MinorProphet (talk) 16:18, 21 July 2024 (UTC)[reply]
In the light of recent outages, I would back up my entire current boot drive with a disc-to-disc copy before making any changes, just in case everything went pear-shaped. MinorProphet (talk) 12:17, 22 July 2024 (UTC)[reply]

Differential equations

edit

Is a differential equation an equation that relates one or more unknown functions and their derivatives? Newton listed 3 types of DEs with the third one being partial derivatives. "Visualization of the heat transfer in a pump casing created by solving the heat equation." SU2CAfrazer123 (talk) 21:47, 17 July 2024 (UTC)[reply]

Yes, this is how our article Differential equation defines it.  --Lambiam 11:00, 18 July 2024 (UTC)[reply]

July 18

edit

Identify for Commons: these old Intel processors

edit

Hopefully @PantheraLeo1359531: doesn't mind but I saw they posted a question on Commons that I thought the Reference desk detectives may like to look at...

Mysterious Intel microprocessor/IC:
I recently bought 2 Intel processors (I couldn't resist, as they look so similar to the famous Intel 4004), but I don't know what the purpose could be. Looking at the ceramic package, I can imagine that the product was created maybe between 1972 and 1975. Maybe someone can give a hint?

If you know the answer you can reply here and I will relay to Commons. Commander Keane (talk) 00:17, 18 July 2024 (UTC)[reply]

I am very thankful for that ;), and looking forward to fantastic answers :D --PantheraLeo1359531 (talk) 07:07, 18 July 2024 (UTC)[reply]
I hunted around for some time, but drew a big blank other than similar chips offered on Ebay with no further description, and the pics on Commons. Is it even a CPU in the accepted sense? I could find absolutely no clues. MinorProphet (talk) 17:28, 20 July 2024 (UTC)[reply]
Several aeons ago (i.e. perhaps 15 years back), I reckon that yer average search engine would have served up at least ten sites hosting the exact information you seek. I remember realising with utter clarity one day how all my finely-honed search query results seemed to have changed almost overnight: and not for the better. Sadly, this has now become the New World Algorithm™ (available in taupe only, except for our special merch recipients—you know who you are.) All changed, changed utterly. Subscribe! Why not sign up for our weekly podcast, where influencers with the intellectual capacity of a piece of seaweed will babble shit for half an hour! I bet at least some of the sites are still there, (sort of, I'm not a betting man), but if there are any left, they are now not worthy of the search engine's notice, since not easily monetizable. I suspect that the old G**gle search engine is no longer available, even on the Wayback Machine. Is this the case? I think we should be told. </rant> MinorProphet (talk) 17:49, 22 July 2024 (UTC)[reply]
Thank you for the answers. I asked in another forum. One said that it might be DRAM chips. Probably certainly not a microprocessor. :) But it's a big problem when information about historical computer hardware disappears --PantheraLeo1359531 (talk) 14:46, 27 July 2024 (UTC)[reply]

University IP address blocked from editing, but something is weird

edit

Just earlier today, while I was at university, I stumbled upon a "This IP address is blocked from editing Wikipedia" banner when I clicked on the edit button of a Wikipedia article to check out some source code. I was like fine, there are thousands of users on this network and so quite inevitable someone's gonna do something bad leading to IP addresses / ranges being blocked from editing. All until I spotted something very odd regarding the IP range that is blocked, compared to the current IP address.

It said, the blocked IP address or range is 122.56.x.x/20, but then it also said, your current IP address is 202.36.x.x. I know quite a bit about subnets and how IPv4 addresses are divided up, so something just didn't seem right to me here! On a /16 to /23 IPv4 range, the left two groups of digits never change. 202.36.x.x is obviously not part of 122.56.x.x/20. I was thinking, how is this possible?!?

If I clicked on the "Talk" or "Contributions" buttons in the upper-right corner, indeed I would get the 202.36.x.x IP address and not one in the 122.56.x.x/20 range. Looking at the IP's contributions, there were like only a dozen or so edits, quite a few from 2019, but absolutely no edits from 2020-2023, and one edit in 2024. The talk page had this shared IP address banner at the top saying that it is registered to the University of Auckland.

According to WHOIS info, the 122.56.x.x/20 subnet is registered to a well-known large ISP for educational institutions here in New Zealand, and the 202.36.x.x IP address is registered under the name of the university itself, with even stuff like the building location address being of the university.

My guess is that the 202.36.* network is actually the university's own "private ISP" kind of network, used for communication between the different campus buildings, while the 122.56.* network is the public ISP that the university network is connected to and is using for internet.

Does anyone else know what's going on here? Man, this issue definitely explains a lot of those IP address unblock requests where the user claims that their IP address is never blocked but they are somehow unable to edit, getting a blocked from editing message. — AP 499D25 (talk) 10:19, 18 July 2024 (UTC)[reply]

202.36.xx.xx is not blocked, as you say. However, at 00:13, 4 August 2023 Ohnoitsjamie blocked 122.56.192.0/18 for two years. Who placed the block which appears on your screen? 2A00:23D0:7C1:5201:5558:3E19:B7AA:479 (talk) 12:45, 18 July 2024 (UTC)[reply]
It's probably going to be 122.56.192.0/20 (block range · block log (global) · WHOIS (partial)), though I don't think the specific network is particularly relevant. There seems to be two questions here. The first is why you are using two different networks. 122.56.x is a Spark network. In my experience, as well as schools, Spark deals a lot with mobile Internet (wifi, 5G, and so on), and I would guess that may be a factor. I also read in an undated article that Spark is helping 'upgrade' the university's networks. Being two local behemoths they're probably best buddies and all entangled anyway. This type of network splitting seems quite common to me, though whether the difference is explained by device, access point, content filtering, other routing considerations, or something more arbitrary, is beyond me. The university does use its own network for at least some Internet stuff - you can see it regularly edits Wikipedia.
The second and more interesting question, if I'm reading the original post correctly, is why a single block message is showing a different current address to the blocked address. Maybe there's some XFF weirdness, a badly coded template, or just a bug somewhere. I seem to vaguely recall autoblocks and cookie blocks being accused of causing this behaviour, but it seems unlikely, and you should get a different message. Someone over at WP:VPT might have some details. I'd suggest we need to see the block message (with obfuscated IPs if you prefer), if this applies. -- zzuuzz (talk) 14:19, 18 July 2024 (UTC)[reply]
And if you can reproduce the situation, also check which address is reported by WhatIsMyIPAddress.com.  --Lambiam 11:15, 19 July 2024 (UTC)[reply]
I occasionally have a similar experience, in which my talk page and contribution record show one IP, but when I press "publish" on an edit I get a block notice specifying another IP address. Two examples: I composed an edit under IP 92.8.218.114 but got a block notice on pressing "publish" for 92.8.218.47. I later composed an edit under IP 91.216.246.45, but on pressing "publish" got a block notice for 85.115.54.202, which is a different network. 79.78.116.149 (talk) 11:57, 19 July 2024 (UTC)[reply]
At home, the public IP address that appears in my router's info page is different to what I would get when going to one of those "What's my IP?" websites. I found out that my ISP actually uses CGNAT to get around IPv4 address exhaustion (learnt the hard way after hours of trying to port forward a game server to no avail). Though strangely, with my previous ISP, some other public IP finder websites would show a slightly different address, within the same subnet or neighbouring one. One thing for certain is the public IP that appears on my router page is never what I get or see on internet websites though. — AP 499D25 (talk) 12:59, 20 July 2024 (UTC)[reply]
Both of you are correct in that the blocked subnet that would appear on the blocked IP address banner was this one (blocked for several years by User:Ohnoitsjamie).
Here's the contribs page for the IP that wasn't blocked btw.
Though to add on to the strangeness, the day after I made this post, when I got back there and tried out the edit button again (on the same Wi-Fi network, using the same laptop, in the same building location), that time it didn't say my IP was blocked, and I seemed to be able to edit! I checked the IP address signature in talk page message previews and the address I would see was the non-blocked one.
I don't remember the block banner saying or mentioning "autoblocked", it was just "This IP address is blocked from editing". — AP 499D25 (talk) 13:08, 20 July 2024 (UTC)[reply]
Can't remember, if you're subject to an autoblock, does this get mentioned in the message? Autoblocks came to mind for me too, and I thought of the following pathway: (1) First range gets blocked. (2) This person, or someone else, tries to edit with it, and is subjected to the block. (3) Using the same computer, the person moves to the second range and is autoblocked. (4) You use the second range and are autoblocked. Nyttend (talk) 23:15, 23 July 2024 (UTC)[reply]
@Nyttend Last week an admin actually showed me this page, where you can see the various forms of system block messages that can be presented to users, and it includes MediaWiki:Autoblockedtext, which explicitly states in the top that "this account has been automatically blocked from editing". I swear the text on the top was just "This IP address is blocked from editing Wikipedia", the same as what I get on my home public IP address which is actually also blocked from editing after an ANEW report I filed because of some other user edit warring on it. I stand by my saying that "automatically blocked" wasn't mentioned anywhere in the block message banner. — AP 499D25 (talk) 09:22, 26 July 2024 (UTC)[reply]

July 19

edit

Oracle "plan" keyword

edit

I am trying to find documentation on the "plan" keyword in Oracle. I am not looking for "explain plan." That is completely different. What I am looking for is the plan keyword in this context:

   select a.id
   from a, b
   plan a
   where a.date between begindate and enddate

I have two different Oracle documentation books in PDF. In both if I search for pages with "plan" that do not have "explain", I come up with zero results. Searching the web, I only find explain plan, which has nothing to do with that as far as I can see. 75.136.148.8 (talk) 20:04, 19 July 2024 (UTC)[reply]

Can you tell us where you saw that code and what version of the software it was designed for? When I tried it, the word "plan" was interpreted as an alias for the table b reference, and the following "a" was considered a syntax error. Here is a fiddle. -- Tom N talk/contrib 00:45, 21 July 2024 (UTC)[reply]
It is used extensively in samples of code for a slide deck for Oracle Health EHR. Nearly every snippet of code has that syntax. I avoided using the actual table names because every slide has a huge "Propietary: Do Not Distribute" watermark on it, but I don't think the table names make a difference. 75.136.148.8 (talk) 11:22, 21 July 2024 (UTC)[reply]

July 20

edit

CrowdStrike outage - C-00000291*.sys file contents

edit

I'm curious as to what exactly caused the current CrowdStrike outage. According to this article,[1] the root cause is a single file located in C:\Windows\System32\drivers\CrowdStrike\ and is named C-00000291*.sys. Even though it ends with the sys extension, it is not a kernel driver. I don't have CrowdStrike on my PC and I'm just wondering if this file is a text file, and if so, has anyone compared the broken version with the fixed version and figured out the delta? A Quest For Knowledge (talk) 21:51, 20 July 2024 (UTC)[reply]

No, this is the Computing ref desk of Wikipedia, we hold ourselves aloof from such minor niggles as the proximate cause of bringing down half the world's IT systems because some CrowdShite CrowdStrike techie couldn't be arsed to check whether their patch worked before releasing it into the wild (and consequently half-asleep IT teams across the globe). It's a channel .sys file to update the Falcon Sensor software, not an AV update, which seems to have caused a page fault, try The Register at e.g.[2] It will be a BitLocker problem for many, since Win 10 removed the ability to boot directly into Safe Mode. Where are your 48-digit recovery keys?[3] On-prem, I hope, in a locked safe, to which there are two keys, since one has gone on holiday with the IT department's boss. MinorProphet (talk) 16:59, 21 July 2024 (UTC)[reply]
It is not a single file; there are different files whose name match the pattern.[4]  --Lambiam 20:57, 21 July 2024 (UTC)[reply]
Apparently the error was not in the patch itself. A logic error in an existing driver, CrowdStrike’s Endpoint Detection and Response (EDR) driver, running in kernel mode, had until recently gone unnoticed. It was triggered when parsing the content of the patched file, a configuration file. (For more detail, see [5], but the reporting on where the logic error was is a bit confused.)  --Lambiam 21:15, 21 July 2024 (UTC)[reply]
As is Microsoft's commitment to OS security.[6] MinorProphet (talk) 23:34, 21 July 2024 (UTC)[reply]
There were at least two procedural problems involved, in addition to the code error. The first procedural error was by CrowdStrike. The second procedural error was by the user community, with assistance by Microsoft. The error by CrowdStrike was inadequate pre-deployment testing. They should have had a small number of target systems on site running applications that resembled some of the operation-critical applications of their customer base. They should have deployed the patch to the target systems, and verified that the target systems came up cleanly. Obviously, they did not do that. They had some sort of system testing prior to deployment, but not a simulation of actual deployment to target systems.
Second, if I unplug my desktop computer from the UPS, and then plug it back in, it comes up to a blue screen that is not exactly a blue screen of death. I will call it a blue screen of birth. It says that Windows did not load properly the last time. It really means that Windows did not shut down properly, but that is a minor detail. It then gives me the option of restarting my PC normally, or of various options. Either the customers didn't have a USB drive or CD-ROM to boot from, or the startup sequence had been altered to make it more automatic to assume a normal restart. Something was wrong with the restart procedures of the customer base. Robert McClenon (talk) 02:47, 25 July 2024 (UTC)[reply]
An Australian firm overcame the restart mess: How a cheap barcode scanner helped fix CrowdStrike'd Windows PCs in a flash, from El Reg. MinorProphet (talk) 12:14, 25 July 2024 (UTC)[reply]
CrowdStrike has issued a "Preliminary Post Incident Review". According to this report, it was a bit more complicated (and, in my eyes, looking even worse for the company) than the story I linked to before. It was the result of four failures coming together. (1) an instance of an "InterProcessCommunication Template" was deployed whose content was invalid. This file was not itself a program, but contained data that was meant to be used to detect a certain attack. (2) The file was validated before deployment by an automatic process. The validating program contained a logic error, because of which it failed to detect that the content of the template instance was not valid. (3) As noted above by Robert McClenon, the deployment process of the company did not include a preliminary live test of the effect of deploying the file in a controlled environment, which would have made the issue manifest before worldwide deployment. This is strange; such live tests are standard across the industry. Omitting it and relying instead on static content validation for something so safety critical and meant to be used in kernel mode is indefensible. (4) The program interpreting the content did not itself attempt to validate the content independently. This is the most forgivable part; unless also developed independently it would likely have suffered from the same error as the validator of point 2. But as a result of interpreting the invalid content it executed an out-of-bounds memory read triggering an exception while in kernel mode, resulting unavoidably in the blue screen of death. This is hard to forgive; the interpreter should have been hardened to do a bounds check before performing a potential out-of-bounds operation.  --Lambiam 18:41, 25 July 2024 (UTC)[reply]
This article "The months and days before and after CrowdStrike's fatal Friday" is subtitled "In the short term, they're going to have to do a lot of groveling". Understatement or what, especially since the CEO of CrowdStrike was the CEO of McAfee when an update borked millions of of PCs worldwide in 2010. Useless managers? They self-promote themselves upstairs. The update was only in operation for 78 minutes, there were many more millions of PCs which escaped. Plenty of links within the article for b/g info, and the comments are fairly negatory and forceful. As I understand it, M$ was forced to allow kernel access to 3rd-party suppliers since it couldn't be trusted to keep tabs on the security of the monopoly of its own OS. 'Page fault in nonpaged area' (0x50) has existed as a concept since bounds-checking was a compulsory module of Coding 101. MinorProphet (talk) 21:59, 25 July 2024 (UTC)[reply]

July 22

edit

How apartheid affected people's lives and how people respond

edit

Group areas act(1950) 41.114.250.185 (talk) 19:01, 22 July 2024 (UTC)[reply]

Your question is unclear but have you read our extensive article on apartheid? Shantavira|feed me 19:11, 22 July 2024 (UTC)[reply]
Please do your own homework. 41.23.55.195 (talk) 06:40, 23 July 2024 (UTC)[reply]
@41.114.250.185: The OP's first language is obviously not English, and possibly not their second language either. How many languages do you speak? Why don't you create a WP account rather than carping anonymously from the sidelines? I'm sure you failed some homework assignments yourself. MinorProphet (talk) 19:14, 23 July 2024 (UTC)[reply]
This is the Computing Reference Desk. HiLo48 (talk) 06:49, 23 July 2024 (UTC)[reply]
Well spotted, top marks! Hurrah! This is indeed the Computing Reference Desk. In 1995 (nearly 30 years go) I was fortunate and privileged enough to be in the group of the very first technical instructors from IBM to present that company's latest hardware offerings (laptops, desktops and PC servers) after the general international embargo ended, the same year when SA won the Rugby World Cup at home. I wasn't employed by IBM directly. It was a joyous occasion, taking place immediately after the 1994 South African general election. South Africa was open to the world again. But all the white people drove around in cars, including the taxi-drivers, and all the black people walked. That's how apartheid affected people's lives. All the techies I taught were white as well. MinorProphet (talk) 19:14, 23 July 2024 (UTC)[reply]
"At least that's slightly relevant to this page. I'll keep it that much off topic by saying that in 1994 I (on behalf of my employer at the time) was one of the first customers for IBM (not Lenovo) ThinkPads in Australia. I still have two of them and they still work!!!! (On mains power of course.. The batteries are long gone.) HiLo48 (talk) 21:24, 23 July 2024 (UTC)[reply]
Yah, but can you still source those replacement red nipples? They wear out, you know. The feel of a brand new one under your fingertip is simply astounding. MinorProphet (talk) 02:28, 24 July 2024 (UTC)[reply]
Do you mean the red nipples that everyone I know immediately removes? Martin of Sheffield (talk) 14:22, 24 July 2024 (UTC)[reply]
We have articles on the Group Areas Act, and apartheid, and Category:Apartheid in South Africa lists many other relevant articles. If you have more questions, then the Humanities Reference Desk is probably the best place to ask. DuncanHill (talk) 23:18, 24 July 2024 (UTC)[reply]

July 24

edit

Can't Back Up System Files

edit

I have a Dell Inspiron 3910 with 16 GB of RAM and approximately 216 GB of solid-state storage as the C: drive. I just got a pop-up message saying something to the effect of "We can't back up your system files". It says that if I free up some space on my "hard drive" (which is hard because it is solid-state), it will be able to back up my system files. I don't recall having seen that message before. I do see that I have 10.1 GB free on my solid-state C: drive out of 216 GB. I also see that my pagefile.sys on the C: drive has grown to 26.6 GB, which is what is taking up the space. My question is: What system backup function is there that was trying to back something up, and was unable to back something up? The message was a pop-up, and I can't bring it back, although maybe that isn't important. So: What system process was trying to back something up to my C: drive and didn't have space for the backup? I will, in the very near future, be getting something like WINDIRSTAT to get a better view of how the 206 GB is being used, to see what if anything to move to my F: drive. So what is trying to back up my system files automagically? Robert McClenon (talk) 18:15, 24 July 2024 (UTC)[reply]

You've probably told us before, but which version of Windows are you using? Is there a load of Dell stuff in your Start Menu? Do you ever use such apps? Could be a cause. Maybe try disabling them sequentially in Services via Windows Task Manager. To reduce disk usage, perhaps try the resident Disk Cleanup app. Try using a fixed-size paging file. Delete the contents of your Temp folder and Recycle Bin. Purge your browser's cache frequently (In Firefox, press Ctrl+Shift+Delete, select 'Cache' only.) Your mileage may vary. MinorProphet (talk) 22:19, 25 July 2024 (UTC)[reply]
User:MinorProphet - You have addressed a question which I might have reasonably been asking, but not the question that I was asking. You have told me how to free up space on my disk. I already have plans for freeing up space on my disk. What I am asking is what process was trying to back something up on my C: drive. But thank you for useful general advice. Robert McClenon (talk) 07:06, 27 July 2024 (UTC)[reply]
I am running Windows 11, and I think that I have the personal version. (Are there a personal version and a business version?) Robert McClenon (talk) 07:06, 27 July 2024 (UTC)[reply]

Fourier transform

edit

Can the Fourier transform, using the Fourier series, analyze a function f(x) on a bounded interval x whose members are -P/2 and P/2 for some positive real number P, with frequency being the reciprocal of period (f=1/T or T=1/f), be programmed in PL/1? Afrazer123 (talk) 23:43, 24 July 2024 (UTC)[reply]

no, as the frequency goes to infinity for this, and the coefficients are real numbers that can be calculated, but not with perfect accuracy. However PL/1 could be used to approximate a finite number of coefficients approximately. see Square wave#Fourier analysis for the calculation. Graeme Bartlett (talk) 12:47, 25 July 2024 (UTC)[reply]
The question is not entirely clear. Does T stand for the variable x or for the constant P?  --Lambiam 13:45, 25 July 2024 (UTC)[reply]
T doesn't stand for the variable x. T does stand, however, for the constant P. Afrazer123 (talk) 02:55, 28 July 2024 (UTC)[reply]

July 25

edit

Are there apps or any software, that can identify one's accent?

edit

E.g, software that can identify a person's native language, when they are currently speaking in a non-native language (e.g. English), rather than in their native language we want to identify.

Yesterday, I presented this question at the language reference desk. However, no one has given me a positive answer yet, except for a possible direction via AI, but without a certain answer. 2A06:C701:7B31:C100:7D63:C50F:C3A5:9744 (talk) 10:18, 25 July 2024 (UTC)[reply]

You received a comprehensive answer at the language desk. The answer is no. Shantavira|feed me 11:48, 25 July 2024 (UTC)[reply]
AI is used for pattern matching and classification. If there is some pattern that classifies speech as having a specific accent, AI can identify the pattern and classify the accent. AI is not magic. It won't do any more than identify a pattern using one of the many methods of pattern matching and then classify using one of the many ways of clustering and classification. 75.136.148.8 (talk) 12:12, 25 July 2024 (UTC)[reply]
A more precise answer is that no respondent here is aware of the existence of such an app. Perhaps the NSA has developed one but is keeping it under wraps. If so, how would we know?  --Lambiam 13:33, 25 July 2024 (UTC)[reply]
It is likely that, for the moment, it is far easier and cheaper to employ non-Artificial Intelligences, i.e. people, with linguistic expertise that enables them to make such identifications. This of course would only apply to specific instances – an AI-like application would be needed for automatic surveillance on a mass scale. {The poster formerly known as 87.81.230.195} 94.2.67.235 (talk) 00:18, 28 July 2024 (UTC)[reply]

How to automatically search and replace text in Linux CLUI in a multi-lined way?

edit

We can automatically search and replace single-lined text in Linux CLUI with awk and sed, but I need a way to do it multi-lined.

  • File A has several HTML structures.
  • File B has this HTML structure:
    <footer class="site-footer">
      <div class="site-footer__inner container">
        {{ page.footer_top }}
        {{ page.footer_bottom }}
      </div>
    </footer>
  • File C has this HTML structure:
    <footer class="site-footer">
      <div class="site-footer__inner container">
        {{ page.footer_top }}
        {{ page.footer_bottom }}
      </div>
      <span class="globalrs_dynamic_year">{{ 'now' | date('Y') }}</span>
    </footer>

How to automatically search in file A and if it contains the text of file B then replacing that text with the text of file C?

How would you do this with C/Perl/Python/PHP/Node.js or something else? 103.199.70.159 (talk) 19:06, 25 July 2024 (UTC)[reply]

While it would be trivial to do this in Python or any other reasonable programming language, if I wanted to do this with a script, my approach would be:
  1. . Convert all three files to a version which eliminates newlines, using sed. For convenience, I would replace the newline character with some character or string which would not occur in the HTML, call it '~' (tilde).
  2. . Now add a newline to change the tilde in every occurrence of "</footer>~" to a newline in each of the three converted files. Do the same for "~<footer class="site-footer">". You end up with files where the html of interest is on a single line.
  3. . Use sed to do the substitution of the single line file C text to replace the single line file B text in the single line file A text.
  4. . Use sed to convert single line file A back to the original formatting by replacing '~' with newline.
This won't work if files B and C are not marked with the exact footer head and tail as you have shown.-Gadfium (talk) 20:46, 25 July 2024 (UTC)[reply]
In any programming langauge, A, B, and C are just text. So, you use a string replace function. In C++, it is (from memory) replace(A,B,C);. In Perl (again from memory), it is A=~s/B/C/;. In PHP, it is $A=str_replace($B,$C,$A);. In Python, it is A = A.replace(B,C);. In Node.js it is A = A.replace(B,C) as well. Note that in a programming language, a string is just a string of characters. It doesn't care if there are newline or return characters in it. So, replacing a substring replaces it all, including the return and newline characters. But, the text has to match perfectly. For example, if A is using two spaces for indentation and B is using tab characters, it won't match. Similarly, if one uses all lower case tag names and another uses all upper case tag names, it won't match. In that case, you need to reformat the text so it is all the same or use regular expressions. 75.136.148.8 (talk) 13:31, 26 July 2024 (UTC)[reply]
The Unix/Linux utility sed can do this; see sed § Multiline processing example. The search ["sed" multi-line replace] gives some more examples.  --Lambiam 23:01, 26 July 2024 (UTC)[reply]
Since the OP asked for a Perl solution, here is a simple one.
my $orig = `cat $ARGV[0]`;
my $repl = `cat $ARGV[1]`;
my $text = `cat $ARGV[2]`;
$text =~ s:$orig:$repl:g;
print $text;
CodeTalker (talk) 18:57, 27 July 2024 (UTC)[reply]

July 27

edit

Fast Fourier transform

edit

Can a relational database, with its set of rows and columns, support the computerization of the Fast Fourier transform ? Afrazer123 (talk) 04:40, 27 July 2024 (UTC)[reply]

No. The FFT algorithms require computations based on the indices. This is not supported by relational databases. The notion of index is in fact alien to the relational model, which is essentially an unordered set of tuples (the rows) of named values (as identified by the row names). One can add the index as a key, basically an extra column, but this does not help. The SQL-type languages that come with such databases also do not support the recursion used by FFT algorithms.  --Lambiam 13:18, 27 July 2024 (UTC)[reply]
Thanks for the reply. How pulsating! It seems there's a discharge of some sort which apparently isn't taken into consideration by the RDB while using an extra column (key) only exacerbates things. Also, SQL appears to be to confining, off-limits for the algorithm's recursion. Afrazer123 (talk) 02:09, 28 July 2024 (UTC)[reply]

July 28

edit

Turning Off Ad Blocker

edit

If I am using Firefox and Windows 11, and a web site asks me to turn off my ad blocker, and I don't know what ad blocker I am using, how do I determine what ad blocker I am using, so that I can turn it off? This is sort of a strange question, because I don't want to deal with ads, but I would rather just ignore the ads than deal with web sites that aggressively fight ad blocking. I do have Norton Safe Web. I don't know if it tries to block ads. Robert McClenon (talk) 00:23, 28 July 2024 (UTC)[reply]

Hi, Robert McClenon! In Windows 10 which I'm on, such blockers appear (with other things) in a drop-down list of 'Extensions' found by clicking a jigsaw-puzzle corner piece at the top right corner of the Firefox window. I had a similar problem (with YouTube) until I discovered that Malwarebytes had added ad blocking to its functions. Hope this helps. {The poster formerly known as 87.81.230.195} 94.2.67.235 (talk) 20:54, 28 July 2024 (UTC)[reply]
The better way to get around this is to reconfigure your RAM firewall to exclude cloud analytic encryption on your virtual platform. Pretty simple fix, will take about 30 seconds. Jidarave (talk) 21:49, 28 July 2024 (UTC)[reply]
The above post is nonsensical. Philvoids (talk) 22:53, 28 July 2024 (UTC)[reply]
This is probably the same troll who reappears here periodically to post gibberish like this before getting blocked. CodeTalker (talk) 06:52, 29 July 2024 (UTC)[reply]
It looks like perfectly legit response from AI. Maybe someone is training their bot to replace human editors on Wikipedia. 75.136.148.8 (talk) 12:50, 29 July 2024 (UTC)[reply]
The responses by LLMs tend to read like something a human could actually have written as a reasonable response, using terms from the question. This response does not. It looks contrived to sound impressive while making no sense whatsoever.  --Lambiam 22:51, 29 July 2024 (UTC)[reply]
I'm glad it's confirmed to make no sense, because although I didn't understand it, I feared that to be because of my limited knowledge of IT.
This sidetrack being dealt with, can anybody give a better answer than mine to the OP's query? {The poster formerly known as 87.81.230.195} 94.2.67.235 (talk) 18:03, 30 July 2024 (UTC)[reply]
A long shot, but are you using NoScript in Firefox by any chance? It's a kind of an ad-blocker on steroids and because it completely blocks scripts (unless you allow them) it can be difficult to know what's getting blocked. Using the "temporary allow" function usually gets me past things, but sometimes I end up having to switch to incognito mode (which I have setup to run without NoScript). Matt Deres (talk) 16:02, 31 July 2024 (UTC)[reply]
In Firefox's Privacy and Security settings, if you have tracking protection set to Strict, try setting it back to Standard. This should help. win8x (talking | spying) 20:19, 31 July 2024 (UTC)[reply]

July 29

edit

Can you delete and then undelete your twitter account?

edit

This is about the incident with Pete Souza and an intactly-eared Donald Trump photo.[7] Souza apparently deleted his twitter account after both he and the Trump photographer took heat. Question: can he undelete it later, and get his old tweets back? I mean using normal Twitter features. Presumably someone famous like Souza could get Elon's, um, ear for a special request, but let's not count that. Thanks. 2602:243:2008:8BB0:F494:276C:D59A:C992 (talk) 23:39, 29 July 2024 (UTC)[reply]

According to [8] you have 30 days to reactivate your account. After that, it cannot be recovered. RudolfRed (talk) 15:50, 30 July 2024 (UTC)[reply]


July 31

edit

Alternating between dark and light mode?

edit

I work on Google Sheets spreadsheets all day and I don't like how bright light mode gets during sunset. I don't like the look of dark mode either, so I'd prefer to only use dark mode during the sunset time or night. Does anyone know if this is a thing that Google has allowed for? Is there an extension that does this? I could try making a Chrome extension but this is not something I have done before. ―Panamitsu (talk) 05:46, 31 July 2024 (UTC)[reply]

Google Maps does this automatically when in satnav mode, but I don't know if it is extendable to other products. -- Verbarson  talkedits 10:30, 31 July 2024 (UTC)[reply]

August 1

edit

Science

edit

July 17

edit

Identify some trees?

edit
 
some kind of pine

This is a small park on the Homer Spit. We mostly have Sitka Spruce as far as evergreens around here, so I suspect these are specimen trees, probably some kind of pine tree but I'm not sure what kind. They aren't very big but that may be because they are out of their usual range. Just Step Sideways from this world ..... today 20:37, 17 July 2024 (UTC)[reply]

Looks like shore pine. --Amble (talk) 22:30, 17 July 2024 (UTC)[reply]
After ec, shore pines? Mikenorton (talk) 22:32, 17 July 2024 (UTC)[reply]
That does seem likely, the needles and cones look the same. Just Step Sideways from this world ..... today 18:42, 18 July 2024 (UTC)[reply]

July 18

edit

Identify some more trees?

edit
 

Scene in Laramie, Wyoming, USA. I'd like to put it into Commons categories for the trees along the street — particularly the big prominent one near the centre — but I don't know anything about this kind of thing. Nyttend (talk) 08:04, 18 July 2024 (UTC) PS, I was guessing blue spruce for the big one at the centre, but the shape is quite different from those of the trees pictured in that article. Nyttend (talk) 08:20, 18 July 2024 (UTC)[reply]

A zoom-in on the image shows some distinctive spruce cones, and it is certainly blue-ish (Caveat: the blue spruce is an uncommon specimen tree in the UK and I'm not sure that I've ever seen one in person). Alansplodge (talk) 10:53, 18 July 2024 (UTC)[reply]
Speaking as a regular WP:Wikiproject Plants contributor, I would prefer that users didn't assign species categories to images such as this unless they were absolutely certain of the identification. Ideally, the image would include the label from the arboretum or botanical garden. Abductive (reasoning) 21:11, 20 July 2024 (UTC)[reply]

Lorentz transformations.

edit

For deriving the Lorentz transformations, our article Derivations of the Lorentz transformations relies on their linearity. How do we know they must be linear? Our article answers: Since space is assumed to be homogeneous, the transformation[s] must be linear. I wonder, how their linearity is deduced from the homogeneity of space, before we've found how they will look like...

For the time being, I'm adding an Einsteinian source for this claim in the article, even though I don't know how Einstein derived this claim. HOTmag (talk) 23:41, 18 July 2024 (UTC)[reply]

The Lorentz transformations are linear transformations by definition. The linearity comes from their domain being a linear "Newtonian" spacetime isomorphic to the product of Euclidean 3-space and a linear time axis.  --Lambiam 09:48, 19 July 2024 (UTC)[reply]
Thanks. However, rather than assuming linearity - from the very beginning (as you do), Einstein's claim I've quoted from our article - derives linearity - from the homogeneity of spacetime. My question was: how can the quote be justified. HOTmag (talk) 10:02, 19 July 2024 (UTC)[reply]
This is the article in which Einstein introduced the special theory of relativity. His spacetime is flat, and the reference frame of a stationary observer is basically the same as for Lorentz, so space is an Euclidean 3-space. The "homogeneity" is that the laws of physics are invariant under an isometric transformation of space. Just homogeneity is not enough; to reach the conclusion, the flatness is essential.  --Lambiam 00:19, 20 July 2024 (UTC)[reply]
Let's assume that the space is flat, and that the laws of physics are invariant under an isometric transformation of space. How do you infer from these assumptions, that the Lorentz transformations (which are actually not isometric) are linear? This is what I can't understand yet... HOTmag (talk) 18:35, 20 July 2024 (UTC)[reply]

July 19

edit

Virgo Cluster in Observable Universe

edit

So I get that in around 100 billion years, our observable universe will be limited to the Local Group due to Hubble expansion (bummer), but would that include the Virgo Cluster? Some sources say that they would join due to gravity (https://astronomy.swin.edu.au/cosmos/V/Virgo+Cluster, https://en.wikipedia.org/wiki/Observable_universe) but I'm also reading responses saying the Hubble expansion is more powerful and therefore Virgo would end up outside of the observable universe. I realize there a lot of variables that we don't know or could change. 184.96.249.124 (talk) 02:28, 19 July 2024 (UTC)[reply]

Virgo cluster is receding from us unfortunately. In 100 billion years it would be more than 300 million light years away. Ruslik_Zero 19:24, 19 July 2024 (UTC)[reply]
The fact that it is receding now does not necessarily imply that it will continue to recede forever. The Andromeda galaxy and the Milky Way once receded from each other but have by now turned around and are approaching each other. Every bound object (more exactly, "virialised object"), say a cluster or group of galaxies, has formed out of an initially expanding, but overdense region of the Universe that through the action of gravity has by now recollapsed (a "little crunch", if you will). Throwing a stone up in the air is a pretty good analogy: When I do that on earth, the stone will first move upwards (recede from me) but then it will invariably turn around and fall back down. When I do it on a sufficiently small asteroid, throwing the stone with the exact same force, it may well escape to infinity and never come back. What the analogy does not capture is the effect of Dark Energy, which prevents structures on very large scales from recollapsing, and, as I wrote in another recent thread, I do not know where the limit is. The article Laniakea Supercluster suggests that this structure is not bound, i.e. will not recollapse. You may want to have a look at the references therein. --Wrongfilter (talk) 08:44, 20 July 2024 (UTC)[reply]
Thank you but I know all of these. However it is well known that Virgo supercluster (as any supercluster) is not gravitationally bound. The present recessional velocity of the Virgo cluster is more that 1000 km/s. There is not nearly enough mass to even slow this expansion rate. If fact this velocity is already increasing due to the cosmic repulsion. Ruslik_Zero 21:11, 21 July 2024 (UTC)[reply]

COVID - Natural Immunity vs Vaccination

edit

I thought that natural immunity via infection from COVID was roughly comparable to immunity via vaccination. However, I just read this article, Study suggests reinfections from the virus that causes COVID-19 likely have similar severity as original infection, which seems to suggest that natural immunity via infection doesn't provide much protection - at least in regard to severe infections. Am I understanding the article correctly? A Quest For Knowledge (talk) 06:07, 19 July 2024 (UTC)[reply]

That would match the experience of people I know who've caught it twice. HiLo48 (talk) 06:12, 19 July 2024 (UTC)[reply]
I'm not sure the results of the study support a statement like "immunity via infection doesn't provide much protection - at least in regard to severe infections" given that only about "a quarter of individuals with either a moderate or severe first infection coinciding with hospitalization also were hospitalized at the time of reinfection", although the difference might not be due to their immune responses. Sean.hoyland (talk) 06:56, 19 July 2024 (UTC)[reply]
The unpredictability of viral infection and immune response at the individual level is quite impressive. I got an AstraZeneca shot, a Moderna shot about 6 months later and a Moderna booster 6 months after that or thereabouts and I didn't experience any symptomatic Covid infections, despite being surrounded by people with active infections quite often. Then finally, a couple of months ago, I had my first symptomatic SARS-CoV-2 infection. And despite having lived through decades of dengue seasons and thousands of mosquito bites in various places during dengue outbreaks without any symptomatic dengue infections, I had my first dengue infection a month after Covid. Hats off to scientists trying to make sense of these immensely complex systems. Sean.hoyland (talk) 07:47, 19 July 2024 (UTC)[reply]
SARS-CoV-2 is a quickly mutating virus with many variants. The immunity provided after infection by one variant is strongest for that specific variant. Evading immunity is a driving factor in the evolution of the virus, which is why we may expect recurring waves, as we are used to for influenza. Next to reinfection with a different variant, the immunity after infection or after vaccination wears off after a couple of months. I have taken all shots and booster shots as soon as they were made available, yet I have been symptomatic twice. Since the vaccines are developed mostly for specific variants, and one usually does not determine the specific variant responsible for the infection of a symptomatic patient, I doubt that there are studies that have determined whether vaccination provides the same level of protection as that after infection. I'm not sure, but so many factors play a role that I think it will be very difficult to collect the data necessary for drawing a conclusion.  --Lambiam 09:29, 19 July 2024 (UTC)[reply]
The immune system having a sort of predictive system via somatic hypermutation complicates matters. I think labs at La Jolla Institute for Immunology have done some work on comparing the immune responses to infection vs vaccination, immune memory etc. Sean.hoyland (talk) 09:57, 19 July 2024 (UTC)[reply]
See also COVID-19 immunity: Infection compared with vaccination (Feb 2022) from the British Society for Immunology. Alansplodge (talk) 10:49, 19 July 2024 (UTC)[reply]

Is the relativistic transformation, between two non-rotating non-inertial reference frames, linear?

edit

HOTmag (talk) 12:44, 19 July 2024 (UTC)[reply]

It is locally linear but coefficients will depend on coordinates. Ruslik_Zero 19:29, 19 July 2024 (UTC)[reply]
Locally linear, so it's not linear. Thank you. HOTmag (talk) 18:15, 20 July 2024 (UTC)[reply]
https://en.wikipedia.org/wiki/Lorentz_transformation says Frames of reference can be divided into two groups: inertial (relative motion with constant velocity) and non-inertial (accelerating, moving in curved paths, rotational motion with constant angular velocity, etc.). The term "Lorentz transformations" only refers to transformations between inertial frames, usually in the context of special relativity. Greglocock (talk) 04:25, 20 July 2024 (UTC)[reply]
I'm not asking about the Lorentz transformations. HOTmag (talk) 18:16, 20 July 2024 (UTC)[reply]

July 20

edit

Element bonding

edit

I learnt in school that different types of elements can bond in different ways, like non-metal atoms bond covalently together while a non-metal atom with a metal atom bond ionically, but why is that? Also, what factor(s) determine why some elements (like hydrogen) share/transfer 1 electron, while others (like carbon and nitrogen) and share/transfer 2 or 3 electrons? Bestfweind (talk) 04:58, 20 July 2024 (UTC)[reply]

More than anything else, it's the number of valence electrons that each atom has, and the number that it needs. Hydrogen needs just one extra valence electron to have a full outer shell, so two hydrogens share a single electron. Nitrogen needs three, so two nitrogens share three electrons. Nyttend (talk) 07:00, 20 July 2024 (UTC)[reply]
Also, you may be thinking of common compounds such as table salt, sodium chloride, which involves an alkali metal (at the left end of the periodic table) and a halogen (near the right end). Like the other alkali metals, sodium has just one valence electron, so it's highly electropositive (ready to give up an electron), and like the other halogens, chlorine needs just one valence electron, so it's highly electronegative (ready to gain an electron), and it's extremely easy to create an ionic bond between atoms that are both highly electropositive/negative and that need to give/take the same number. Nyttend (talk) 07:06, 20 July 2024 (UTC)[reply]
The bond is better described as ionic when one atom is much more electronegative than the other. In something like table salt you can approximate the valence electron of Na as having passed completely to the Cl, although in reality it's not a complete transfer. Covalency happens when the electronegativities are similar. It is all a continuum from one to the other: Na–Cl will be more ionic than Li–Cl, which will in turn be more ionic than C–Cl.
The first-row elements B–Ne have a larger tendency towards multiple bonding than their higher homologues in lower rows, because their atoms are so small that non-bonding pairs of electrons result in significant repulsion. It's less of an issue for things like Al–Ar. Double sharp (talk) 07:24, 20 July 2024 (UTC)[reply]
Our Double bond rule is quite light on cited content for the concept itself, and gives (with cite, that I can't access) a different rationale. Please update if you've got a ref. DMacks (talk) 03:02, 21 July 2024 (UTC)[reply]
doi:10.1002/anie.198402721. I've updated the page. :) Double sharp (talk) 03:46, 21 July 2024 (UTC)[reply]
Thanks! I keep meaning to split up that hybrid (ha!) of an article, but obviously keep never getting around to it. DMacks (talk) 04:37, 21 July 2024 (UTC)[reply]
@DMacks: Also, I added a link to double bond rule in kainosymmetry, where this (together with other unusual features of first-row elements) is discussed. Double sharp (talk) 06:10, 23 July 2024 (UTC)[reply]

Big Bang vs supernatural creation

edit

When there are evidence in support of Big Bang, why do people believe in God? CometVolcano (talk) 16:41, 20 July 2024 (UTC)[reply]

What better explanation of the Big Bang is there? You did note who first proposed it (second sentence of article)? -- Verbarson  talkedits 17:28, 20 July 2024 (UTC)[reply]
It also depends on how one defines "God". ←Baseball Bugs What's up, Doc? carrots18:04, 20 July 2024 (UTC)[reply]
God could have started things off with a big bang. The two are not mutually exclusive. There are physics Nobel laureates who are Christians, e.g. Charles H. Townes. Clarityfiend (talk) 02:40, 21 July 2024 (UTC)[reply]
Also renowned cosmologist and Quaker George F. R. Ellis. Clarityfiend (talk) 03:32, 21 July 2024 (UTC)[reply]
See Religious interpretations of the Big Bang theory. Double sharp (talk) 04:09, 21 July 2024 (UTC)[reply]
If a belief is not falsifiable, evidence seems irrelevant. Sean.hoyland (talk) 11:41, 21 July 2024 (UTC)[reply]
If image sleuths were to discover that the unexplained "Little Red Dots" observed by the JWST,[9] as seen from the solar system against the cosmic expanse, actually spell out a message in a well-known two-dimensional barcode, namely, "Of course I exist, you blockheads. Yours forever, God", would this seem relevant evidence?  --Lambiam 12:09, 22 July 2024 (UTC)[reply]
Carl Sagan postulated a somewhat similar idea in one of his books. However, there's also the logic problem postulated in the Hitchhiker's Guide. ←Baseball Bugs What's up, Doc? carrots19:40, 22 July 2024 (UTC)[reply]
Most people who believe in a god believe in the same one as their parents, implying that they do so because of what some would call cultural influences, but which I call love bombing and brainwashing from birth. HiLo48 (talk)
"Of course I exist, you blockheads. Yours forever, God" - I guess it could be evidence of something, trickster djinns, the breaking of the 4th wall between simulator and simulated etc. They could simply arrange in person book signings for their best-selling books, go on some podcasts etc. Sean.hoyland (talk) 07:35, 23 July 2024 (UTC)[reply]
Nay I say! No sceptic can deny the uncontrovertably unfalsifiable revelation that the number pi found in epiphany when expanded to sufficiently many decimal places actually contains the message "Of course I exist, you blockheads. Yours forever, God", perfectly encoded in ASCII, itself proof of a divine miracle predating knowledge that humans gained no earlier than 1963 as attested by these witnesses. Philvoids (talk) 20:54, 24 July 2024 (UTC)[reply]
I will add that the big bang is what happened after the creation, but was not the singularity that was the start of the Universe. So creation is at time 0. Big bang in the interval (0,CMB visibility] Graeme Bartlett (talk) 01:59, 23 July 2024 (UTC)[reply]
That claim definitely needs a Reliable source. HiLo48 (talk) 02:11, 23 July 2024 (UTC)[reply]
It is stated in our article on the Big Bang, with sources (see the "Singularity" section), that some authors use "Big Bang" to just mean the initial singularity and others include some periods after it. I'd argue that the name Big Bang nucleosynthesis sort of implies that at least the first few minutes (when deuterium, helium, and lithium are synthesised) can be taken as part of the "Big Bang". Double sharp (talk) 06:03, 23 July 2024 (UTC)[reply]
And Creation? There is no science behind that at all. HiLo48 (talk) 06:36, 23 July 2024 (UTC)[reply]
Graeme Bartlett said that "creation is at time 0". That's a clearly scientific definition, though I suppose you can call it instead the initial singularity, if you like that name better. I suppose physics before the quark epoch is not yet empirically testable, in the sense that current accelerators cannot reach the energies needed to explore electroweak unification, but that's a matter of our current level of technology. Double sharp (talk) 08:47, 23 July 2024 (UTC)[reply]
It's also a matter of theory — beyond a certain point we need a theory of quantum-gravity to proceed, and that does not exist yet. The "Big Bang" singularity is an extrapolation based on known physics, which is known (pun...) to not be applicable to those conditions. So the evidence for a "Big Bang" is not quite what OP appears to imagine it to be. --Wrongfilter (talk) 09:58, 23 July 2024 (UTC)[reply]
A very good point indeed. My apologies; once quarks stop being confined, it's not something I know much about anymore. :) Double sharp (talk) 03:29, 24 July 2024 (UTC)[reply]
A common mistake, made by people at both sides of the controversy, is that God and religion are not meant to provide answers about the physical world. God is not meant to be an answer to the origin of the universe, the origin of life or any such question that requires a scientific answer. “Give to Caesar what belongs to Caesar, and to God what belongs to God” Cambalachero (talk) 03:45, 24 July 2024 (UTC)[reply]
And that statement is just an opinion or point of view. What does "meant" mean? See Proximate and ultimate causation and Meaning of life. Graeme Bartlett (talk) 07:54, 25 July 2024 (UTC)[reply]
How about: A great deal of unnecessary conflict has been created by people who assume that 2000-3000 year old Hebrew literature is the same genre as nineteenth-century scientific journals, and can therefore be understood using exactly the same assumptions and literary methods. -- Verbarson  talkedits 20:54, 25 July 2024 (UTC)[reply]
Yes, the thousands years old Hebrew literature was written at a time when humans understanding of the nature was rudimentary and primitive, when people thought the Earth was the center of the universe and concept of atoms, subatomic particles and periodic table did not exist. --CometVolcano (talk) 16:01, 26 July 2024 (UTC)[reply]
Democritus: Say what? Clarityfiend (talk) 08:41, 27 July 2024 (UTC)[reply]
Aristarchus: Hmm? Double sharp (talk) 08:45, 27 July 2024 (UTC)[reply]
From well before these guys. Graeme Bartlett (talk) 12:24, 27 July 2024 (UTC)[reply]
Indeed. *facepalm* Double sharp (talk) 14:04, 27 July 2024 (UTC)[reply]
At that time, humans had no understanding of the nature of the universe. They did not know there are many galaxies in the universe. It was not untill the Great Debate (astronomy) humans did not know that Milky Way is one of billions of galaxies in the universe. --CometVolcano (talk) 19:25, 27 July 2024 (UTC)[reply]
And a generation (or two) ago scientists hadn't noticed the 95% of the universe we now call dark energy and dark matter. Just imagine what our grandchildren will discover (and how condescending they might be about our ignorance). -- Verbarson  talkedits 21:48, 27 July 2024 (UTC)[reply]
+1 Double sharp (talk) 04:07, 28 July 2024 (UTC)[reply]

Since it doesn't only refer to an object's length, but also to any abstract length, so why isn't this general fact mentioned in the lead of the article Length contraction, nor in (most of) the common literature discussing the phenomenon of length contraction?

Explanation: The lead of our article claims the following: If we (travel on a train and) observe a moving "object" (e.g. a moving building), for which   is the object's length measured in the object's reference frame, then we will meausre the object's length to be shorter, i.e.   So, why does the lead of our article (as well as the professional literature discussing this phenomenon) only mention an "object", even though this is also true for abstract lengths? Just substitute "distance" (e.g. between two buildings) for "object", and you'll get the following true analogous sentence: If we (travel on a train and) observe a moving distance (e.g. a distance between two co-moving buildings), for which   is the distance's length measured in the distance's reference frame (i.e. in the reference frame of the buildings), then we will meausre the distance's length to be shorter, i.e.  

Indeed, our article contains some hints, e.g. in the chapter "Basis in relativity": Here, "object" simply means a distance with endpoints that are always mutually at rest, i.e., that are at rest in the same inertial frame of reference. Yet, this general fact is neither mentioned nor implied, neither in the lead - nor in the chapters that prove the length contraction - nor in (most of) the common professional literature. HOTmag (talk) 19:26, 20 July 2024 (UTC)[reply]

The phrase "any length in the moving object's reference frame" is ambiguous. Supposedly it is the length of some (other) object. Is it co-moving with the first object? By "a moving object measures (some length)", do you mean, "an observer to whom this moving object is at rest measures (some length)"? It is not very clear.  --Lambiam 20:20, 20 July 2024 (UTC)[reply]
There once was a fellow called Fisk
Whose fencing was strikingly brisk
So fast was his action
That FitzGerald contraction
Reduced his rapier to a disk
—I thought this was an original Lear, but can't find it. It was printed in our physics books at school in the 1970s.Martin of Sheffield (talk) 20:48, 20 July 2024 (UTC)
[reply]
Our articles say that Lear died in 1888 and Length contraction wasn't postulated by George FitzGerald until 1889. AlmostReadytoFly (talk) 11:11, 22 July 2024 (UTC)[reply]
My answers to all of your questions are positive.
Let's focus on the following concrete example: If a moving observer measures a given distance between two trees to be one mile long, then we measure this distance to be shorter, i.e.   mile long, right? So, why isn't this fact mentioned in the article? It only mentions an "object's length", which really includes the observer's length, yet not any abstract length, e.g. a distance between two trees and likewise. The phenomenon of length contraction is not only about an object's length, right? HOTmag (talk) 22:04, 20 July 2024 (UTC)[reply]
If the observer is moving along a row of trees, the trees are moving relative to the observer, so this moving observer will observe length contraction by a factor of   for the distance between successive trees, compared to this distance as measured by an observer in whose frame of reference the trees are not moving. So, if "we" are the latter observer, we measure this distance to be longer, by a factor of   than the length reported by the moving observer. If, on the other hand, the trees are moving with the observer, who (from their point of view) is standing still among a row of trees while seeing us whizz along in a rocket, this observer, who is moving only relative to us and not to the trees, will not observe length contraction for the distance between successive trees, whereas to us in the rocket the trees are moving, so we measure the inter-tree distance to be shorter by a factor of   Do you feel our article should explicitly mention the fact that length contraction is not observed by an observer in whose frame of reference the object is not moving?  --Lambiam 06:32, 21 July 2024 (UTC)[reply]
I'm really referring to the latter case you've mentioned, in which the trees are co-moving with the observer, who sees them at rest. It's only us who see, both the observer and the trees, co-moving at the same velocity.
As for your last question: No, I don't want our article to mention the obvious fact that no length contraction is observed by any observer in whose frame of reference the object being measured is at rest. However, I do want the lead of our article, as well as the professional literature discussing the phenomenon of length contraction, to mention that this phenomenon is not only about a given object's length but also about a given abstract length.
Let me explain my point: The lead of our article claims (in my own words) that: if we (travel on a train and) observe a moving "object" (e.g. a moving tree), for which   is the object's length measured in the object's reference frame, then we will meausre the object's length to be shorter, i.e.   right? I guess you agree (if not to my exact own words then to their content). So, why does the lead of our article (as well as the professional literature discussing this phenomenon) only mention an "object", even though this is also true for abstract lengths? Just substitute "distance" (e.g. between two trees) for "object", and you'll get the following true analogous sentence: If we (travel on a train and) observe a moving distance (e.g. a distance between two co-moving trees), for which   is the distance's length measured in the distance's reference frame (i.e. in the reference frame of the trees), then we will meausre the distance's length to be shorter, i.e.   right? This was my question from the very beginning. HOTmag (talk) 07:57, 21 July 2024 (UTC)[reply]
Certainly two trees set a distance apart is an object that has length. They don't have to be touching anymore than the solar system's planets do. Modocc (talk) 11:22, 21 July 2024 (UTC)[reply]
I think the term "system's length" is more appropriate, than the term "object's length", as far as a distance (between two trees or two planets and the like) is concerned. Have you ever called the solar system "an object"? HOTmag (talk) 11:58, 21 July 2024 (UTC)[reply]
Our solar system is a physical object. As are galaxies and our universe. Of course, these kinds of objects have spatial dimensions and a time dimension. Modocc (talk) 12:11, 21 July 2024 (UTC)[reply]
Which name is more appropriate: a "solar system" or a "solar object"?
On the other hand, which name would you prefer: a "system" consisting of two trees, or an "object" consisting of two trees? HOTmag (talk) 13:14, 21 July 2024 (UTC)[reply]
Depends on the physical context. The term "system" regarding physical objects tends to be more abstract and typically refers to processes like with manufacturing and computers. An atmospheric cloud is both a system of particles and an object. Modocc (talk) 13:24, 21 July 2024 (UTC)[reply]
Back to the trees observed by an observer on a moving train, who sees them co-moving and measures the distance between them: They are pretty wide, so the length being measured is not required to include their width, but only the distance between them, i.e. excluding them. This distance is influenced by the effect of length contraction. Would you call this distance an "boject", a "system", or simply a "distance"? The same question may be asked about the distance between two stars, excluding them. HOTmag (talk) 14:10, 21 July 2024 (UTC)[reply]
The length between buttons is the same as the distance between them. With relativity the measured contraction is not an "effect", it is a change in coordinates. The stationary Earth is an oblique sphere, but with reference frame Lorentz boosts it is pancake-shaped instead. Of course, isometries of Minkowski spacetime are defined by the Poincaré group and its subgroup the Lorentz group. Modocc (talk) 14:39, 21 July 2024 (UTC)[reply]
With relativity the measured contraction is not an "effect". I've never disagreed. I called it "effect", just because it's called "effect" - in the lead of our article Length contraction - and in common speech.
The length between buttons is the same as the distance between them. This is exactly what I'm claiming from the very beginning. That's why I'm asking, why the lead of that article - only attributes that contraction to objects - and not also to distances as in your example of a distance between two buttons. HOTmag (talk) 17:02, 21 July 2024 (UTC)[reply]
Right. Matter is composed of various objects and referring to distances between their elements is simply redundant when length suffices, especially when talking about object velocities. Modocc (talk) 17:46, 21 July 2024 (UTC)[reply]
Referring to distances between their elements is simply redundant when length suffices. Does mentioning "length" alone, suffice? Our article is not satisfied with mentioning "length" alone, and mentions an "object's length", so I asked: Why "object's length" only, and not also a "distance's length", as in your example of a distance between two buttons. This distance has nothing to do with matter, because it excludes the buttons. HOTmag (talk) 18:35, 21 July 2024 (UTC)[reply]
The distance between buttons is spacial is it not. They are embedded in actual space. Thus the distances between the ends of rulers are the same as their lengths. Modocc (talk) 18:47, 21 July 2024 (UTC)[reply]
is it not. Yes it is.
the distance between the ends of a ruler is the same as the ruler's length. Your argument is the following: With regard to length contraction, OBJECT includes a ruler, whereas RULER includes any abstract length, Hence OBJECT is supposed to include any abstract length...
Just as, with regard to time dilation, OBJECT includes a clock, whereas CLOCK includes any abstract time, hence OBJECT is supposed to include any abstract time......
According to your explanation, the leads of our articles Length contraction and Time dilation, should've used, either the terms "object's length" and "object's time", respectively, or the terms "ruler's length" and "clock's time", respectively, or "length" alone and "time" alone, respectively.
However, this is not the case. The lead of our article Length contraction attributes "length" to an "object", rather than to a "ruler", whereas the lead of our article Time dilation attributes "time" to a "clock", rather than to an "object". How do you justify this asymmetry, between length contraction and time dilation? HOTmag (talk) 19:38, 21 July 2024 (UTC)[reply]
Rulers and clocks are objects that measure space and time. With relativity their asymmetric reference frame dependencies are a consequence of a coordinate change given the supposed existence of SR and GR's spacetime which conserves the distances of its worldlines. It is by no means a classical model. Modocc (talk) 20:02, 21 July 2024 (UTC)[reply]
When I recently asked you about how "you justify this asymmetry between length contraction and time dilation", I didn't refer to what you're calling now "their asymmetric reference frame dependencies", but rather to the asymmetry between - how the lead of our article length contraction attributes "length" to an "object" rather than to a "ruler" - and how the lead of our article time dilation attributes "time" to a "clock" rather than to an "object".
Please notice, that according to your explanation in your previous response, the leads of those articles should've used, either the terms - "object's length" and "object's time" - respectively, or the terms - "ruler's length" and "clock's time" - respectively, or "length" alone and "time" alone respectively. However, this is not the case in the leads of those articles. My recent question was: Why, and it only referred to the leads of those articles (as well as to the common professional literature). HOTmag (talk) 20:36, 21 July 2024 (UTC)[reply]
Our articles state: "Length contraction is the phenomenon that a moving object's length is measured to be shorter than its proper length, which is the length as measured in the object's own rest frame." and "Time dilation is the difference in elapsed time as measured by two clocks, either because of a relative velocity between them (special relativity), or a difference in gravitational potential between their locations (general relativity). When unspecified, "time dilation" usually refers to the effect due to velocity." Both descriptions refer to measurements of objects in relative motion. Modocc (talk) 21:05, 21 July 2024 (UTC)[reply]
As you quote, the lead of our article length contraction states: "Length contraction is the phenomenon that a moving object's length is measured to be shorter than its proper length", right? So the lead of this article attributes "length" to one "object", instead of attributing length measurement to two "rulers" - a stationary one and a moving one, right?
On the other hand, as you quote, the lead of our article time dilation states: "Time dilation is the difference in elapsed time as measured by two clocks", right? So the lead of this article attributes time measurement to two "clocks" - a stationary one and a moving one, instead of attributing time to one "object", right? My question was: How do you justify this asymmetry between the leads of those articles. HOTmag (talk) 21:46, 21 July 2024 (UTC)[reply]
The first article does not attribute "length" to an "object". Objects have lengths as a property and there are different ways to measure them. In addition, not all objects tick in any obvious way and measure time, so of course clocks are mentioned. Twins do attain different ages per the Twin paradox. :-) Modocc (talk) 22:01, 21 July 2024 (UTC)[reply]
The first article does not attribute "length" to an "object". Objects have lengths as a property. Maybe you didn't interpret me well. By "to attribute X to Y" I mean "to mention X as the property of Y". The expression "a person's feelings" attributes feelings to a person, just as the expression "an object's length" attributes "length" to an "object". Both lengths and feelings are properties attributed to an object/person - respectively, i.e. a length/feeling is a property - an object/person has - respectively.
not all objects - [mark and] tick [moments] in any obvious way - and measure time, so of course clocks are mentioned [in the lead of our article time dilation]. The same is true for rulers: Not all objects - mark and tick centimeters in any obvious way - and measure length, so rulers should apparently have been mentioned in the lead of our article length contraction. So why weren't they mentioned in that lead, even though they should've been mentioned in that lead, just as clocks are mentioned in the lead of our article time dilation? HOTmag (talk) 23:30, 21 July 2024 (UTC)[reply]
IMHO the articles' ledes are fine, as is, and they appear to be written in accordance with the literature and my understanding of it. Without going into what would amount to original research I don't believe I have anything more to contribute here today. Modocc (talk) 23:41, 21 July 2024 (UTC)[reply]

July 21

edit

Microplastics detachment

edit

Why, even though plastics degrade slowly, over hundreds to thousands of years, microplastic particles detach from its parent body (often bottles), contaminating the surrounding area, but glass bottles, for example, seemingly don't have such property? 212.180.235.46 (talk) 10:58, 21 July 2024 (UTC)[reply]

Because plastics degrade and glass doesn't. See Polymer degradation. Alansplodge (talk) 11:08, 21 July 2024 (UTC)[reply]
Glass in the sea does degrade too. As it washes up and down on a beach, the surface becomes rough. Graeme Bartlett (talk) 12:20, 22 July 2024 (UTC)[reply]
Also, most glass bottles are made from materials that, when worn down into particles, are virtually identical to natural sands, so do not contaminate the marine environment. See also Sea glass (which I have always known as drift glass). {The poster formerly known as 87.81.230.195} 94.2.67.235 (talk) 00:27, 23 July 2024 (UTC)[reply]

Colors. (The Color Painter)

edit

I'm not sure where to start. This work defines 12 basic colors. Some are named, and I tried to match them up to sRGB equivalents...

However, I'm no expert on colors.

The original is written in the 1880's, and the author was connected with commercial printers in the US.
It seems reasonable to assume the basic colors were originally intended to match up with available basic pigments available at the time of publication.

So what are the likely 12 original colors (ideally based on pigments that would be generally available to a printer in the US) as modern (and sRGB equivlants?)

The page on Wikisource: s:Page:The color printer (1892).djvu/23 , someone on commons tried to tweak the colors in s:File:The color printer (1892) - Basic tones.svg ShakespeareFan00 (talk) 16:15, 21 July 2024 (UTC)[reply]

The author writes, "These colors were adopted because the writer believes that a greater variety of mixed colors can be produced from this selection than from any other containing the same number; besides, these colors are not only the most useful, but also, the most common, and best known among printers." So they were obviously widely available as printing inks, although I doubt they were thought of as "basic" pigments. We have a List of inorganic pigments; I expect most of these 12 are among them. Even if identified, it may not be obvious how to place them in sRGB colour space.  --Lambiam 19:23, 21 July 2024 (UTC)[reply]
I wouldn't sadly know where to start looking for information as to common printing inks in the 1880's, but from initial reading around, I'm finding that the articles on pigments don't necessarily have sample colors on them. ShakespeareFan00 (talk) 17:10, 22 July 2024 (UTC)[reply]
You probably know this already, but 'The Printer's Manual: An Illustrated History: Classical and Unusual Texts on Printing from the Seventeenth, Eighteenth, and Nineteenth Centuries' by David Pankow says that Earhart's 12 colors came from "twelve stock inks". Maybe they had the same names that he used...not that that helps you get closer to your objective in any way whatsoever. Sean.hoyland (talk) 17:23, 22 July 2024 (UTC)[reply]
Wasn't there a museum that kept a collection of old pigments / inks? ShakespeareFan00 (talk) 19:46, 24 July 2024 (UTC)[reply]
The Forbes Pigment Collection at the Harvard Art Museums.[10] Their online CAMEO database, of which the Forbes Pigment Database is a section, runs on MediaWiki. DMacks (talk) 23:53, 25 July 2024 (UTC)[reply]
Thanks. So it's a case of working out what the stock inks were. In the scan some of the color samples like Vermillion and Rose Lake seem to be different from what I was finding with those names. And whilst the work says it used zinc white, using an approximation I found for that as an HTML triplet, gave colors that were too desturated compared to whats in the scan. Some degree of pigment fading or color change is not unepexected in a more than 100 year old work, but it would be nice to try and figure out collectively what the originals might have been :) ShakespeareFan00 (talk) 19:45, 24 July 2024 (UTC)[reply]
Vermilion is actually not one specific colour but a colour family. The pigments are very likely still available today – if not as commercial printing inks, then as artists' paints. There is no guarantee, though, that two colours with the same name from different producers are identical. Also, there is no guarantee that the colours you see on your screen are the same as those on the physical pages of the book. The average RGB triple for the scan of the colour labeled BLUE is (63, 125, 161), or  #3f7da1 , which is not very blue, more like  steel blue , so the scanned B values are perhaps systematically too low.  --Lambiam 00:12, 25 July 2024 (UTC)[reply]

July 24

edit

The challenge of space surgery

edit

What are the main challenges of performing surgery in outer space? Have there been any actual attempts to do so either on animals or on humans (other than in fairly controversial Russian disaster movies), and if so, were they successful? 2601:646:8082:BA0:55D1:7827:9FF8:5400 (talk) 08:32, 24 July 2024 (UTC)[reply]

The challenges in a zero-gravity setting are manifold. Surgery commonly induces some bleeding. In Earth-bound surgery, gravity keeps the blood from floating away. Free-floating blood droplets will present an unacceptable risk, so foolproof procedures need to be developed to prevent the blood or other bodily fluids from escaping. Free-floating surgical instruments are also not acceptable, but tethering them is awkward. AFAIK no surgery has been attempted under zero-gravity conditions, whether experimental or out of necessity.  --Lambiam 10:40, 24 July 2024 (UTC)[reply]
What, not even cutting off a rat's tail, as claimed in the above-mentioned Russian movie? 2601:646:8082:BA0:55D1:7827:9FF8:5400 (talk) 11:11, 24 July 2024 (UTC)[reply]

(Zero) Gravity

edit

Before the Russkies went and actually filmed a movie in space, how did moviemakers film zero-gravity scenes for older movies set in space (such as You Only Live Twice, Apollo 13, Moonraker, Deep Impact, Armageddon, Space Odyssey 2001 (what, no article?!), Mission to Mars and Gravity)? Did they film the scenes in the pool, or did they use the Vomit Comet, or what? 2601:646:8082:BA0:55D1:7827:9FF8:5400 (talk) 09:03, 24 July 2024 (UTC)[reply]

The article is at 2001: A Space Odyssey.  --Lambiam 10:15, 24 July 2024 (UTC)[reply]
Space Odyssey 2001 was directed by Sham Lee Qbrick. Clarityfiend (talk) 11:02, 25 July 2024 (UTC)[reply]
For some techniques, see 2001: A Space Odyssey § Zero-gravity effects and Technologies in 2001: A Space Odyssey. Other films have used CGI; see e.g. Gravity (2013 film) § Visual effects.  --Lambiam 10:25, 24 July 2024 (UTC)[reply]
The weightless scenes in Apollo 13 were filmed in short sections on a Reduced-gravity aircraft (see Apollo 13 (film)#Filming). Deor (talk) 13:47, 24 July 2024 (UTC)[reply]
Yes, the Vomit Comet as noted above. ←Baseball Bugs What's up, Doc? carrots18:48, 24 July 2024 (UTC)[reply]

July 25

edit

Vicarization

edit

Greetings! I was reading the https://en.wikipedia.org/wiki/Fauna_of_Puerto_Rico article and came across the above word, it is blue-linked to "Speciation" but I cannot find that word on the speciation article. I couldn't find a definition in several dictionaries i checked either...Do you know what this word means? could it be made more clear in the article?140.147.160.225 (talk) 18:47, 25 July 2024 (UTC)[reply]

The link should be to Allopatric speciation. To my ignorant mind it appears that the correct word is "vicariance", not "vicarization". --Wrongfilter (talk) 19:06, 25 July 2024 (UTC)[reply]
Yes. VICARIANCE n. (biology) The separation of a group of organisms by a geographic barrier, resulting in differentiation of the original group into new varieties or species [11] Philvoids (talk) 12:07, 26 July 2024 (UTC)[reply]
thanks so much! I've updated the article 140.147.160.225 (talk) 18:42, 29 July 2024 (UTC)[reply]

Fels-Naptha

edit

At Talk:Fels-Naptha I raised the issue that the article makes that unsourced claim that "Fels-Naptha once contained naphtha, a skin and eye irritant", but I was unable to find any source showing that Fels-Naptha (note the single "h") soap used to have Naphtha (note the two "h"s) in it, when it was removed, or why it was removed. This may be an urban myth. Can anyone find a source for those claims? --Guy Macon Alternate Account (talk) 19:52, 25 July 2024 (UTC)[reply]

Following up links in the references of that article and of Naphtha leads to several sources that state naphtha was dropped from the ingredients, at least one saying this was due to fears it might be carcinogenic. I don't know if any of those sources count as 'Reliable', and none give their sources. This seems unsurprising given the degree to which naphtha was and is used in many products and processes.
'Naptha' seems merely to be a common variant spelling, particularly in product names, presumably because the string -phth- is uncommon in English and likely to be mispronounced and misread by non-chemists. {The poster formerly known as 87.81.230.195} 94.2.67.235 (talk) 20:49, 28 July 2024 (UTC)[reply]

July 26

edit

Absorption of matter, without changing the absorber's restmass. Possible?

edit

When a stationary system absorbs a new stationary body, the system's restmass increases by the new body's restmass.

Something similar happens when an electron absorbs light. See Wikisource: The free electrons absorb some of the ultraviolet energy that initially set them free and form an ionized layer.

However, when a stationary body collides with a moving body, the stationary body gains kinetic energy, without changing this body's restmass.

Can a moving body be absorbed by an absorber (like in the first case), but with the absorber's restmass remaining the same as before (like in the second case)? HOTmag (talk) 12:35, 26 July 2024 (UTC)[reply]

A free electron cannot absorb light. Ruslik_Zero 13:24, 26 July 2024 (UTC)[reply]
Photons caught by a trap increases its mass. As Ruslik_Zero points out, photons interacting with free electrons are not absorbed. Instead they interact elastically, but inelastic collisions are always additive increasing the invariant mass of the absorber. Modocc (talk) 14:26, 26 July 2024 (UTC)[reply]

Must the answer to the question in the title be negative?

As for free electrons absorbing light, I've struck it out, but is the book "Electronics Technician" wrong? It's quoted in Wikisource: [12]: The free electrons absorb some of the ultraviolet energy that initially set them free and form an ionized layer.

HOTmag (talk) 14:46, 26 July 2024 (UTC)[reply]

Not wrong. The electrons were bonded so the photons' energies broke them without changing their intrinsic rest masses. Modocc (talk) 15:38, 26 July 2024 (UTC)[reply]
Ok, so I've just added back the first comment about free electrons absorbing light (I've also added your clarification). Anyway, I'm still curious to know the answer to the question in the title. HOTmag (talk) 16:27, 26 July 2024 (UTC)[reply]
In Compton scattering, a free electron gains energy and momentum from a photon, but it does not "absorb" it. --Wrongfilter (talk) 16:52, 26 July 2024 (UTC)[reply]
Right, matter absorbs radiation per Quantum electrodynamics. Its rest mass increases unless the energy gets scattered elsewhere. Modocc (talk) 18:06, 26 July 2024 (UTC)[reply]
The electron's mass does not increase. --Wrongfilter (talk) 18:26, 26 July 2024 (UTC)[reply]
With Compton scattering, it cannot absorb the photon. Modocc (talk) 18:29, 26 July 2024 (UTC)[reply]
Who said it does? I'm out. --Wrongfilter (talk) 18:31, 26 July 2024 (UTC)[reply]
Sorry. I certainly did not nor did I say the electron's mass increased! And when I realized I simply repeated what you said I was going to fix that. Modocc (talk) 18:35, 26 July 2024 (UTC)[reply]
Right, the mass of the bound masses increases and with respect to electrons only their energy increases. Modocc (talk) 18:38, 26 July 2024 (UTC)[reply]
HOTmag, as I was trying to say, bound matter's rest mass increases unless the energy absorbed by it gets emitted again. Modocc (talk) 18:49, 26 July 2024 (UTC)[reply]
In general, the energy of waves are absorbed: See Absorption (acoustics) and Absorption (electromagnetic radiation). Also, all matter is thought to be comprised of matter-waves. Perhaps that helps. Modocc (talk) 19:50, 26 July 2024 (UTC)[reply]
To sum up, electrons' masses are intrinsic and elemental, but the bound rest masses of objects are not. Both absorbed light and matter can increase the latter's (bound rest masses) mass as you observed, but neither can increase the former's mass (the electron's). Thus the answer is no, absorbers do not absorb matter without increasing their rest masses unless they release it, like you noted with particle annihilation, if only because their masses are not as elemental as their constituents... Modocc (talk) 00:24, 27 July 2024 (UTC)[reply]
Let me be more clear, now without mentioning photons:
When a stationary system absorbs a new stationary body, the system's restmass increases by the new body's restmass.
However, when a stationary body collides with a moving body, the stationary body gains kinetic energy, without changing this body's restmass.
Can matter be absorbed by an absorber (like in the first case), but with the absorber's restmass remaining the same as before (like in the second case)? HOTmag (talk) 19:30, 27 July 2024 (UTC)[reply]
The first case, absorption, always adds rest mass (the second case changes the object's KE, but not its rest mass like within particle accelerators). Modocc (talk) 20:11, 27 July 2024 (UTC)[reply]
I didn't ask about the first case, i.e. about a stationary system absorbing a stationary body so that the whole system's restsmass increases, nor about the second case in which the restmass doesn't change.
I wonder, why there can be no third case, i.e, a case in which a system (whether a stationary one or a moving one) absorbs a moving body, so that the system's kinetic energy increases but the whole system's restsmass doesn't change. Is there any reasoning or explanation behind this fact of absence of such a third case? HOTmag (talk) 21:56, 27 July 2024 (UTC)[reply]
When an object is at rest its KE is zero, but conservation of energy requires that every object's total energy to be the sum of its parts. We call it rest mass and absorption(s) increases it. Modocc (talk) 22:36, 27 July 2024 (UTC)[reply]
I think the conservation of energy is not sufficient for the full explanation: Without the conservation of momentum, one can still argue, that before the absorptoin, the absorber was at rest - hence carried no kinetic energy, while the other body about to be absorbed carried some kinetic energy. After the absorption, the whole system remained surprisingly with the same mass as before, but gained the absorbed body's kinetic energy. What's wrong with that? The wrong thing is my neglecting the conservation of momentum. HOTmag (talk) 00:58, 28 July 2024 (UTC)[reply]
The absorbed body adds, at a minimum, a mass-energy equal to KE/c2 to the absorber which gains its KE. In addition, for the n-body system, its rest mass and total energy is conserved and unchanged whether they are far apart or bonded together, or internalized and perhaps superimposed. To calculate their combined rest mass one simply adds up their energies in its center-of-momentum frame. In this reference frame the momentum vanishes and their total energy is therefore its rest mass. Modocc (talk) 03:25, 28 July 2024 (UTC)[reply]
As to your first sentence about adding a mass   to the absorber: Please notice, that without the conservation of momentum, one can still argue that before the absorption, the body about to be absorbed carried a total energy   that included - both an internal energy   - and a kinetic energy   equivalent to a mass of the size   you've mentioned. After the absorption, the whole system remained surprisingly with the same mass as before, whereas the absorbed body's internal energy   was not added to the absorber's internal energy as an addition of the size  , but rather the absorbed body's total energy   was added to the absorber's kinetic energy as an addition of the size  . What's wrong with that, without assuming the consevation of momentum, which may actually be not conserved (as you can see in my following thread)?
As to your last claim that "for the n-body system, its rest mass and total energy is conserved and unchanged whether they are far apart or bonded together": AFAIK, what's conserved is the mass-energy as a whole, but the mass alone doesn't have to be conserved: Check: an electron-positron pair, becoming energy alone, without conserving the mass alone (unless one attributes mass to photons, which is a controversial and debatable possibility). HOTmag (talk) 07:11, 28 July 2024 (UTC)[reply]
Energy contributes to the rest mass. For example, the gluons' energy within the proton contributes to its overall rest mass. It's a widely accepted concept. Modocc (talk) 12:35, 28 July 2024 (UTC)[reply]
Not always. Check: an electron-positron pair, becoming energy alone, without contributing any mass to the light emitted (unless one attributes mass to photons, which is a controversial and debatable possibility). HOTmag (talk) 12:38, 28 July 2024 (UTC)[reply]
The rest mass of the 2-body system is conserved. Modocc (talk) 12:54, 28 July 2024 (UTC)[reply]
Before the electron and the positron annihilated each other and became energy, the system's rest mass was positive, but after they annihilated each other, the system became light carrying no restmass. HOTmag (talk) 13:24, 28 July 2024 (UTC)[reply]
The light carries only energy and momentum yes, but there is a center-of-momentum reference frame in which the particle-waves' momentum vanishes, but their energy, their 2-body rest mass, does not. Modocc (talk) 13:44, 28 July 2024 (UTC)[reply]
Yes, their energy does not vanish, but their 2-body restmass does vanish, once they annihilate each other and become light, which actually carries no restmass, so the energy they carried before they annihilated each other does not contribute any mass to the light emitted. HOTmag (talk) 17:19, 28 July 2024 (UTC)[reply]
Place the event in an opaque container. Both the container's total energy and momentum are unaffected because its rest mass includes the photons. Modocc (talk) 19:55, 28 July 2024 (UTC)[reply]
Yes, when a photon is absorbed it contributes to the absorber's restmass.
However, when an electron-positron pair becomes light in the free space, the pair's restmass vanishes.
This proves that restmass alone, in the free space (rather than in an opaque container), doesn't have to be conserved.
Indeed, restmass is conserved if one assumes both the conservation of energy and the conservation of momentum, but if one only assumes the conservation of energy without assuming the conservation of momentum, then one can still argue, that although any massive body about to be absorbed carries a total energy   that includes - both a kinetic energy   - and an internal energy   equivalent to restmass, still after the absorption, the absorber remains surprisingly with the same restmass as before, whereas the absorbed body's internal energy   is not added to the absorber's internal energy as an addition of the size  , but rather the absorbed body's total energy   is added to the absorber's kinetic energy as an addition of the size  . What's wrong with that, without assuming the consevation of momentum, which may actually be not conserved (as you can see in my following thread)? HOTmag (talk) 22:35, 28 July 2024 (UTC)[reply]
The container need not contain free space and even if it does as a system its total energy still does not change. Think also of the atmospheric cloud. It consists of enormous numbers of massive and massless particles moving at various velocities, but it is nearly stationary to you and the clouds' total energy that can be calculated is called rest mass. Modocc (talk) 23:00, 28 July 2024 (UTC)[reply]
Yes, the container, as well as the atmospheric cloud, are systems each of which conserves the restmass. However, in my previous response I didn't talk about any container, nor about any atmospheric cloud. I only said, that "when an electron-positron pair becomes light, [not in a container nor in an atmospheric cloud, but rather] in the free space, then the pair's restmass vanishes". This proves that restmass alone, in the free space (rather than in a container or in an atmospheric cloud), doesn't have to be conserved. Then I added the crucial last paragraph in my previous response. HOTmag (talk) 06:45, 29 July 2024 (UTC)[reply]
We've had the two-photon thing before. --Wrongfilter (talk) 09:32, 29 July 2024 (UTC)[reply]
Yes, I remember, but this time I'm talking with Moddoc about an electron-positron pair, which is another issue. HOTmag (talk) 10:54, 29 July 2024 (UTC)[reply]
How many photons do you think come out of an electron-positron annihilation event? --Wrongfilter (talk) 11:13, 29 July 2024 (UTC)[reply]
2. HOTmag (talk) 12:54, 29 July 2024 (UTC)[reply]

July 27

edit

Grooming behavior

edit

In videos of grooming monkeys put all the trash they clean into own mouth, as if eating it, rather than throwing out (same happens when a pet monkey is grooming a human). Why do they do that and isn't harmful for their health? 212.180.235.46 (talk) 10:00, 27 July 2024 (UTC)[reply]

How would they know any different? ←Baseball Bugs What's up, Doc? carrots11:41, 27 July 2024 (UTC)[reply]
If they find lice, these are delicious and nutritious. Also yummy and more important are flakes of salt (dried-up sweat). Dandruff is harmless. If it was generally harmful to their health, evolution would have weeded out this specific behaviour long time ago.  --Lambiam 12:32, 27 July 2024 (UTC)[reply]
I guess when you are a social animal, putting parasites you have found back on the ground probably isn't a good idea. This is, however, what my dog does with ticks. But the ticks are apparently not delicious, even for fire ants. Sean.hoyland (talk) 05:23, 28 July 2024 (UTC)[reply]
Also, small things sometimes (often?) have some antibacterial and antifungal things in their biological toolkits to help them stay alive. Maybe eating them can be beneficial. Sean.hoyland (talk) 05:36, 28 July 2024 (UTC)[reply]

What's the opposite of "sticky"?

edit

When a given object tends to get attached to close objects, it may be called "sticky". What about the opposite phenomenon? i.e. How should a given object be called, when it's "reluctant" to get attached to close objects? Even if all objects are at rest, so the adjective "elastic" is not sufficient for describing the opposite of "sticky". HOTmag (talk) 20:20, 27 July 2024 (UTC)[reply]

Repulsive, eg. the wall repels objects and the suffix -phobic is used in chemistry, eg. hydrophobic. Modocc (talk) 20:33, 27 July 2024 (UTC)[reply]
Thank you. HOTmag (talk) 22:01, 27 July 2024 (UTC)[reply]
There is non-stick, which describes objects made of (or coated with) a material with a very low coefficient of friction. Mikenorton (talk) 20:39, 27 July 2024 (UTC)[reply]
Thank you. HOTmag (talk) 22:02, 27 July 2024 (UTC)[reply]
Hoban (1975) used the term "anti-sticky". [13]  Card Zero  (talk) 03:42, 29 July 2024 (UTC)[reply]
"anti-sticky" is good. I guess you could write "not sticky" as "  sticky", a downside being that it looks like a stick being described as stick-like. Sean.hoyland (talk) 07:06, 29 July 2024 (UTC)[reply]
Thank you. HOTmag (talk) 13:11, 29 July 2024 (UTC)[reply]
Teflon? Teflon-like? Huldra (talk) 21:59, 29 July 2024 (UTC)[reply]
Also nonadhesive. Modocc (talk) 11:12, 30 July 2024 (UTC)[reply]
Slippery Doug butler (talk) 11:24, 30 July 2024 (UTC)[reply]

July 28

edit

What is the geological composition of Lascaux?

edit

I am an artist currently working on a project themed around prehistory and I'd like to know about the geological composition of Lascaux (as well as other early human settlements and pre-human geography in general), so I can better represent it. Additionally, is there any sort of database I can use for these purposes? Aedenuniverse (talk) 01:20, 28 July 2024 (UTC)[reply]

See karst. Not sure that helps much. Sean.hoyland (talk) 06:05, 28 July 2024 (UTC)[reply]
Specifically, it is reported as being calcarenite from the upper Coniacian. Sean.hoyland (talk) 06:10, 28 July 2024 (UTC)[reply]
The function of Lascaux is a topic of academic debate, but it was not a settlement. It, and similar caves, are not suitable for serving as dwellings. Human groups of that time lived in tents or the open air, and may occasionally have found shelter in much shallower caves.  --Lambiam 12:02, 28 July 2024 (UTC)[reply]

Conserving the kinetic energy, without conserving the momentum. Possible?

edit

There being no external forces, inelastic collisions conserve the whole system's momentum, without conserving the whole system's kinetic energy.

What about the opposite physical process? I.E. is there any physical process, e.g. with external forces, which conserves the whole system's kinetic energy, without conserving the whole system's momentum? HOTmag (talk) 07:27, 28 July 2024 (UTC)[reply]

  Resolved
I've just thought about it:
The whole system is: a single elastic body.
The physical process is as follows: the single elastic body collides with an elastic wall, being the external force.
Result: the single elastic body is pushed back to the oppposite direction, with the same speed as before the elastic collision took place.
Therefore: the elastic body's kinetic energy is conserved, yet the elastic body's momentum is not. QED. HOTmag (talk) 10:05, 28 July 2024 (UTC)[reply]
Nah, you've ignored newton 3. QED Greglocock (talk) 23:47, 28 July 2024 (UTC)[reply]
Why do you think I ignored it? I think I didn't. HOTmag (talk) 06:47, 29 July 2024 (UTC)[reply]
My second thought is to underline 3 words in my response below. Ignoring what is happening by calling one involved object "the whole system" is disingenuous. Philvoids (talk) 18:21, 29 July 2024 (UTC)[reply]
The decision of what the "whole system" is, depends on our choice. Here is a typical example: If two perfectly elastic bodies collide with each other, we can choose, whether to call the two-body system: "the whole system", in which case no external force is involved - hence the momentum is conserved in our "whole system" chosen, or to call one colliding body alone "the whole system", in which case the other body exerts an external force on our "whole system" chosen - hence the momentum is not conserved in our "whole system" chosen. This is how all of physics works, with no exceptions. HOTmag (talk) 20:39, 29 July 2024 (UTC)[reply]
The wall will gain some momentum. In a multi body collision the centre of gravity of the system remains at constant velocity (N1) in the absence of external forces. You are 100% wrong, and are just making up random explanations to cover up your lack of understanding. I'll be ignoring you from now on. Greglocock (talk) 23:54, 29 July 2024 (UTC)[reply]
I'm referring to a very specific reference frame, which is the wall's reference frame, in which every observer referring to the elastic body as "the whole system" attributes no change to the wall's momentum.
I'm bad at analyzing personal comments, so I'm letting the users decide who is right and who is wrong.
HOTmag (talk) 09:18, 30 July 2024 (UTC)[reply]
If you're analysing a collision, you can't just pick one of the things that's colliding and call it the whole system. If it was the whole system, there would be nothing else for it to collide with. "The whole system" is not a reference frame. It's the whole set of objects, by definition; you can't pick and choose which objects it includes. AlmostReadytoFly (talk) 09:57, 30 July 2024 (UTC)[reply]
A side note: I've never claimed that "the whole system" is a reference frame.
As to your main response: Can you give any example of a two-body system, for which our referring to one of them as the "whole system" may contradict the laws of physics? AFAIK, there is no example of this kind. Recommendation: before you try to think about such an example, take another look at the example I've already given in my previous-previous response (i.e. the one beginning with "The decision"). HOTmag (talk) 10:35, 30 July 2024 (UTC)[reply]
If you have a two-body system, the two bodies are the system. If you say you have a one body system, then say that that body collides with something else (e.g. a wall), you're contradicting yourself. AlmostReadytoFly (talk) 11:30, 30 July 2024 (UTC)[reply]
Maybe the expression "the whole system" confuses you. I can replace it by the expression "the sub-system chosen", in which case no contradiction follows, even according to your attitude.
One contradicts oneself when one says "x" and then says "not x". My analysis is not the case, even when the expression "whole system" is used, but I'm changing it for you, to avoid confusion. HOTmag (talk) 12:32, 30 July 2024 (UTC)[reply]
"When I use a word," Humpty Dumpty said in rather a scornful tone, "it means just what I choose it to mean——neither more nor less."
If you just want a physical process where a body keeps its kinetic energy but not its momentum, consider a body in a circular orbit. AlmostReadytoFly (talk) 13:00, 30 July 2024 (UTC)[reply]
Humpty Dumpty didn't care if the way he spoke could confuse others, but I do care, and that's why I changed the expression "whole-system" to "sub-system".
Yes, also the body you suggest can be chosen as the sub-system. But also the body I've suggested can. HOTmag (talk) 17:56, 30 July 2024 (UTC)[reply]
Your Question: is there any physical process, e.g. with external forces, which conserves ... kinetic energy, without conserving ... momentum?
Answer: Yes, a circular orbit. AlmostReadytoFly (talk) 18:24, 30 July 2024 (UTC)[reply]
I've already approved your answer, in my previous response, so I wonder why you had to repeat the same answer. I only added that also my answer (that preceded yours) was correct. HOTmag (talk) 18:59, 30 July 2024 (UTC)[reply]
(ec)
All types of collision (inelastic or elastic) conserve momentum. Total kinetic energy would be conserved (meaning no release of sound or heat) only in an impractical perfectly elastic collision. To identify a process that conserves a system's kinetic energy but may be affected by external forces, one needs firstly to clarify whether the system shall qualify as an Isolated system where thermodynamic laws apply. The Second law of thermodynamics observes that the entropy of isolated systems left to spontaneous evolution cannot decrease, as they always tend toward a state of thermodynamic equilibrium where the entropy is highest at the given internal energy. Philvoids (talk) 10:10, 28 July 2024 (UTC)[reply]

July 29

edit

Access to reference 9 in Petrichor

edit

Garg, Anu (2007). The Dord, the Diglot, and an Avocado Or Two: The Hidden Lives and Strange Origins of Words. Penguin. p. 399. ISBN 9780452288614.

Do you have access to the page 399? I would like to find out what it says about the two researchers, Isabel Bear and Dick Thomas. (In ruwiki the same source is cited and it is written that Thomas was from UK, whereas the enwiki says they are both from Australia).

Thank you in advance for your help. Gryllida (talk, e-mail) 00:41, 29 July 2024 (UTC)[reply]

The ref says, "In 1964, two Australian researchers, I.J. Bear and R. G. Thomas..." but does not provide any further biographical informaion about either one. Our article links to enwiki articles about each of them: Isabel Joy Bear and Richard Grenfell Thomas. Bear's article makes a strong claim for her being Australian, even though she did work for a few years in the UK. Thoman's article does not have any hint of any national connection other than Australia. DMacks (talk) 01:02, 29 July 2024 (UTC)[reply]
Thanks, Bear's article says "In the 1950s Bear moved to the United Kingdom, where she worked at the Harwell Science and Innovation Campus. She moved to the University of Birmingham, where she worked as a postdoctoral researcher in the department of metallurgy. Whilst working in Birmingham Bear became interested in solid-state chemistry. Bear joined the Council for Scientific and Industrial Research (CSIRO) in 1953, [...]" -- does it mean she apparently worked in the UK between 1950 and 1953?
The Nature paper was in 1964. Gryllida (talk, e-mail) 04:05, 29 July 2024 (UTC)[reply]
Yes, Reference [3] after the first sentence in your quote says explicitly "
In the UK (1950-53)
During three years in the UK she was employed first as an Experimental Scientist in the Metallurgy Division of the Atomic Energy Research Establishment at Harwell, and later as a Research Assistant in the Metallurgy Department of Birmingham University." AlmostReadytoFly (talk) 15:45, 29 July 2024 (UTC)[reply]

The fastest Internet speed during rain?

edit

So ChatGPT says the fastest Internet speed during rain is fiber topics, then cabled Internet, then mobile wireless Internet. I just want a 2nd opinion if anyone agrees or disagrees? Specifically, ChatGPT said:

  • Most Affected: Mobile wireless internet is the most affected by rainy weather due to signal attenuation.
  • Moderately Affected: Cabled internet can be affected if the infrastructure is old or damaged, but it is generally more resilient than mobile wireless internet.
  • Least Affected: Fiber optic internet is the least affected by rain due to its well-protected, light-based transmission system.

Thanks. 66.99.15.162 (talk) 19:36, 29 July 2024 (UTC).[reply]

Fibre optic cable is always the fastest. The bot is right that wireless technology suffers from rain by signal attenuation, which, provided the protocol is designed to make use of good conditions, can slow down internet speed in rain. Some attenuation isn't too bad, as it reduces interference between nearby cellphone towers, without significantly affecting signal strength on short distances. Once the transmitter reaches maximum power, more rain reduces possible speed.
Wired technology is generally unaffected by rain, unless there's so much rain that it enters the cabinets housing routers etc. or causes landslides, ripping the cables apart. Maybe ChatGPT thinks (to the extend that machines can think) that copper cable networks tend to be older than fibre optic networks and therefore more susceptible to such water intrusion.
Copper networks are typically faster than wireless for the same reason as why speaking tubes are better than providing everybody with a megaphone: the more people shout over the same medium, the more confusion. PiusImpavidus (talk) 09:47, 30 July 2024 (UTC)[reply]
The speed of mobile wireless Internet access depends more on the generation of broadband cellular network technology deployed locally (2G, 3G, 4G, 5G) than on the weather conditions. If there is no cellular coverage, the only solution is satellite Internet access, which can stream at a high rate but has a high latency. In all cases (wireless, cable, fiber) the bandwidth may depend on the contract with the provider – often one can opt for a subscription with a higher rate at a higher cost. And in all cases the actual latency and streaming rate may be much lower than the promised one.  --Lambiam 09:57, 30 July 2024 (UTC)[reply]

Historic (pre-1800s) Wildfires in California

edit

I read that historically, about 2-4 million hectares would burn a year in California, https://www.propublica.org/article/they-know-how-to-prevent-megafires-why-wont-anybody-listen and https://www.sciencedirect.com/science/article/abs/pii/S0378112707004379. A claim made in the sources was that smoke was a feature of the landscape, rather than an oddity as it is now. If the "extreme" modern season tends to be around 2 million ha with moderate air quality impact, what would be some rough estimates for the average pm2.5 levels across pre-1800s California in late summertime? Takedalullaby (talk) 21:01, 29 July 2024 (UTC)[reply]

Sounds like The Burning City by Larry Niven and Jerry Pournelle. They claim that the Los Angeles valley was originally called Iyáangẚ, "the valley of smoke". by the Tongva. Abductive (reasoning) 20:17, 30 July 2024 (UTC)[reply]

July 30

edit

Rare quasi-cancer disease in children

edit

Friends recently received a diagnosis for their toddler, and they told me what the doctor had called it, but now I can't remember its name. I know that it has a Wikipedia article, because I read the article when they first told me.

All I remember of the little guy's symptoms is that he frequently has joint pain at unexpected times. The doctor explained to them that the disease causes lesions in random places (including in the bones, if I remember rightly), and because these bulges occur in places where they shouldn't be, some interfere with ordinary movement and cause pain. The disease is treated with chemotherapy, and apparently there's some debate among the experts over whether it should be classified as a kind of cancer. I think the doctor gave a reasonably good prognosis for the disease with treatment and a dreadful prognosis without treatment. I don't remember if the Wikipedia article mentioned if the cause is known, or if it is, what causes it. It's not so rare that the exact number of diagnoses is known, but it's classified as rare (at least here in Australia) because it occurs only once per several thousand individuals. I've looked through Category:Syndromes with musculoskeletal abnormalities without finding it.

While this desk doesn't provide medical advice, remember that I'm not asking for diagnosis: I'm basically starting with a diagnosis and trying to work out the name. This is similar to the strep-infection question from Wikipedia:Kainaw's criterion. Nyttend (talk) 19:27, 30 July 2024 (UTC)[reply]

Histiocytosis? Ruslik_Zero 20:39, 30 July 2024 (UTC)[reply]
Ah, it's Langerhans cell histiocytosis. Now I remember my confusion when talking with the friends, since initially I thought they meant it was something pancreatic. Thanks for the pointer! Nyttend (talk) 21:35, 30 July 2024 (UTC)[reply]

July 31

edit

Is there any simple necessary sufficient condition that conserves kinetic energy, without mentioning kinetic energy?

edit

Note that absence of external force is a simple necessary sufficient condition that conserves momentum, without mentioning momentum (just as absence of external torque is a simple necessary sufficient condition that conserves angular momentum, without mentioning angular momentum).

However, the absence of external force is not a necessary condition (that conserves kinetic energy), because a given body's kinetic energy can be conserved even when external forces are exerted on that body, e.g. when it's in a circular orbit (in which case the space must have more than one dimension), or when the body elastically collides with a wall sharing a reference frame with an observer who measures the body's kinetic energy (in which case the space is allowed to have a single dimension).

The absence of external force is not a sufficient condition (that conserves kinetic energy) either, as can be shown when two bodies inelastically collide with each other: The two body system's kinetic energy is not conserved, although no external force is exerted on that two body system. HOTmag (talk) 07:11, 31 July 2024 (UTC)[reply]

In a system of two equal bodies circling each other around a common centre of mass, the net momentum of the system is constant, yet there are forces at play.  --Lambiam 09:18, 31 July 2024 (UTC)[reply]
The forces you are talking about, are internal ones, each of which is exerted on only a part of the two body system, on which no external force is exerted. Any way, for the sake of clarity, I've just added the word "external" to my first post. HOTmag (talk) 09:37, 31 July 2024 (UTC)[reply]
.
.
  Resolved
I've just thought about it, assuming that restmass does not change:
The conservation of velocity is a simple necessary sufficient condition that conserves momentum, without mentioning momentum.
The conservation of speed (i.e. of the absolute value of velocity) is a simple necessary sufficient condition that conserves kinetic energy, without mentioning kinetic energy. HOTmag (talk) 10:49, 31 July 2024 (UTC)[reply]
You write ‘The absence of external force is not a sufficient condition ...’ This is debatable. I say that the kinetic energy of the constituents of the system doesn't necessarily contribute to the kinetic energy of the entire system. The energy associated with the movement of the inelastically colliding bodies relative to their common centre of mass is counted as internal energy of the system, not as kinetic energy. Take for example a bottle of warm gas in a circular orbit around Saturn. After a while, the gas cools down. The kinetic energy of the gas molecules decreases, but the kinetic energy of the bottle of gas remains the same; the thermal energy decreases. PiusImpavidus (talk) 18:08, 31 July 2024 (UTC)[reply]
You're right, so thanks to your important comment, I've just struck out the wrong paragraph in my first post (See above). HOTmag (talk) 20:54, 31 July 2024 (UTC)[reply]
Consider again a system of two equal bodies orbiting around a common centre of mass. Choose the coordinate system such that the common centre of mass of the two-body system is at rest. If one body's momentum equals   at some instant of time, that of the other at the same instant of time equals   so the momentum of the system is   Now apply external forces rotation-symmetrically to the objects so as to reverse their motion, making them circle again around their common centre of mass – which has not budged – but in the opposite sense. The symmetry guarantees that the momentum of the system remains   at all times, so the absence of external forces is not a necessary condition for conserving momentum.  --Lambiam 20:06, 31 July 2024 (UTC)[reply]
Your case does not involve an external force but rather involves an external torque. Any way, for the sake of clarity, I've just added this clarification to the first paragraph of my first post. HOTmag (talk) 20:54, 31 July 2024 (UTC)[reply]
...but equal and opposite applied forces like compression conserves momentum too. In other words, absence of external forces is a sufficient condition but it is not necessary. Modocc (talk) 21:35, 31 July 2024 (UTC)[reply]

Is there any simple necessary sufficient condition, that conserves a given system's total energy, without mentioning energy/mass?

edit

Just as absence of external force is a simple necessary sufficient condition that conserves momentum, without mentioning momentum. HOTmag (talk) 10:57, 31 July 2024 (UTC)[reply]

The net work done by the system is equal to the net heat received. PiusImpavidus (talk) 17:45, 31 July 2024 (UTC)[reply]
It's not a necessary condition: Consider a harmonic oscillator, in which the total energy is conserved, even though the (changing) net work done by the system is not equal to the (zero) net heat received. HOTmag (talk) 21:09, 31 July 2024 (UTC)[reply]

August 1

edit

Mathematics

edit

July 13

edit

Given the results from powers of tau in the trusted setup ceremony ; the verifying and the proving key, how can I find the point [f] resulting from the trusted setup in Groth16 ?

edit
Moved to the Computing section of the Reference desk —  --Lambiam 13:36, 16 July 2024 (UTC)[reply]

July 15

edit

Next Julian period

edit
Moved to the Science section of the Reference desk —  --Lambiam 13:24, 16 July 2024 (UTC).[reply]

July 16

edit

In SageMath, how to use GF() on a very large finite field ?

edit
Moved to the Computing section of the Reference desk —  --Lambiam 13:37, 16 July 2024 (UTC)[reply]


July 18

edit

Functions whose every derivative is positive growing slower than exponential

edit

Is there any smooth function with the following two properties:


 , i.e. the nth derivative of f is strictly positive for every x and n.

  for every b > 1. The hard case is when b is small.


Functions like   (for a > 1) are the only ones I can think of with the first property, but none of them has the second property because you can always choose b < a. So I am asking whether there is any function with the first property that grows slower than exponential.

120.21.218.123 (talk) 10:09, 18 July 2024 (UTC)[reply]

Wouldn't any power series with positive coefficients that decrease compared to the coefficients of the exponential do? The exponential is  , so e.g.   should do the trick. The next question is whether you can find a closed-form expression for this or a similar power series. --Wrongfilter (talk) 13:02, 18 July 2024 (UTC)[reply]
Good thinking. It is of course the case that the first property holds for any power series where all coefficients are positive. Plotting on a graph, I think your specific example doesn't satisfy the second property, but others where the coefficients decrease more rapidly do. 120.21.218.123 (talk) 13:26, 18 July 2024 (UTC)[reply]
           --Lambiam 13:45, 18 July 2024 (UTC)[reply]
A half-exponential function will satisfy your requirements. Hellmuth Kneser famously defined an analytic function that is the functional square root of the exponential function.[1]  --Lambiam 14:04, 18 July 2024 (UTC)[reply]

References

A variant of Wrongfilter's idea that I think does work:
 
(taking   to be  ).
Numerical evidence suggests that   One might therefore hope that   would also work. However, its second derivative is negative for    --Lambiam 22:20, 20 July 2024 (UTC)[reply]
And some higher derivatives are negative for even larger values of x. The eighth derivative is negative for  , for instance. 120.21.79.62 (talk) 06:48, 21 July 2024 (UTC)[reply]
The fourteenth derivative is negative for  . That's as high as WolframAlpha will let me go. 120.21.79.62 (talk) 06:54, 21 July 2024 (UTC)[reply]
Yes, shifting the graph along the x-axis by using   won't help.  --Lambiam 11:46, 21 July 2024 (UTC)[reply]

July 21

edit

If the sum of the first m factorial numbers is equal to the sum of the first n positive integers

edit

If the sum of the first m factorial numbers is equal to the sum of the first n positive integers, i.e. 1! + 2! + 3! + … + m! = 1 + 2 + 3 + … + n, then (m,n) = (0,0), (1,1), (2,2), (5,17), right? 220.132.216.52 (talk) 20:30, 21 July 2024 (UTC)[reply]

The triangular numbers, modulo 19, are reduced to one of 10 possibilities: 0, 1, 2, 3, 6, 7, 9, 10, 15 and 17. The sum 1! + 2! + 3! + ... + m!, for m > 17, modulo 19, is reduced to 8. Therefore no further factorial sums are triangular.  --Lambiam 22:19, 21 July 2024 (UTC)[reply]
I looked at mod 7 with about the same result. The left hand side is 5 for m≥6 and the right hand side can never be 5. So you only have to check m from 1 to 5. (Btw, I would count 0! = 1 as a factorial number, so the sums of factorials would be 1, 2, 4, 10, 34, ... . (sequence A003422 in the OEIS)) --RDBury (talk) 22:28, 21 July 2024 (UTC)[reply]
Wrote up a quick MATLAB script to find numbers which can be used as modulos to show that the list is finite, it starts:  . Obviously if a number appears in the list then all its positive multiples do too. The list of nontrivial numbers starts   GalacticShoe (talk) 03:01, 22 July 2024 (UTC)[reply]
Submit to OEIS! —Tamfang (talk) 20:31, 28 July 2024 (UTC)[reply]

July 22

edit

Computing the centre of a triangle...

edit

(On math section as it's a geometric/trig problem essentially)

In CSS, color-mix() takes 2 Params.. However I have colors to mix that take 3 or more params.

Whilst with 2 params you can do a simple linear interpolation, based on the weights of the 2 params, I wasn't sure how it could be done for 3.

One approach I had considered was (at least for an RGB blend) is to compute the centrepoint of a triangle in 3D space, where the 3 points of the triangle are the three colors. However that would assume equal weights of each color, I figured

So for a given "triangle" defined by (r1,g1,b1),(r2,g2,g2), (r3,b3,g3) and a mix ratio of w1:w2:w3  compute the centroid(?) of the triangle representing the blended color. ?

Alternatively is there a different math/geometrical technique that is used in actual computer graphics work?
ShakespeareFan00 (talk) 17:18, 22 July 2024 (UTC)[reply]

I don't have an actual computer graphics answer, but the interpolation method still works for three points, simply take the weighted sum of points assuming   (if not, then just define new values   which do add to  .) In other words, you can just take   (or, more concisely,  .) GalacticShoe (talk) 17:45, 22 July 2024 (UTC)[reply]
Thanks. I thought I was thinking along the right lines..
In case you are wondering why I asked -s:Page:The_color_printer_(1892).djvu/55 ShakespeareFan00 (talk) 17:52, 22 July 2024 (UTC)[reply]
The weighted average makes sense for additive colour mixing, but the colour resulting from pigment mixing is not so easily determined. For example, the colours   and   are complementary. Their sum in RGB colour models is   and their average is  , as grey as it gets. However, mixing red and green paint gives more of a brown colour.[14] A colour model that is more likely close to that of The Color Printer is the RYB colour model. If the pigments are fully opaque, the subtractive model is adequate, but generally pigments are not fully opaque.  --Lambiam 21:22, 22 July 2024 (UTC)[reply]

This is something I actually did! Back in the '80s, I was consulting for a company called ImageSet. One of our specialties was tools to allow people to take the equivalent of screenshots on MSDOS (using a TSR program), and then convert them to matching colors to be printed. We used trilinear interpolation to achieve the desired colors, if I remember right. It was a ridiculous amount of precision, especially considering that every CRT user adjusted the brightness etc. to their own favored wrong colors. --jpgordon𝄢𝄆𝄐𝄇 17:06, 27 July 2024 (UTC)[reply]

July 23

edit

Abel's reciprocity relation

edit

Do we have an article that covers Abel's reciprocity relation for differentials of the third kind? Tito Omburo (talk) 18:37, 23 July 2024 (UTC)[reply]

Google Books and Google Scholar searches for "Abel's reciprocity relation" do not yield any results,[15][16] so is this perhaps better known under a different name?  --Lambiam 21:22, 23 July 2024 (UTC)[reply]
I've seen it called the "first reciprocity law", but that's a bit non-specific. Tito Omburo (talk) 22:31, 23 July 2024 (UTC)[reply]
In this book I find a treatment of "The Reciprocity Theorem" for "Abelian differentials of the second and third kind", and in this one one of what is called "the reciprocity law for differentials of the first and third kinds". I'm only vaguely familiar with the first principles of differential geometry; the limited preview afforded by Google Books does not allow me to understand the notation and see how these two theorems, which superficially look rather different, are related. Both are apparently related to "Abel's theorem" on sums of integrals of certain types of function. We appear to treat only a very simple application of the latter theorem under the name "Abel's identity", so I am afraid the answer to your question is negative.  --Lambiam 09:29, 24 July 2024 (UTC)[reply]

Type of curvature?

edit

Where   (polar radius of curvature {RoC}) and   is the normal RoC, what is   (especially the vertex latitude,  )? Since it involves   it would suggest a type of curvature, rather than RoC—?
Likewise with geocentric latitude,  , there is   (with   being the geocentric radius).
This is based on   (so, technically, isn't N the geographic prime vertical RoC and R, besides being the geocentric radius, is also the geocentric prime vertical RoC?). -- Kaimbridge (talk) 18:43, 23 July 2024 (UTC)[reply]

The ratio   can be rewritten as   where   is the eccentricity of the ellipsoid. Eccentricity is a dimensionless quantity, also when the radii   and   are expressed using units of length such as kilometres or miles, and so is this ratio. At the poles ( ) it equals   and at the equator ( ) it equals   If the radii are dimensioned lengths, then so is curvature; its dimension is then the inverse of length. Therefore there is no plausible way to interpret this dimensionless ratio as a form of curvature.  --Lambiam 21:05, 23 July 2024 (UTC)[reply]
But I thought the root definition of curvature is that its radius is its inverse, and anything else, here b', is just a modifier.
What about the idea of   and   being different prime vertical RoC (in the case of the foundational, parametric   latitude, the prime vertical RoC is just  )? -- Kaimbridge (talk) 03:57, 24 July 2024 (UTC)[reply]
I do not understand what you are asking. A geometric interpretation of these quantities? Indeed, the radius of curvature is the inverse of the curvature; my point was that if one is dimensioned, so is the other, while the ratio about which the question appeared to be is inherently dimensionless, so it cannot be some type of curvature. I also do not understand what you mean by "just a modifier". It is the prime-vertical radius of curvature at the pole, which you proceed to divide by that at at geodetic latitude    --Lambiam 08:17, 24 July 2024 (UTC)[reply]
By "just a modifier", I mean   prime-vertical curvature, so b' is a modifier of that curvature.
Okay, so what about   in this situation? Would   in   be considered the geocentric prime vertical RoC?
In terms of purpose/context,   and   are used (as vertex latitudes   and  ) in the geographic and geocentric (respectively) calculation of geodetic distance (rather than the usual   based, parametric calculation, which has a neutral value,  ), all using the same, iteratively found, geodetically adjusted longitude difference ( ). -- Kaimbridge (talk) 06:54, 25 July 2024 (UTC)[reply]
A radius of curvature at a given spot is the radius of an osculating circle. It can be viewed as a vector from the centre of that circle to the given spot. If the direction of this RoC, viewed as a vector, is the vertical direction, it is a vertical RoC. (If, moreover, the plane of the osculating circle is perpendicular to the meridian through the given spot, so it intersects the ellipsoid along the east–west direction, it is the local prime-vertical RoC.) Unless the ellipsoid is a sphere, the geocentric radius at any other spot than the poles or equator, viewed as a vector from the centre to that spot, is not in the vertical direction, so it is not the radius of a locally osculating circle and it is not particularly meaningful to interpret it as either vertical or as a RoC, let alone both.  --Lambiam 10:41, 25 July 2024 (UTC)[reply]


July 26

edit

I joined the X and Y axes together. What Wikipedia page already mentions the concept?

edit

I had / have this fairly rather long blabbersome brilliant idea. My question is since there is no way that I, with a rather low IQ, could come up with something "new", then my idea certainly must be merely a rehash of some ideas already mentioned on several Wikipedia pages, in fact probably just a sentence on just one page. But which one(s)? Thanks. Jidanni (talk) 04:56, 26 July 2024 (UTC)[reply]

Your page is cumbersome to follow, but if I'm correct in interpreting it, you are essentially proposing the use of a pairing function or Hilbert curve. It is not possible to continuously reduce dimension in this manner (more precisely, 1D and 2D space are not homeomorphic). It would help if you would more rigorously formulate the function you are proposing rather than merely using examples.--Jasper Deng (talk) 06:22, 26 July 2024 (UTC)[reply]
Wikipedia defines "pairing function" only for natural numbers, but below I use the term for (not necessarily unique) functions wrapping up two values (not necessarily integers) into a single one of the same type.
Letting   stand for the unit interval   the Hilbert curve can be described as a function   Input one number   output   a pair of numbers. This function is surjective, which implies that there exists an inverse pairing function   Being an inverse means that when   we have   Function   is also continuous, but it is not injective. Many output pairs are reached several times; for example,   So the inverse is not unique.
Numbers in   can be written in base 2; for example,     and   This expansion is not unique:   We can view these binary expansions as infinite sequences   The function   given by   interprets a binary expansion as a real number. This function   is continuous and surjective, just like function   before, so it has an inverse. But, as before, function   is not injective, so the inverse is not unique. However, by convention, it has a "canonical" inverse: other than   the only possible expansion of   in the domain of   avoid sequences ending in an infinite stream of  s.
Now, using   we can define a bicontinuous pairing function   such that
 
This means that we can give a "canonical" definition for   by using   and its canonical inverse:
 
The function   can be described in the form of a 4-state finite-state automaton that gobbles up two streams of bits and produces a single stream of bits. It takes two bits at a time, one from each of the two input streams, and outputs two bits on the output stream.
I suspect that the "brilliant idea" is akin to this way of pairing   and   I expect the idea is well known, but perhaps only as folklore, and I doubt that it is described or even hinted at in Wikipedia mainspace.  --Lambiam, edited 10:51, 28 July 2024 (UTC) (originall 11:11, 26 July 2024 (UTC))[reply]

July 27

edit

Data estimation with excessive log functions

edit

In health care, I noticed that many estimation algorithms make extensive use of log functions. For example, the ASCVD 10-year risk estimation from "2013 ACC/AHA Guideline on the Assessment of Cardiovascular Risk" sums up a coefficient times the log of age, a coefficient times the log of total cholesterol, a coefficient times the log of HDL, etc... It is a set of coefficients, each multiplied by the log of an attribute. Is this type of function or algorithm the result of a specific type of data modeling? It looks to me like they took a sample data set and correlated the log of each attribute, one at a time, to the outcome and produced a coefficient that represents how correlated the log of that attribute is in the sample set. But, I'm just guessing and I'd prefer to know how this type of function is actually produced. 75.136.148.8 (talk) 10:54, 27 July 2024 (UTC)[reply]

I'm not familiar with how this estimator was devised, but model building is an art, especially in cases where the data is noisy and the causal processes are poorly understood. Social scientists routinely use purely linear regression models, because that is what they were taught as students, it is the default model of R, which many use, and everyone else in their field does this. When a variable (independent or dependent) can only assume positive values, it cannot have a normal distribution. This is an indication that pure linear regression may not be the best approach when devising an estimator. So then it is good practice to use a data transformation that makes the observed distribution more normal. I don't know if this is why they did what they did. Another possibility is that they just computed the correlation coefficients and saw they were higher when using a logarithmic scale.  --Lambiam 11:52, 27 July 2024 (UTC)[reply]

Are there other triangular numbers with all digits 6?

edit

6, 66, 666 are all triangular numbers, are there other triangular numbers with all digits 6? 218.187.67.217 (talk) 16:42, 27 July 2024 (UTC)[reply]

These correspond to solutions of the Diophantine equation  
 
 
 
For each solution, the number   is an all-6 triangular number.
I don't expect any further solutions, but neither do I see an argument exhibiting that they cannot exist. The weaker requirement   has four solutions for   for each given value of   corresponding to the final digits   For example, for   they are   The polynomials in   in the rhs of the Diophantine equation are irreducible. It seems that considerations based on modular arithmetic are not going to give further help.  --Lambiam 19:59, 27 July 2024 (UTC)[reply]
The discriminant of the quadratic is  . This needs to be a perfect square for there to be a solution, so we need   for some integer k. Since   will get "closer" to being an even perfect square as p approaches infinity, I heuristically wouldn't expect more than a finite amount of solutions to exist.--Jasper Deng (talk) 03:34, 28 July 2024 (UTC)[reply]
This gives yet another way of phrasing the problem. Define the recurrent sequence   by:
 
It goes like this:
 
The first four values are squares. Will the sequence ever hit another square?  --Lambiam 10:05, 28 July 2024 (UTC)[reply]
It turns out that because the discriminant is added or subtracted to 3 and then divided by 2a=24 in the quadratic formula, there are even more stringent restrictions: the numerator has to be divisible by 24, so we must have   and thus  . That restriction alone would seem to greatly reduce the amount of candidates (only every other odd perfect square satisfies that).--Jasper Deng (talk) 04:49, 29 July 2024 (UTC)[reply]
If the sequence   ever hits another square   its square root   will satisfy this requirement. This can be seen as follows. For   since   and         The only residue classes for   modulo   that have   are   in all four cases,    --Lambiam 10:13, 29 July 2024 (UTC)[reply]
Right. For any modulus m you can use the recursion to easily compute ap mod m. It's a bit harder, but still possible to then determine if ap is a quadratic residue mod m. If it isn't then you can eliminate that ap as a non-square. Do this for a few thousand prime (or prime power) values of m and you have a sieve which only let's though those ap's that are square and a vanishingly small number of "false positives". (There are going to be some m where all the values of ap are quadratic residues, but this won't happen if 10 is a primitive root mod m, and this occurs at a relatively constant rate.) This could be implemented in Python (or whatever) fairly easily to eliminate all the non-square ap's up to some value, say p≤10000. Keep in mind that a10000 would have around 10000 digits, but there's no need for multiprecision arithmetic to carry this out. However, all you would be doing is creating a lower bound on the next highest square ap, you wouldn't actually be proving there are none. (That's assuming the sieve didn't produce an actual square ap with p≤10000.) It shouldn't be hard to use a probabilistic argument to show that the "expected" number of squares is finite, but this wouldn't be a proof but rather an indication that it's unlikely that there will be additional squares above a given bound. In any case, I couldn't think of anything that would answer the original question better than a somewhat wishy-washy "probably not". --RDBury (talk) 13:10, 29 July 2024 (UTC)[reply]


July 30

edit

Axiom of choice, axiom of countable choice, any others?

edit

We have the Axiom of countable choice, which is weaker than Axiom of choice, so is it possible, meaningful or even already done to have an Axiom of aleph 1 choice or Axiom of cardinality of continuum choice, weaker than the Axiom of choice? Also, could the Axiom of dependent choice actually be such an axiom, corresponding to some particular aleph?Rich (talk) 20:20, 30 July 2024 (UTC)[reply]

The notion of cardinality is a fragile one without AC, but you can certainly define the axiom "If   is a sequence of nonempty sets, the product is nonempty", and that could be reasonably thought of as   choice. That's stronger than dependent choice (DC), since it's enough to construct Aronszajn trees, and you can arrange that the Solovay model satisfies DC + no Aronszajn trees. I'm confident it's weaker than full choice, but I don't know enough about symmetric extensions to show it.--Antendren (talk) 06:40, 31 July 2024 (UTC)[reply]
Interesting. it makes me wonder if a hierarchy of AC axioms as adjuncts to ZF, akin to what i've read about a hierarchy of Large cardinal axioms added to ZFC could be discovered.Rich (talk) 02:29, 1 August 2024 (UTC)[reply]

August 1

edit

Humanities

edit

July 17

edit

Monograms or cyphers with double letters

edit

Do you know any monogram or cypher (royal or otherwise) that use the same initial twice and facing in opposite directions (for example a capital letter and its mirror image) either back to back (the inverted letter first, when using a left-to-right script) or facing each other (first the usual letter). I'm specifically interested in the Latin alphabet but if you've got examples in other scripts, do include them. I know there's a cursive double L that Louis XIV used. I'd thought there was a double E that some Elizabeth (Elizabeth I?) or other used but I can't find it. Anyway, any example is good and gratefully accepted. 178.51.74.75 (talk) 01:58, 17 July 2024 (UTC)[reply]

c:File:Royal Monogram of King William IV of Great Britain, Variant.svg kinda seems like it does, while c:File:Royal Monogram of King George V of Great Britain.svg achieves a very similar effect by using different letters cleverly. Folly Mox (talk) 02:22, 17 July 2024 (UTC)[reply]
Also the monogram of the late Prince Philip which shows a double "P", and appears in the logo of The Duke of Edinburgh's Award. Alansplodge (talk) 08:53, 17 July 2024 (UTC)[reply]
The monogram of Prince Carl Philip of Sweden has been made artificially symmetric by intertwining CP with its mirror image CP.  --Lambiam 11:53, 17 July 2024 (UTC)[reply]
There are a couple of examples in the gallery at royal cypher that seem to fit the bill, most obviously that of King Carol II of Romania (not very clear in our photo, but described as "two opposed Cs"). That of King George I of Greece appears to be two crossed Γs (the Greek letter gamma). There are also a couple that don't quite fit your criteria because the letters themselves are symmetrical and so you can't tell what direction they're facing: Sheikh Maktoum bin Rashid Al Maktoum of Dubai (seems to include two Ms) and King Michael I of Romania (four Ms with both reflectional and rotational symmetry). Proteus (Talk) 09:42, 17 July 2024 (UTC)[reply]
There are a several examples in commons:Category:Royal monograms of Denmark and commons:Category:Royal monograms of Sweden (and presumably in other subcategories of commons:Category:Royal monograms).  --Lambiam 12:08, 17 July 2024 (UTC)[reply]
As an "otherwise" example, the Chanel logo [[17]] 2A01:E0A:CBA:BC60:B108:C36D:53EC:399 (talk) 11:36, 18 July 2024 (UTC)[reply]
J. R. R. Tolkien
 
-- Verbarson  talkedits 21:53, 18 July 2024 (UTC)[reply]
Thanks guys. Is there any theory out there as to how this originated. Would this have originated as an abbreviation of a name being written in two different directions (the right-to-left form being a mirror image of the name)? Problem is I've never ever seen anything like that? Have you? Another possibility: it symbolizes the name (represented by its initials) being "broadcast" in two directions? Speculation. Any actual theory put forward in a reliable source? Also: any idea who first originated this sort of design? How old it is? 178.51.74.75 (talk) 02:42, 19 July 2024 (UTC)[reply]
Symmetry is (or can be) beautiful? I doubt there is any relationship with boustrophedon writing, though Tolkien (see example above) said that some elves wrote bi-directionally. -- Verbarson  talkedits 07:54, 19 July 2024 (UTC)[reply]
"the development of the more stylised royal cypher seems to date from the reign of William and Mary. To emphasise their joint rule, their initials were interlaced and – apparently simply to add symmetry – the first R was reversed."[18]  --Lambiam 10:46, 19 July 2024 (UTC)[reply]
Another example: ABBA (band)#Official logo. --142.112.148.225 (talk) 04:31, 19 July 2024 (UTC)[reply]
The monogram of King Frederik X of Denmark have two mirrored letter F. See Frederik X#Personal symbols. Dipsacus fullonum (talk) 10:04, 20 July 2024 (UTC)[reply]

July 18

edit

When was "Confessions of a Yakuza" (Junichi Saga) released?

edit

I tried adding a short description and later found that there's no uniform "publication date" I've seen on Wikipedia and on other sites.

  1. The Wikipedia page for Confessions of a Yakuza says it was released in 1991.
  2. The author's Wikipedia page (Junichi Saga) says the book was published in 1989.
  3. An entry on Google Books states thr book was published in 1995.
  4. Its listing on the Internet Archive states its publication year as 1995.
  5. In GoodReads, the book was saud to be originally published in London and Tokyo in 1991.

I tried checking, and the Internet Archive copy of the book states:

The original publication was in 1989, in Nihonggo. It was in 1991 when Kodansha International Ltd. published the book. I originally wanted to ask the question here, but I might let other people know of this concern.

Also, I still can't identify whether the book is fictional or not (i.e. biographical, or a fictional character in nonfictional world). RFNirmala (talk) 13:52, 18 July 2024 (UTC)[reply]

The original Japanese version was published in 1989. The translation was originally published in 1991, but under the title The Gambler's Tale (see here). The 1995 version (the "first paperback edition" from Kodansha) corroborates both of these dates, giving the publication date as 1989 and the copyright date for the translation as 1991. Dekimasuよ! 14:09, 18 July 2024 (UTC)[reply]
Thank you? Is it possible to help me in editing the articles accordingly? I can't edit much in the following days RFNirmala (talk) 14:57, 18 July 2024 (UTC)[reply]

July 19

edit

Was Isaac spared in the Junius Manuscript?

edit

Apparently the story of Abraham and Isaac occurs twice in the book. Do these stories agree, and do they belong to the category of traditions in which Isaac was sacrificed? Temerarius (talk) 02:37, 19 July 2024 (UTC)[reply]

The story of the sacrifice of Isaac is only told in Genesis A. In lines 2908–2922, a messenger from God commands Abraham not to slay his remaining son.  --Lambiam 10:12, 19 July 2024 (UTC)[reply]
Stout of heart he mounted the high downs, and his son with him, according as Eternal God commanded, until he stood upon the ridge of the high land in the place which the Firm and Faithful Lord had showed him. And there he built a pyre and kindled a flame and bound his son, hand and foot, and laid Isaac, the lad, on the altar, and seized his sword by the hilt. With his own hand he would have slain him, and quenched the flame with the blood of his son. Then a thane of God, an angel from on high, called unto Abraham with a loud voice. In stillness he abode the herald's message and answered the angel. Swiftly the glorious minister of God addressed him from the heavens: "Slay not thy son, dear Abraham, but take the lad from the altar alive. The God of glory is gracious unto him! Great shall thy reward be, Hebrew prince, true meed of victory and ample gifts, at the holy hands of the Heavenly King. The Lord of spirits will bless thee with His blessing because His love and favour were dearer unto thee than thine own son." [19]
Alansplodge (talk) 10:32, 19 July 2024 (UTC)[reply]
There are indeed two passages in that translation Alansplodge linked to, one in section XLI (around lines 2897-2908), and one in section XLVII (starting with lines 397-416). The second seems to be part of a summary of the genealogy of the patriarchs injected within a narrative of the Exodus. Both passages contain the sacrifice story including the divine command to spare Isaac in the end. Fut.Perf. 10:56, 19 July 2024 (UTC)[reply]
Many narratives now in the 'Old Testament' are amalgams of oral tales that originally had different variations in different communities, and even after initial redaction may have been further modified later to suit a changing religious and/or political agenda. There is no doubt that the early Canaanites, who included those who later differentiated themselves as Israelites and Judahites, practiced human sacrifice, probably including of firstborns, so extra- (and pre-) biblical variants of the Binding of Isaac story where Jacob was sacrificed probably existed: the eventual biblical version likely reflects a 'policy change' of child sacrifice being abandoned as part of a general distancing from other Caananites, and identity assertion as a separate people (see Canaanite religion#Practices).{The poster formerly known as 87.81.230.195} 94.2.67.235 (talk) 10:50, 20 July 2024 (UTC).[reply]
We're talking here, though, about poems in Old English written in the late 10th century CE. The only knowledge its authors would have of ancient Caananite practices would be through the medium of the Old Testament.  --Lambiam 13:38, 20 July 2024 (UTC)[reply]
In RSV Judges 11:30-39, the Israelite warlord Jephthah swore that in exchange for a victory against the Ammonites, the first person who "comes forth from the doors of my house to meet me, when I return victorious from the Ammonites, shall be the Lord’s, and I will offer him up for a burnt offering." Sadly, that person was the daughter, whom he loved. She agreed to be sacrificed because of his pledge. "And at the end of two months, she returned to her father, who did with her according to his vow which he had made. She had never known a man. And it became a custom in Israel that the daughters of Israel went year by year to lament the daughter of Jephthah the Gileadite four days in the year." Note: these Ammonites were the Middle Easterners, not the Mesozoic Cephalopods. Edison (talk) 22:05, 22 July 2024 (UTC)[reply]

When Treasuries mature on a holiday

edit

I just bought a Treasury bill maturing on 12/31 with one of those brokerages that pay out only the day after. As that will be New Year's Day, my money basically doesn't earn interest for two days, compared with a brokerage that pays out on the day of. Suppose it were a long holiday. That would be money out of reach for even longer. Hardly seems fair. Imagine Reason (talk) 14:03, 19 July 2024 (UTC)[reply]

Do you have a question? --Error (talk) 14:47, 19 July 2024 (UTC)[reply]
Sorry, are there any similar problems in financial transaction? Imagine Reason (talk) 19:12, 19 July 2024 (UTC)[reply]
Not sure if this is what you are asking about, but in banking there is often a delay between when you deposit funds in a checking account, and when those funds become available for withdrawal. This can be several days if you deposit over a long weekend. Blueboar (talk) 14:21, 20 July 2024 (UTC)[reply]
And during that interval, even if merely overnight, the bank can invest the funds in the Money market and collect the interest paid on it. For an individual deposit this will only yield a small amount, but when the yields of all such funds held by the bank are aggregated, the total is significant.
The same applies to funds paid to a solicitor for imminent property purchases and so on. In the UK, for some transactions the solicitor is legally obliged to do so, and pay the interest gained to their client; in others they can keep the money themselves. [Disclosure: my father was a solicitors' chief accountant, and made such investments daily.] {The poster formerly known as 87.81.230.195} 94.2.67.235 (talk) 10:14, 21 July 2024 (UTC)[reply]
Be thankful you are not charged negative interest.  --Lambiam 13:44, 20 July 2024 (UTC)[reply]

Conflicts between Soviet Union and Russian SFSR

edit

Have there ever been any conflicts between the Soviet Union and the Russian SFSR when Russia was still a part of the Soviet Union? JIP | Talk 20:25, 19 July 2024 (UTC)[reply]

1991 Soviet coup attempt lists several Soviet organizations on one side and the RSFSR on the other. --Error (talk) 00:36, 20 July 2024 (UTC)[reply]
(ec) The Russian SFSR had a totally dominant position in the Soviet Union until 1990, so any conflicts must date from the brief period from the declaration of State Sovereignty of the Russian Soviet Federative Socialist Republic on 12 June 1990 and the dissolution of the USSR on 26 December 1991.  --Lambiam 00:45, 20 July 2024 (UTC)[reply]
Define conflict. There was certainly some competition regarding delimitations of decision-making, that always happens. But what is important in this case is that there was no republican-level RSFSR Communist Party until the very end of the Soviet Union, unlike all other SSRs. Generally speaking power-struggles happened not between governments of administrative units of the USSR but between party organs (state organs implemented party policy, not the other way around). So the management of the RSFSR state government was directly under the Central Committee of the CPSU, the same party body controlling the USSR government. Towards the end stage of the USSR Russian nationalism was increasing in importance, in the sense of wanting more self-rule of RSFSR. The foundation of the Communist Party of the Russian Soviet Federative Socialist Republic was the key step in this direction. --Soman (talk) 13:25, 20 July 2024 (UTC)[reply]
While Russia had its own Supreme Soviet, as Soman notes it didn't have as many independent institutions as the other Union Republics, making it institutionally less powerful than the rest, even while its demographic dominance made Russians dominant throughout the Union. In my mind, this situation was somewhat comparable that of to England and the UK; e.g. Scotland, Wales, and NI have their own devolved parliaments, but there's no devolved English parliament, even while England is rather dominant demographically. Of course, the typical dominance of the Party leadership over both Party and government, at all levels, meant that there weren't frequent conflicts or disputes between the USSR and the RSFSR; there was nothing comparable to the West Lothian question, as far as I'm aware. Both scenarios have grown out of the same basic situation: a powerful core state of several centuries ago was able to conquer and annex adjoining territories, which in the 20th century were given additional autonomy, but there was no history of the core state being separate from the whole polity (because the core state functionally was the whole polity), and thus the top-level politicians didn't give comparable autonomy to the core. See asymmetric federalism for a discussion of the underlying concept. Nyttend (talk) 22:14, 23 July 2024 (UTC)[reply]
Tony Blair's solution in the UK was "regional assemblies" in eight regions of England, but that didn't work out... AnonMoos (talk) 19:14, 26 July 2024 (UTC)[reply]
Bring back the Heptarchy, I say. (There actually is a Wessex Independents [sic] Party) {The poster formerly known as 87.81.230.195} 94.2.67.235 (talk) 20:33, 28 July 2024 (UTC)[reply]

July 20

edit

Should Afghanistan be considered part of West, Central, or South Asia?

edit

exactly what it says on the tin

Duckmather (talk) 04:11, 20 July 2024 (UTC)[reply]

Based on the article, South Asia. GalacticShoe (talk) 04:21, 20 July 2024 (UTC)[reply]
Also, Afghanistan is a member of the South Asian Association for Regional Cooperation.  --Lambiam 09:39, 20 July 2024 (UTC)[reply]

Look, these geographic units are completely arbitrary. You can consider Afghanistan as part of all of these regions, and these regional identities are not necessarily mutually exclusive. If you define 'West Asia' as Greater Middle East, then Afghanistan can be part of it, but let's be clear that the notion of a 'West Asian' region is a construct that only exist in the mind of people outside the supposed region. In terms of Central Asia vs South Asia, Afghanistan doesn't match perfectly in neither, Afghanistan was not directly part of Soviet Union as the rest of Central Asia and was not directly part of British empire as the rest of South Asia. Then the UN georegions is a different story, but UN region allocation is not a particularly important categorization except for UN employees. --Soman (talk) 13:36, 20 July 2024 (UTC)[reply]

And it's confusing for UN employees too. WHO places Afghanistan in the ludicrous 'Eastern Mediterranean Region'. OCHA where I work, at least places Afghanistan in the somewhat more sensible 'Asia Pacific' region. (There is a concept called the "One UN" that nobody in the UN believes in.) Hayttom (talk) 00:40, 21 July 2024 (UTC)[reply]
And other entities don't put it in Asia Pacific; Australia Post has five international pricing zones (New Zealand, Asia Pacific, US and Canada, UK and Europe, and Rest of the World), and per their zones page, Afghanistan is "Rest of the World", like Iran, but unlike India or Pakistan. I suppose the situation is like that of Germany and Austria: are they Central or Western Europe? See plenty of discussions of this subject at Talk:Germany. Nyttend (talk) 22:19, 23 July 2024 (UTC)[reply]
{{ping|Soman}} Here's a follow up question: Culturally, which of these three regions is Afghanistan most similar to? Duckmather (talk) 00:50, 23 July 2024 (UTC)[reply]
Hi @Duckmather:. Ping template won't work if you place nowiki tags around it. I don't know, to be frank. Depends a bit on what criteria of 'culture' you are looking at. Afghanistan is majority Pashtu, in the border areas of Pakistan you'd find a lot of commonalities in terms of food, clothing, language, religion, etc. But what would be the 'cultural' link between Herat and Jaffna? I'm tempted to say that Central Asia is somehow closer, at least in terms of cuisine, but Central Asian socieities are generally much more secularized. Whilst Afghanistan has large Tajik and Uzbek populations my personal understanding is that there isn't the same level of cross-border cultural linkages as with the Pakistan border areas. But I could be wrong here. --Soman (talk) 12:23, 23 July 2024 (UTC)[reply]
"West Asia"? Isn't that Europe?--User:Khajidha (talk) (contributions) 14:08, 24 July 2024 (UTC)[reply]

Word for office selling lottery tickets

edit

Is there a word in English for a office selling lottery tickets? Dipsacus fullonum (talk) 09:30, 20 July 2024 (UTC)[reply]

Lottery tickets are sold by "lottery retailers", which are typically not specialized offices but, for example, tobacco shops, gas stations and convenience stores. For example, tickets for Landbrugslotteriet in Denmark are sold by many, diverse retailers.[20] In some countries they are also sold by street hawkers.[21]  --Lambiam 10:05, 20 July 2024 (UTC)[reply]
This is true now, but historically there have been many offices/stores in Denmark whose sole purpose has been to sell lottery tickets. The Danish word for such lottery sales outlets is “lotterikollektion”. My question is whether there is an equivalent word in English for a shop that only sells lottery tickets. Dipsacus fullonum (talk) 10:20, 20 July 2024 (UTC)[reply]
Tickets for the National Lottery (United Kingdom) can be bought from newsagents, supermarkets and local grocer's shops, although at the outset in the 1990s, post offices were the main outlets.
I was curious about the 17th and 18th century goverment lotteries, such as the Million Lottery, but it seems that the £10 tickets could only be afforded by the very wealthy and were obtained from the Bank of England through a stock broker. [22] Alansplodge (talk) 11:09, 20 July 2024 (UTC)[reply]
As far as I know, none of the English speaking countries ever had shops dedicated to just selling lottery tickets, so they wouldn’t need a word for such shops. Blueboar (talk) 12:53, 20 July 2024 (UTC)[reply]
(edit conflict) If there were a need for a word, then "lottery office" would probably suffice. English is not fond of the compound words that are found in other Germanic languages. Alansplodge (talk) 15:21, 20 July 2024 (UTC)[reply]
English is very fond of them (see Noun adjunct), but it only spells them as single words (with no internal space or hyphen) in limited circumstances... AnonMoos (talk) 18:23, 20 July 2024 (UTC)[reply]
When I was in Portugal the streets were full of people selling lottery tickets. I assumed this was normal for countries operating a lottery (basically all of them except Britain). There was also the Totobola, a kind of national football pool which I believed to be widespread in Europe, although I don't know who sold the tickets - the results were announced on the radio at 7 PM on a Sunday night. When the British lottery came in it was controlled almost as much as the football pools, the emphasis being on tickets not being available on the street, and the pools seem to have largely died out. Similar coupons which were not pools were not so restricted - fixed odds football coupons are available in betting shops.[Edit] Totobola is apparently run by the Roman Catholic Church. 2A00:23A8:1:D801:A94B:5D04:D466:F650 (talk) 15:17, 20 July 2024 (UTC)[reply]
Google Maps seems to prefer "lottery retailer". Abductive (reasoning) 21:01, 20 July 2024 (UTC)[reply]
Yes, but that refers to any store selling lottery tickets - which might also (primarily) be a gas/petrol station, a grocery store, a tobacconist, etc. Blueboar (talk) 21:36, 20 July 2024 (UTC)[reply]
The lottery distribution network is based on that of the football pools. Nobody is allowed to visit National Lottery headquarters - agents come to winners. Bookmakers were formerly "turf accountants". It was illegal to visit their offices (the only place a bookmaker was allowed to operate was on the racecourse or at a dog track). Off-course credit betting was allowed, but punters had to phone their bets in (or post them). Notwithstanding, punters did visit their local "turf accountant". Every so often offices were raided - advance notice was given by the police of who would be targeted. Bookmakers employed "runners" who would illegally visit factories, hand out betting slips and results and collect cash bets. Pools companies termed these people "collectors" - clients could not visit the pools companies' offices but had to post coupons with the stake for the previous week's losing coupons. Later they were allowed to enclose the current week's stake. This system enabled those who wanted to cut ties with the pools company to stop receiving supplies of coupons through the post - they would send in coupons without money. The companies would then write back to say that because of the clients' "bad debts" they were being removed from the mailing list for coupons. Some collectors visited clients to collect coupons and stakes and they may have had an office clients could visit - Kelly's Post Office London Directory 1976 has an entry for "Vernons Pools: area office (7), 556 Garratt la SW 17". Otherwise, collectors operated much like lottery outlets - they did not advertise, but if you went into their shop you might well see coupons on the counter. One place you would not see football pool coupons was in the betting shops after legalisation. Television was not allowed, so commentary and results came over a telephone line from the Exchange Telegraph Ltd, otherwise known for relaying share prices. Casino bets were settled by cheque. In the nineteenth century the Prince of Wales was involved in the "Royal Baccarat Scandal", when it was alleged the gambling was conducted in a house where gambling was a business (and hence illegal) rather than socially in a private home. 92.29.246.121 (talk) 14:09, 22 July 2024 (UTC)[reply]
Sorry, I don't see the connection between my question and this story from an unspecified location. Dipsacus fullonum (talk) 14:57, 22 July 2024 (UTC)[reply]
I do not see a response discussing how it was done in England. It used to be that a broker (the predecessor to stock brokers) purchased the right to sell lottery tickets from the crown. The broker then ran a brokerage, where lottery tickets could be purchased. But, successful brokers didn't require customers to come to them. They hired runners to go to the customers and sell the tickets. Also, specific to your question, there was no requirement that a brokerage could only sell lottery tickets. They certainly ran a successful business to have the income to purchase rights to broker the lottery tickets. 12.116.29.106 (talk) 15:35, 23 July 2024 (UTC)[reply]
Such shops appear to exist in at least one US state, in connection with the Ohio Lottery. Growing up, I remember seeing shops with names such as "So-and-so's Lotto", which suggested to me that they were primarily or exclusively retailers of lottery tickets. (My bank account is too small, and my awareness of probability too large, to allow me to patronise such businesses, so I don't know if they sell anything else.) But I've never heard a term that includes them while excluding convenience shops, groceries, etc. Nyttend (talk) 22:00, 23 July 2024 (UTC)[reply]

What part of this is a hate crime?

edit

I'm reading this news article [23] about 2 men who allegedly spray painted “FREE GAZA” on an Israeli flag at a grocery store. The police are charging them with damage to property, disorderly conduct and hate crimes. I get why they're being charged with damage to property and disorderly conduct, but I'm not sure what part of this is a hate crime. If they had spray painted "kill all the k***s", that would be hate crime. I don't know if a serious political opinion like "free Gaza" is being treated as a hate crime. Or maybe there's more to what happened then was explained in the news article? A Quest For Knowledge (talk) 23:40, 20 July 2024 (UTC)[reply]

The example with the slur is gratuitous (I'd suggest refactoring your question) and we are not going to be able to solve all of the real or philosophical problems involved in the war here, but to answer the question it would make sense to go to the applicable criminal code, which is here. The two are accused of "misdemeanor criminal damage to property", and the applicable law defines this as a hate crime when it involves "the actual or perceived race, color, creed, religion, ancestry, gender, sexual orientation, physical or mental disability, citizenship, immigration status, or national origin of another individual or group of individuals, regardless of the existence of any other motivating factor or factors." Presumably the prosecutors will argue that this applies, but the accused have not been convicted of any crimes yet, so some jury will be tasked with answering the question you pose, or perhaps the accused will plead guilty to lesser charges. Dekimasuよ! 03:33, 21 July 2024 (UTC)[reply]
Somewhat obviously, the lawyers for the defendant may argue that the request to end the war was not directed to an individual or group of individuals (who would not be able to end the war anyway, whether they want it to end or not), but towards the State of Israel, represented by its flag. The flag was presumably put up to express support of Israel in view of the current conflict, and the spray-painting was clearly intended as a protest against such support, but AFAIK there is no reason to assume that the store employees who put up the flag were Jewish, and even less so that the alleged protesters knew their religion, ancestry, or national origin, which would be a mens rea requirement for this being a hate crime. While it is generally unpredictable how a jury will respond, this specific charge (not by the police, but by prosecutors), unless they know something not revealed in the news article, gives a bit the feeling of throwing things against the wall to see what will stick.  --Lambiam 08:48, 21 July 2024 (UTC)[reply]
Defacing any country's flag can be a hate crime. ←Baseball Bugs What's up, Doc? carrots18:08, 21 July 2024 (UTC)[reply]
Per Texas v. Johnson and United States v. Eichman, desecration of the Flag of the United States is protected speech and thereby not a crime.  --Lambiam 20:39, 21 July 2024 (UTC)[reply]
I wouldn't count on that ruling applying in this case. Also, can you imagine the outrage if someone vandalized a Gaza flag with pro-Israel writing? ←Baseball Bugs What's up, Doc? carrots22:20, 21 July 2024 (UTC)[reply]
(a) You wrote, "any country's flag". Last time I heard, the US was still a country. (b) I have a hard time imagining the hypothetical perp being charged with a hate crime. This may, of course, be due to a weakness of my power of imagination.  --Lambiam 13:56, 22 July 2024 (UTC)[reply]
You did not imagine our "hypothetical perp" might be Canadian, either that or something about human nature. fiveby(zero) 17:05, 22 July 2024 (UTC)[reply]
There is no "flag of Gaza" (there are the Palestinian and Hamas flags). Anyway, in U.S. law, a precedent was set in the Bremen flag incident of 1935, when a Nazi flag was destroyed, but at the time it had a somewhat ambiguous legal status in Germany (it certainly wasn't the official main national flag), so a U.S. court ruled that no diplomatic insult had occurred... AnonMoos (talk) 17:07, 22 July 2024 (UTC)[reply]
It will be up to a jury and the court system to decide whether it's a hate crime. ←Baseball Bugs What's up, Doc? carrots19:45, 22 July 2024 (UTC)[reply]
Also, someone could paint "free Gaza" on a Hamas flag, on the premise that it is Hamas who hold the Palestinians hostage and have brought this disaster upon Gaza. ←Baseball Bugs What's up, Doc? carrots19:47, 22 July 2024 (UTC)[reply]
You think Gaza was free before the Hamas takeover? DuncanHill (talk) 19:55, 22 July 2024 (UTC)[reply]
Compared to what they have become? So who's the idiot?[24]Baseball Bugs What's up, Doc? carrots20:06, 22 July 2024 (UTC)[reply]
Nothing to do with flags, but though Gaza around 1980 was no paradise, Gazans had a large degree of freedom of movement then, and there were no heavily-fortified borders. The difference between then and now is almost entirely due to the effects of decades of attacks on Jews emanating from Gaza... AnonMoos (talk) 21:38, 22 July 2024 (UTC)[reply]
Why do you choose 1980? Hamas was founded in 1987, Israeli blockade began in 2005, Hamas Gaza takeover was 2007. But you know that already. There have been decades of attacks on Palestinians emanating from Israel... But you know that too. DuncanHill (talk) 09:35, 24 July 2024 (UTC)[reply]
Maybe you've forgotten the terrorist attacks on Israelis, in particular the one during the 1972 Summer Olympic Games. ←Baseball Bugs What's up, Doc? carrots13:13, 24 July 2024 (UTC)[reply]
I presume that part of the issue here is that it isn't just "free Gaza" but that "free Gaza" has specifically been spraypainted onto an Israeli flag, which directs it "at" someone much more obviously than spraypainting "free Gaza" onto a local bridge or abandoned building or what-have-you. But to answer your question in general, you may be interested to read this article, which outlines how this sort of thing has gone recently in Canada. -- asilvering (talk) 04:16, 21 July 2024 (UTC)[reply]
"Free Gaza" is protected speech. Vandalizing someone else's property is not. ←Baseball Bugs What's up, Doc? carrots23:56, 22 July 2024 (UTC)[reply]

July 21

edit

Attestations of Ishtar in later eras

edit

I've been under the impression that the goddess Ishtar is not much attested after the bronze collapse. Not necessarily that the goddess disappeared, but that the name went out of vogue. What late inscriptions are there?

Temerarius (talk) 15:02, 21 July 2024 (UTC)[reply]

If she is considered the same goddess as ʿAštart, she was worshipped into the 2nd century CE.  --Lambiam 20:31, 21 July 2024 (UTC)[reply]
I'm looking specifically for the latest attestations of the spelling Ishtar.
Temerarius (talk) 00:09, 26 July 2024 (UTC)[reply]
In ancient times, each relevant language had its own adapted version of the deity name (Greek Astarte, Hebrew `Ashtarot, etc). The alphabetic spelling "Ishtar" didn't exist until the 19th-century decipherment of Cuneiform script by modern European scholars. The cuneiform spelling of Ishtar could not have continued in use longer than cuneiform script itself did; according to our article, "The last known cuneiform inscription, an astronomical text, was written in 75 AD. The ability to read cuneiform may have persisted until the third century AD." -- AnonMoos (talk) 19:16, 26 July 2024 (UTC)[reply]

July 22

edit

List of authors of the Atlas Van der Hagen

edit

Is there such a thing as a list of authors of the Atlas Van der Hagen? In case this is an easier question: What I really want to know is whether John Ogilby was one of them. Elinruby (talk) 02:31, 22 July 2024 (UTC)[reply]

File:Atlas Van der Hagen-KW1049B13 058-The City of ALGIER.jpeg has "Artist/Author John Ogilby". It is not clear to me, though, what this is based on.  --Lambiam 14:32, 22 July 2024 (UTC)[reply]

Where is this shop, in "Portway", United Kingdom?

edit
 
Postcard hand-labelled "The Stores, Portway"

The reverse of the above postcard, published in the United Kingdom, is hand-lettered "The Stores, Portway". Where is or was it? Can we date it more precisely? Andy Mabbett (Pigsonthewing); Talk to Andy; Andy's edits 09:02, 22 July 2024 (UTC)[reply]

As it's you, I'm presuming the Portway is the one in Rowley Regis, West Midlands. Which would put it here, on 11 Portway Rd (there's a matching hill behind too).
https://www.google.com/maps/@52.4934044,-2.0417817,3a,35.9y,311.95h,93.22t/data=!3m7!1e1!3m5!1sXN-dVEbwjrkejOaLY5x_2Q!2e0!6shttps:%2F%2Fstreetviewpixels-pa.googleapis.com%2Fv1%2Fthumbnail%3Fpanoid%3DXN-dVEbwjrkejOaLY5x_2Q%26cb_client%3Dmaps_sv.share%26w%3D900%26h%3D600%26yaw%3D311.951541624746%26pitch%3D-3.2204070573608448%26thumbfov%3D90!7i16384!8i8192?coh=205410&entry=ttu Andy Dingley (talk) 09:45, 22 July 2024 (UTC)[reply]
Thank you. Others agree, though it's coincidence that it's local to me. Andy Mabbett (Pigsonthewing); Talk to Andy; Andy's edits 13:40, 22 July 2024 (UTC)[reply]
Details such as the shape of the house and roof, the brickwork and placement of the chimney and doors and windows agree, so there can be little doubt it is the same building, unless a very similar building did not survive. (The building at 9 Portway Rd is its mirror image, and 20 Portway Rd is very similar, so more of the same type may have been built.)  --Lambiam 13:43, 22 July 2024 (UTC)[reply]
Here is a specimen of the same type of post card, estimated to be from "circa 1910s".[25] Image searches did not turn up ads or packaging for Fairy Soap, Lyons Tea or Lyons' Extract with matching text and lettering.  --Lambiam 13:25, 22 July 2024 (UTC)[reply]
Thank you. Andy Mabbett (Pigsonthewing); Talk to Andy; Andy's edits 13:40, 22 July 2024 (UTC)[reply]

I was amused to notice that the front door advertised Lyons' tea and coffee while the adjacent window advertised that "People who know drink Lyons' Extract" instead. Either Lyons was hoping for a presence in all markets or else they seem to have been undercutting themselves! --142.112.148.225 (talk) 02:39, 23 July 2024 (UTC)[reply]

Tea, coffee, and (coffee & chicory) extract were three different products from J. Lyons and Co.'s extensive range of food products. Tea and coffee are beverages, Lyon's extract was mainly a cooking ingredient. Hardly surprising that a grocery would carry all three. {The poster formerly known as 87.81.230.195} 94.2.67.235 (talk) 11:34, 23 July 2024 (UTC)[reply]

July 23

edit

File:Secret Service Director Hosts RNC Public Safety Briefing.webm

edit

Any clue who the people in this video are (other than Kimberly Cheatle obviously). I assume they must be high ranking officials and not just random SS agents Trade (talk) 00:19, 23 July 2024 (UTC)[reply]

The man is FBI Special Agent in Charge Michael Hensle.[26]  --Lambiam 13:32, 23 July 2024 (UTC)[reply]
The woman is U.S. Secret Service 2024 RNC Coordinator Audrey Gibson-Cicchino.[27]  --Lambiam 13:59, 23 July 2024 (UTC)[reply]

"Mary was not the mother of Jesus"

edit

I'm not quite managing to wrap my head around this bit in Trojan Horse scandal:

At the Oldknow academy, children were asked whether they believed in Christmas and encouraged to chant "no we don't" in response. The pupils were told at an assembly not to send Christmas cards and that Mary was not the mother of Jesus.

The original source is the "Kershaw Report", I believe (see opening paragraph of the article lede for context):

We are told that in or around December 2013, a Christmas assembly was held during which statements were made that children should not be sending Christmas cards, that Jesus was not born of Mary and that it was unbelievable that Christians believe in the Christmas story; children were encouraged to chant `No, we don't' when asked questions such as `Do we celebrate or believe in Christmas?' and whether they believed that Jesus was born on Christmas day.

The basic idea is that this is portraying "Islamists" disparaging Christianity by disparaging Christmas, I take it. Claiming that Mary was not the mother of Jesus seems a strange way to go about that, though. Going by Mary in Islam and Jesus in Islam, they're both somewhat major figures in the Koran (dozens to hundreds of mentions), and their relationship is clear and no different than in the Christian version. The modern Christmas tradition is such a hodgepodge that it takes practically no effort to pick any number of holes in it, so why come up with something so outlandish?

Maybe this is just a straightforward mistake? Like, someone at the assembly in question did say that, but that someone was a giant ignoramus, or someone else downstream misunderstood or misremembered what was said? Or maybe I'm missing something... which is why I'm posting this.

Thanks for any elucidation!

- 2A02:560:4DE3:6F00:8DF4:9B2E:34:B383 (talk) 15:57, 23 July 2024 (UTC)[reply]

Double quotation marks are for quotations in general. Single quotation marks are for quotations within quotations. (I believe that in some old forms of English it might have been the other way around, but I'm sure it is this way in most forms of English today.) Georgia guy (talk) 16:22, 23 July 2024 (UTC)[reply]
wp:deny
The following discussion has been closed. Please do not modify it.
The source for the claim that children were told that Jesus was not born of Mary is behind a paywall. Can you quote the relevant passage? 92.29.249.130 (talk) 16:36, 23 July 2024 (UTC)[reply]
This one?

At the Oldknow academy, children were told at an assembly that they should not send Christmas cards and that Mary was not the mother of Jesus. Children were asked whether they believed in Christmas and encouraged to chant "no we don't".

I'm assuming their source is the report I quoted in the OP, so I skipped straight to that.
- 2A02:560:4DE3:6F00:8DF4:9B2E:34:B383 (talk) 16:46, 23 July 2024 (UTC)[reply]
Generally speaking, the usual strategy is to flip between single and double at each level, AFAIK. So quotations within quotations use whatever the outer quotation didn't use, and quotations within quotations within quotations go back to whatever the outer quotation did use, and so forth. Using singles for the outer quotation is a bit, but not a lot, unusual in modern writing, I'd say.
- 2A02:560:4DE3:6F00:8DF4:9B2E:34:B383 (talk) 16:43, 23 July 2024 (UTC)[reply]
In American English, the double quotation marks are on the outside, the singles on the inside. In British English, the opposite is common, but not universal; a habit originating in the late 19th century. There are more languages where single outside – double inside is common (for example Dutch, which has copied British habits). PiusImpavidus (talk) 08:41, 24 July 2024 (UTC)[reply]

The Mary and Jesus of Islam differ in a number of ways from the Mary and Jesus of the Bible or Christianity: In the Qur'an, Mary is identified with Miriam of the Old Testament (the daughter of Amram and the sister of Aaron), while Jesus did not die on the cross, but only appeared to do so (reviving the old Docetic heresy). However, there's nothing about Jesus not being the son of Mary, and I don't think that such a belief is mainstream among Muslims, according to any information I have. However, disrespecting non-Muslim holidays is more widespread (some varieties of Sunnism in Saudi Arabia, Pakistan, etc. disrespect those Shi`i holidays which are not also practiced by Sunnis). AnonMoos (talk) 21:35, 23 July 2024 (UTC)[reply]

One possible explanation is that the author of the report made up the story that the children were told that Mary was not the mother of Jesus, ignorant of their significance and relationship in Islam.  --Lambiam 22:35, 23 July 2024 (UTC)[reply]
Possible, yes, but again, it doesn't make much sense to me that someone simply making stuff up would come up with this, when there's such a wealth of more pertinent and plausible ways to criticize Christmas. The explanation that seems least unlikely to me at this point is that this is a case of Chinese whispers, basically: Something was said at the assembly, and something ended up in the report as a result, but there was so much distortion in between that the sense was largely lost. Especially if the report relied on a single account for this, and if the person giving the account was a child.
- 2A02:560:4DE3:6F00:8DF4:9B2E:34:B383 (talk) 23:36, 23 July 2024 (UTC)[reply]
If the author made this inflammatory accusation based on unverifiable hearsay, it is just as bad as making it up. In either case they are ignorant about the religion they seek to disparage. Criticism of Christmas as having become a pagan feast in servitude of Mammon is also raised by Christians. Whatever the origin of the story, the reason it was selected for the report must have been that the author knew it would offend the religious sensibilities and draw the ire of a segment of his audience.  --Lambiam 20:26, 24 July 2024 (UTC)[reply]
To put it another way, the author wanted publicity. ←Baseball Bugs What's up, Doc? carrots21:08, 24 July 2024 (UTC)[reply]

July 24

edit

un fine

edit

Inspector Maigret (Rupert Davies) has just walked into a bar and asked for "un fine" and to talk to the patron. It's far too long since I drank in France, was the commissaire asking for a cognac? Thanks, DuncanHill (talk) 23:51, 24 July 2024 (UTC)[reply]

It is a brandy: a cognac or possibly one of the others listed on the French Wikipedia [28] (upon translating the page with Google). Modocc (talk) 06:23, 25 July 2024 (UTC)[reply]
Wikt:fine#French:
Noun - fine f (plural fines) - a number of high grade French brandies (usually AOC certified). Alansplodge (talk) 13:29, 25 July 2024 (UTC)[reply]
Hopefully, monsieur le commissaire asked for une fine, or he might have been served with a fine for grammatical impropriety.  --Lambiam 19:34, 25 July 2024 (UTC)[reply]

July 25

edit

Path of would-be Trump assassin Crooks

edit

There is conflicting information when, during the Attempted assassination of Donald Trump, the would-be assassin Thomas Matthew Crooks was first spotted on "the roof" or "a roof". While our article currently follows media who claim that he had been spotted on a/the roof at 5:52 p.m., even directly by the Secret Service, other media report that he was climbing on the roof where he shot from at 6:09 p.m. (which makes a lot more sense). It seems possible that Crooks was on another roof first and then climbed from roof to roof. Either way - has the path he took from the ground to his shooting location been somewhat reconstructed by now? I didn't find anything in the news as available online. --KnightMove (talk) 10:30, 25 July 2024 (UTC)[reply]

Indeed, ABC News is the main source for this timeline, which is most certainly wrong. But it motivates my question. --KnightMove (talk) 11:54, 25 July 2024 (UTC)[reply]
Out of curiosity, why do you think the abc timeline is wrong? Blueboar (talk) 12:30, 25 July 2024 (UTC)[reply]
It seems entirely plausible that he climbed on to the roof (perhaps unencumbered by his rifle, but with his rangefinder), climbed down again (to retrieve his [hidden?] rifle and set its range), and then up again to carry out the attack. {The poster formerly known as 87.81.230.195} 94.2.67.235 (talk) 12:48, 25 July 2024 (UTC)[reply]
(edit conflict)
  1. Crooks actually climbed on the roof ~6:09 p.m., as per many witnesses and reliable sources.
  2. Crooks being on the roof since 5:52 in sight of the Secret Service; the Secret Service not doing anything against him, but leading Trump to the podium; AND Crooks then not firing, although he had so much time to prepare and some 7 min opportunity to shoot at Trump unimpaired - that's total nonsense (and fodder for conspiracy theories).
  3. There is even evidence to deconstruct how that error happened. This BBC article somewhat supports the error, although their own facts in the article make clear what happened:
"Later, around 17:45 local time, Crooks was spotted again, this time by a counter sniper officer around the Agr International building - the one the gunman later scaled up to aim at Trump."
"By 17:52 - 19 minutes before the shooting - the Secret Service was made aware that Crooks was spotted with a rangefinder, and disseminated that information to other officers on site, CBS reported."
So another (!) counter-sniper - not the Secret Service counter-sniper teams who would later aim at Crooks - spotted Crooks around (!) the roof, and at 17:52 the Secret Service was informed about this.
And some media merge this into the wrong claim "Secret Service snipers spotted Crooks on the roof at 17:52". --KnightMove (talk) 12:49, 25 July 2024 (UTC)[reply]
It is my understanding that two different organisations were involved in the maintenance of security, the Secret Service (federal, ie your counter snipers) and a local (?) organisation, be that state or entirely private (the person who initially confronted the assassin, was threatened with the firearm and "dropped" off the roof). It may be possible that the communication between these two entities was grossly deficient. Of course, this is pure speculation in the absence of reliable official information. --2001:871:6A:1B71:B0F9:2533:F61:54B2 (talk) 17:46, 25 July 2024 (UTC)[reply]
The FBI and the Pennsylvania State Police each also had a role in implementing the security measures around the rally.[29][30]  --Lambiam 19:11, 25 July 2024 (UTC)[reply]

J B Priestley quotation

edit

In 1970, Isacc Asimov wrote two short stories, "2430 A.D." and "The Greatest Asset", inspired by the following quotation:

Between midnight and dawn, when sleep will not come and all the old wounds begin to ache, I often have a nightmare vision of a future world in which there are billions of people, all numbered and registered, with not a gleam of genius anywhere, not an original mind, a rich personality, on the whole packed globe.

The above Wikipedia articles on the short stories, and many other on-line sources, attribute this to J B Priestley. However, as far as I can tell, it doesn't appear on any standard list of Priestley quotations (including WikiQuote), but only in connection with the Asimov stories. Did Priestley actually write this? If so, where? If not, did someone else write it? It's unlikely that Asimov made it up himself, but not impossible. 194.73.48.66 (talk) 17:42, 25 July 2024 (UTC)[reply]

It's from a 1957 book called "Thoughts in the Wilderness", which you can view (with restrictions) on archive.org - the quote is on page 127. AndrewWTaylor (talk) 19:23, 25 July 2024 (UTC)[reply]
J. B. Priestley (1957). Thoughts in the Wilderness. London. p. 127.  --Lambiam 19:24, 25 July 2024 (UTC)[reply]
Thanks! I've added the title of the book to the Asimov articles. 194.73.48.75 (talk) 20:28, 25 July 2024 (UTC)[reply]

Bohnert's Ethics

edit

One question, does anyone know Bohnert's system of ethics and deontic logic and can explain it or link to an explanatory text? 2A02:8071:60A0:92E0:9B9B:C02B:F5CC:22D1 (talk) 21:54, 25 July 2024 (UTC)[reply]

Who is Bohnert? ←Baseball Bugs What's up, Doc? carrots03:02, 26 July 2024 (UTC)[reply]
Herbert G. Bohnert (1918-1984), philosopher, professor at Michigan State University. I cannot help with OP's question. --Wrongfilter (talk) 06:15, 26 July 2024 (UTC)[reply]
Sorry, searching in websites related to logical positivism et al brings no trace of Herbert Bohnert. There is a stack of H. Bohnert´s publications in Google Scholar which may be useful to your research. --2001:871:6A:1B71:C00:3397:39D7:68BE (talk) 17:14, 26 July 2024 (UTC)[reply]
It seems that mentions of Bohnert in connection to deontic logic refer to: Herbert Gaylord Bohnert, "The Semiotic Status of Commands". Philosophy of Science 12:4, 1945, pp. 302–315. It is available online (doi:10.1086/286873) but behind a paywall. He appears to attempt to give a translation of commands to propositional logic. Very roughly, let   stand for "Sam eats his veggies" and   for "Sam is punished". Then the command "Sam, eat your veggies!", denoted formally as   is given the meaning   "[Sam eats his veggies] or [Sam is punished]". Or, as Sam's mother might phrase it, "Sam, eat your veggies, or else!".  --Lambiam 22:14, 26 July 2024 (UTC)[reply]

July 26

edit

Allah (peace be upon him)

edit

Would it be considered unusual for someone to say "peace be upon him" after mentioning God himself the same way that Muslims say it after mentioning any Islamic figure? – MrPersonHumanGuy (talk) 19:51, 26 July 2024 (UTC)[reply]

Based on Peace be upon him (Islam), I would think not. ←Baseball Bugs What's up, Doc? carrots20:08, 26 July 2024 (UTC)[reply]
Yes. The term "peace" in this phrase means "the peace of God". It would be just as strange as to say "May God have mercy upon Him", where "Him" refers to God Himself.  --Lambiam 22:20, 26 July 2024 (UTC)[reply]
Kind of like saying, "God bless God." ←Baseball Bugs What's up, Doc? carrots02:58, 27 July 2024 (UTC)[reply]
John 12:28? --Trovatore (talk) 20:28, 27 July 2024 (UTC)[reply]
Right. He will glorify His name, and He will be a bust (be a bust, be a bust) in the Hall of Fame. ←Baseball Bugs What's up, Doc? carrots03:21, 28 July 2024 (UTC)[reply]
@Baseball Bugs: Although I found the Hall of Fame part to be funny, your repetition of "be a bust" in parentheses makes me think you're trying to make a reference to song lyrics. If that's what it's supposed to be, then I'm afraid I don't get what you're trying to reference here. – MrPersonHumanGuy (talk) 22:58, 28 July 2024 (UTC)[reply]
This:[31]Baseball Bugs What's up, Doc? carrots03:02, 29 July 2024 (UTC)[reply]
Well, we (species Homo Sapiens) seem to have created hundreds, or even thousands, of Gods in our image (and to our liking). Blessing your multiple fellow gods may be totally civilised if a random cluster of divinities meet in the supermarket. Quite possible, that they only smite, drown, immolate, holocaust or whatever registered believers of their own party. We seem to have no article to the treaties between Yahveh, Allah or the Flying Spaghetti Monster on negligable genocide. --2001:871:6A:1B71:6D4B:BB6E:1759:BC95 (talk) 14:44, 27 July 2024 (UTC)[reply]
(edit conflict) From my last visit to a church I remember this benediction pronounced at the end of the service:

The peace of God, which passeth all understanding, and the blessing of God almighty, the Father, the Son, and the fellowship of the Holy Spirit be upon you and remain with you always.

These may not be the exact words - at school the Head said this daily at the end of assembly, but one day forgot the words and had to improvise. The Roman Catholic equivalent was the simple Ite, missa est until the Latin Mass was recently banned, much to the regret of the faithful, who consider the vernacular Mass inferior. 2A00:23D0:E1C:1301:6825:BE00:2408:BF0B (talk) 14:49, 27 July 2024 (UTC)[reply]

In the Abrahamic religions (including Islam), there is only the one God. Whether various Hindu gods would try to bless each other is beyond the scope of the OP's question. ←Baseball Bugs What's up, Doc? carrots17:45, 27 July 2024 (UTC)[reply]
Having done some research, I found that Morning Prayer, Evening Prayer, the Litany, the burial service, and some forms of prayer for use at sea, including "after victory or deliverance from an enemy" end with 2 Corinthians xiii.14:

The grace of our Lord Jesus Christ, and the love of God, and the fellowship of the Holy Ghost, be with us all evermore.

The blessing at the end of the Communion service, the Ordination service, and "a form of prayer for the twentieth of June, being the day on which Her Majesty began her happy reign" is as follows:

The peace of God, which passeth all understanding, keep your hearts and minds in the knowledge and love of God, and of his Son Jesus Christ our Lord: and the blessing of God Almighty, the Father, the Son, and the Holy Ghost, be amongst you, and remain with you, always.

If some people are being made deacons and some priests it is only said once. Note that the Archdeacon of Canterbury, like all other Archdeacons, is a priest or "clerk in holy orders" [32] (at 21:08) (note the reference at the beginning of the service to the Access to Justice Institution). The Confirmation service ends with these words:

The Blessing of God Almighty, the Father, the Son, and the Holy Ghost, be upon you, and remain with you for ever.

Before the blessing in the Communion service the Gloria in excelsis deo is said: in Latin (as sung at Catholic Mass) it begins:

Gloria in excelsis Deo; in terra pax hominibus

The Commination ("the denouncing of God's anger and judgements against sinners") ends with these words:

The Lord bless us, and keep us: the Lord lift up the light of his countenance upon us, and give us peace, now and for evermore.

Turning now to the Protestant Episcopal Church, a book published by the "Aux. N.Y. Bible and Common Prayer Book Soc.y A.D. 1816" with the motto "Whosoever will, let him take the water of life freely" and certified by Bishop Benjamin T. Onderdonk (who I see was the subject of misconduct allegations, like someone else I could mention) at New-York, June 1, 1832 says in the preface:

Rulers may have grace, wisdom, and understanding to execute justice, and to maintain truth;" Charles please note[1] 'and that the People "may lead quiet and peaceable lives, in all godliness and honesty."'

The burial service includes Psalm 3913:

Hear my prayer, O Lord; and with thine ears consider my calling: hold not thy peace at my tears.

Thanksgiving was "the first Thursday in November".

The "form of consecration of a church or chapel, according to the order of the Protestant Episcopal Church in the United States of America, as established by the bishops, the clergy, and laity of said church, in general convention, in the month of September, A. D. 1799" ends as does the Communion.

"An office of institution of ministers into parishes or churches prescribed by the Protestant Episcopal Church in the United States of America; established in general convention of the bishops, the clergy, and laity, 1804; and set forth, with alterations, in general convention, 1808" finishes with this benediction:

the God of peace, who brought again from the dead our Lord Jesus Christ, the great Shepherd of the sheep, through the blood of the everlasting covenant, make you perfect in every good work to do his will, working in you that which is well pleasing in his sight, through Jesus Christ; to whom be glory for ever and ever.

In the Roman Catholic Mass, after the Consecration there is a prayer for the dead:

Ipsis, Domine, et omnibus in Christo quiescentibus, locum refrigerii, lucis et pacis, ut indulgeas, deprecamur. Per eundem Christum, Dominum nostrum.

A little later, before the congregation receives the bread,

Pax Domini sit semper vobiscum.

In the Discalced Carmelite proper offices: a supplement to the Divine Office the liturgy of the hours according to the Roman rite as renewed by decree of the second Vatican council and promulgated by the authority of Pope Paul VI (Carmelite Priory, Boars Hill, Oxford, 1990) the Benedictus at Morning Prayer ends:

He will give light to those in darkness, those who dwell in the shadow of death, and guide us into the way of peace.

Psalm 119 (120) is prescribed at certain hours on a solemnity:

V2: Deliver my soul, O Lord, from lying lips: and from a deceitful tongue.

V3: [a little clairvoyance] What reward shall be given or done unto thee, thou false tongue: even mighty and sharp arrows, with hot burning coals

V5: My soul hath long dwelt among them: that are enemies unto peace.

V6: I labour for peace, but when I speak unto them thereof: they make them ready to battle.

Psalm 694 wraps it:

They that hate me without a cause are more than the hairs of my head: they that are mine enemies, and would destroy me guiltless, are mighty.

Have we moved on from 1540 when, according to "On this day", "King Henry VIII of England had his chief minister Thomas Cromwell executed for treason and heresy"?

In The hours of prayer from Lauds to Compline inclusive compiled from the Sarum Breviary and other rites (third edition, London, 1928) Sunday and Monday at Lauds closes with a prayer "for the peace of the Church", the same as the one which concludes Compline, thus the last prayer of the last service of the day:

O Lord, we beseech thee mercifully to hear the prayers of thy Church, and grant that we, being delivered from all adversities, may serve thee with a quiet mind, and enjoy the peace all the days of our life; through Jesus Christ our Lord, who liveth and reigneth, with thee and the Holy Ghost, ever one God, world without end.

On Tuesday to Saturday, Lauds concludes with the "ferial [weekday] petitions", which include the following:

V Peace be within thy walls
R And plenteousness within thy palaces

The same (Psalm 1227) is said on Tuesday at Vespers.

Terce on Sunday ends with the prayer "May the souls of the faithful, through the mercy of God, rest in peace."

But in most (though not all) varieties of Christianity, 'God' is said to comprise a trinity of three persons, including also 'Christ' and 'The Holy Ghost', so persumably Christ could bless The Holy Ghost (etc.). {The poster formerly known as 87.81.230.195} 94.2.67.235 (talk) 20:26, 28 July 2024 (UTC)[reply]
That would still amount to God blessing God. ←Baseball Bugs What's up, Doc? carrots21:53, 28 July 2024 (UTC)[reply]
Yes, that's the point: the theological contrivance would make that seeming absurdity possible, if it were factual. {The poster formerly known as 87.81.230.195} 94.2.67.235 (talk) 04:09, 29 July 2024 (UTC)[reply]
The Agnus Dei features in both the Mass and the Communion service. In Latin it concludes:

Agnus Dei, qui tollis peccata mundi, dona nobis pacem.

It is sung after the Prayer of Consecration.

In the Mass approved by C Eykens, Vicar General, at Antwerp on 24 January 1966, if the celebrant is a bishop he says "Peace be with you" after the Gloria in Excelsis. After the Prayer of Consecration he blesses himself:

omni benedictione coelesti et gratia repleamur. Per eundem Christum Dominum nostrum.

He then prays for the dead:

Memento etiam, Domine, famulorum famularumque tuarum___et___, qui nos praecesserunt cum signo fidei, et dormiunt in somno pacis.

Then follows the prayer quoted earlier, beginning Ipsis, Domine. In the Common of Our Lady, Psalm 137 is said, including the line Qui posuit fines tuos pacem (note the numbering of the Psalms differs). The Litany of the Most Holy Name of Jesus includes the line "Jesus, God of peace". In the evening prayers, the "Prayer for a happy death" is

O God, who has doomed all men to die, but hast concealed from all the hour of their death, grant that I may pass my days in the practice of holiness and justice, and that I may deserve to quit this world in the peace of a good conscience and in the embrace of Thy love. Through the same Christ Our Lord.

The Prayer for the Dead, (De profundis, Psalm 129 (130)) is followed by the words "May they rest in peace." Under "Various Devotions" the Litany of the Sacred Heart includes the line "Heart of Jesus, our peace and reconciliation." The Way of the Cross includes the line "My loving Jesus, it was not Pilate; no, it was my sins that condemned Thee to die."

References

  1. ^ Health Secretary Wes Streeting, speaking to Sky News and reported in the Evening Standard on Friday, spells it out:

    We believe very strongly in the rule of law, that's not just domestically but internationally, and the separation of powers between judges and politicians.

    "Everyone has got to be held accountable under the law..."

July 27

edit

What is "official Islam"? ~ Encyclopaedia of Islam

edit

Following quote from Encyclopaedia of Islam used by @Louis P. Boog at ongoing Talk:Jinn#RfC: Proposed additions of text 1 talks of wording "official Islam". The source Encyclopaedia of Islam entry by D.B. MacDonald and H. Massé does not seem to include specific definition of wording "official Islam".

  • Requesting help in ascertaining, What is "official Islam"? to the authors D.B. MacDonald and H. Massé (based on either their individual academic literature or may be they imported term from Bibliography they used or general academic literature)
This question is formed with 'what' instead of 'which' due to it's academic aspect.
  • Quote from Title: Ḏj̲inn, Encyclopaedia of Islam:

"..II. In official Islam the existence of the d̲j̲inn was completely accepted, as it is to This day, and the full consequences implied by their existence were worked out. Their legal status in all respects was discussed and fixed, and the possible relations between them and mankind, especially in questions of marriage and property, were examined. .."

~ D.B. MacDonald, H. Massé. Title: Ḏj̲inn. Encyclopaedia of Islam New Edition Online (EI-2 English) DOI: Source Editors: P.J. Bearman First-online: 24 Apr 2012 ISSN: 1573-3912 Publisher: Brill. Wikipedia Library link provided at WP:REREQ

Bookku (talk) 10:21, 27 July 2024 (UTC)[reply]

Possibly written Islam as defined by trained and educated Ulama, as opposed to more folklorish or popular concepts? There have been some cases of folk Islam elaborations gaining acceptance among scholars of Islamic law... AnonMoos (talk) 11:44, 27 July 2024 (UTC)[reply]
This article:
Németh, Kinga (2024). "The Jinn – The Culprit of the Arabic World". Különleges Bánásmód, 10, Special issue, 107-122. doi:10.18458/KB.2024.SI.107
contrasts "official (Sunni) Islam" with "vernacular Islam", which incorporates beliefs based on vernacular legends. While the term "official Islam" is not explicitly defined in the article, it is connected in the text to the points of view of "Islamic institutions", "scholars of Islam" and "schools of Islam".  --Lambiam 13:03, 27 July 2024 (UTC)[reply]
It means "standard/mainstream/mainline" Islam (coming in Shia and Sunni flavours). As supported by Islamic scholarship, or most of it. Arguably not an appropriate term, especially for the Sunni, any more than "official Protestantism". Johnbod (talk) 14:28, 27 July 2024 (UTC)[reply]

July 28

edit

In which countries, outside the United States, it is celebrated? CometVolcano (talk) 15:28, 28 July 2024 (UTC)[reply]

The clue is in the title. It's national not international. Shantavira|feed me 15:49, 28 July 2024 (UTC)[reply]
It is also known as International Masturbation Day. --CometVolcano (talk) 15:50, 28 July 2024 (UTC)[reply]
and? Nanonic (talk) 19:53, 28 July 2024 (UTC)[reply]
I don't know if you can say this day is "celebrated". Masturbation itself is celebrated every day, year-round, worldwide. Very few people outside the US, and I suspect also in the US, will have heard of a day specifically designated as Masturbation Day.  --Lambiam 12:26, 29 July 2024 (UTC)[reply]
Source? The folklore is that if you do it you will go blind. 2A02:C7B:107:4100:D868:90B:63A8:DAA6 (talk) 10:28, 30 July 2024 (UTC)[reply]
It would only make sense if it happened just one day per year. As with fireworks on July 4th. ←Baseball Bugs What's up, Doc? carrots11:45, 30 July 2024 (UTC)[reply]
No fireworks on New Year's Eve? 82.13.6.9 (talk) 14:51, 30 July 2024 (UTC)[reply]
That's also once a year. ←Baseball Bugs What's up, Doc? carrots17:19, 30 July 2024 (UTC)[reply]

July 29

edit

"Great Thoughts" anthology

edit

Does anyone know anything about an anthology of poetry, titled "Great Thoughts", that was published before 1890 and probably after 1840? It's mentioned multiple times in a book I'm working on in Wikisource, with notices like this one that say some of the poems in this collection already appeared in "Great Thoughts". The book these mentions appear in is by a single author, a Alice E. Argent, but I don't know her birth date, so that can't narrow the search. Thanks, — Alien333 (what I did & why I did it wrong) 14:37, 29 July 2024 (UTC)[reply]

Possibly the series Great Thoughts from Master Minds, published by A. W. Hall, Hutton Street, London. DuncanHill (talk) 15:04, 29 July 2024 (UTC)[reply]
Got it, that's it, since the first line of that poem is in there. Thanks for the help! — Alien333 (what I did & why I did it wrong) 15:12, 29 July 2024 (UTC)[reply]
Glad to be of assistance, many mentions of Alice E. Argent in newspapers of the period (1870s - 90s), including in connexion with Great Thoughts. DuncanHill (talk) 15:18, 29 July 2024 (UTC)[reply]
Indeed, after digging a bit deeper, I was able to find her full name, birth/death dates, etc., and expand the Wikidata item. — Alien333 (what I did & why I did it wrong) 15:25, 29 July 2024 (UTC)[reply]

Dutch - pl. help confirm

edit

Just I helped a little (probably new) user in their now accepted AFC article about nineteenth century speed skater Anke Beenen.

  • Anke Beenen#Popularity says

    On 22 December 1879 her speed skating club "Thialf" paid attention celebrated her silver anniversary by building a large gate in front of the house of mayor Daniël de Blocq of Scheltinga on the Heideburen in Heerenveen.

1) The article's popularity section mentions one 'mayor Daniël de Blocq of Scheltinga'.
WP has an article Daniël de Blocq van Scheltinga of a twentieth century person. Obliviously they are supposed to be different persons being from different centuries but on safer side wish to get that confirmed.
2) I did not get why the club would create 'a large gate in front of the house of mayor'? Since I had already asked good number of questions to the new user I do not make new user that I am going after them.
3) '.. on the Heideburen in Heerenveen.' What is 'Heideburen' being referred to here?

Bookku (talk) 15:07, 29 July 2024 (UTC)[reply]

"The Heideburen" is a street. There is a Wikidata item Heideburen
"a large gate" sounds like a triumphal arch of some kind. Temporary ones would be erected to celebrate a victory, visiting royalty, etc. DuncanHill (talk) 15:23, 29 July 2024 (UTC)[reply]
the sentence seem referring to 'in front of the house of mayor', does not even say 'in front of the office of mayor' intrigues me; may be I am just over thinking Idk. Bookku (talk) 15:29, 29 July 2024 (UTC)[reply]
Perhaps a bad translation? DuncanHill (talk) 15:33, 29 July 2024 (UTC)[reply]
that can be possibility. Many thanks for inputs Bookku (talk) 15:50, 29 July 2024 (UTC)[reply]
(ec)And the Van Scheltinga family seem to have included many Daniël de Blocqs. DuncanHill (talk) 15:33, 29 July 2024 (UTC)[reply]
According to the source used in the article (Leeuwarder nieuwsblad, 30 januari 1937), it was indeed a temporary arch, made from blocks of ice, on top of which three figurines were placed, depicting Beenen, her skating partner Jouke Schaap and the 'ice god' Thialf in the middle. The source also clearly says it was in front of the mayor's house. - Lindert (talk) 16:30, 29 July 2024 (UTC)[reply]
Many thanks for the confirmation Bookku (talk) 16:50, 29 July 2024 (UTC)[reply]
The street, Heideburen, was then part of the municipality of Schoterland; you can see the municipal border on this map from 1926 (the black dashed line behind the houses along the canal; Heideburen is the red road along the canal). Dutch Wikipedia has an unsourced article with a list of mayors (w:nl:Lijst van burgemeesters van Schoterland), which gives the name of the mayor as Hans Willem de Blocq van Scheltinga. The newspaper article linked above only gives his name as mayor De Blocq van Scheltinga. Dutch "van" means "of" in English. Translating such prepositions in names of nobility isn't unheard of. The municipal hall or whatever you want to call it was in the next village, Oudeschoot. PiusImpavidus (talk) 18:22, 29 July 2024 (UTC)[reply]
A 17th-century Daniël de Blocq of Scheltinga was grietman of Schoterland (w:nl:Lijst van grietmannen van Schoterland). Someone got confused, it seems.  --Lambiam 22:36, 29 July 2024 (UTC)[reply]
@DuncanHill mentioned nl:Van Scheltinga family seem to list a 'Mr. Daniël de Blocq van Scheltinga (1835-1878), lid provinciale staten' before Jhr. mr. Hans Willem de Blocq van Scheltinga (1870-1933), gemeenteontvanger en wethouder van Rheden.
But there seem to be some confusion about name and post if we compare all three articles mentioned above. Of course in historical details such discrepancies can be routine too. Bookku (talk) 05:14, 30 July 2024 (UTC)[reply]
Two sources that confirm that Hans Willem was (at some time) the mayor of Schoterland: [33], [34] The Rheden Hans Willem, born in 1870, was nine years old at the time the ice arch was erected, which was too young to be appointed to a mayorship, so this is a different individual. The Schoterland mayor was Hans Willem jr.; the Rheden wethouder has the right age to have been Hans Willem III. It is a safe assumption that the author who named the mayor "Daniël" did so by mistake. In 1879, the town Heerenveen did not exist as a single municipality but was divided over two municipalities, Aengwirden en Schoterland, so the mayor in front of whose house the arch was erected cannot have been mayor of Heerenveen. I have replaced the last part of the sentence by:
a large gate in front of the house of the mayor of Schoterland, de Blocq van Scheltinga, on the Heideburen street in Heerenveen.
 --Lambiam 11:07, 30 July 2024 (UTC)[reply]
Many thanks to all of you for all the search and support. Bookku (talk) 12:54, 30 July 2024 (UTC)[reply]

USA Banking questions.

edit

So I do know of someone that owed the credit card section of a bank, $27,000, before the bank finally disconnected. Which brings me to several theoretical questions.

1. If someone owes the credit card of Bank A $27,000, do the other banks know about it? I heard that they do not know directly, but they can do through shared databases. But the database doesn't tell the bank an exact amount, just a range amount.

2. What if you owe Bank A $27,000, but you already have over $40,000 in Bank B. What can Bank A do in regards to that?

3. What if you owe the credit-card side of Bank A $27,000, but you do have $27,000 in the commercial-banking side of Bank A. Can Bank A seize that amount? And if so, do they send you a letter notifying you of that, in which you can just withdraw that money out, or can they seize it from you without notifying you, or do they have to take you to court for that, in which you would just withdraw that money out?

4. What if you owe Bank A $27,000, but you have more than that in Bank B. But then, Bank A and Bank B got merged into the same bank? Heh. 66.99.15.162 (talk) 19:41, 29 July 2024 (UTC).[reply]

First, credit cards are rarely issued by specific banks… they are issued by specialized companies such as Visa, or Mastercard. You will continue to owe the credit card company for any debt on the card, even if you switch banks.
And yes, if you default on a significantly large debt, the credit card company can take legal action and place a lien on your assets, wherever those assets might be located. Blueboar (talk) 20:10, 29 July 2024 (UTC)[reply]
In the US there are several commercial consumer reporting agencies that keep track of people's so-called "credit scores", which are supposed to represents the creditworthiness of individuals; see Credit score § United States. Lenders can access your credit score. The situation under question 2 is, from a legal point of view, not different from the general situation that someone refuses to pay a debt while having valuable assets. What Bank A can do in the situation under question 3 depends on the contract you have with the bank (the rules you agreed to when opening the account), but I cannot imagine it includes transferring money without notification. If you should know, or have reason to expect, that a debtor may take you to court, then, I think, it is a crime to squirrel your assets away in order to avoid the anticipated seizure.  --Lambiam 21:36, 29 July 2024 (UTC)[reply]

Jurisdiction matters, as does whether the debt is being serviced. 1. is hugely dependent on the jurisdiction. In some places, sharing personal data is a violation of the law; in others, it is just good practice. 2. needs some explanation as to why Bank B needs to know about the debt to Bank A. Imagine the customer is Elon Musk, and ask yourself why Bank B needs to know this information. 3. See 1. Above. In general, the two “sides” of a bank – personal and corporate – are actually two different bank accounts. If the corporate side is not linked to (e.g., collateral for) the corporate side, in many jurisdictions the personal side can only be accessed via a court-ordered bankruptcy proceeding. 4. There is no reason for a bank to merge two bank accounts simply because the corporate entities (bank owners) merged. More, the debt is not said to be in arrears or delinquent, so there would be no reason to notice that it is (or isn’t) related to an unrelated deposit. DOR (ex-HK) (talk) 20:06, 29 July 2024 (UTC)[reply]

To Dor (HK). When you say jurisdiction matters, I guess you already stated that this is not a federal court issue and is state. So 1 state will allow Bank A to seize money from Bank B, and another state would not? Wonder if that's correlated with red vs. blue states. Could the same hold true for the credit card and savings coming from the same bank? Also, would the state law be for the state in which you opened the account, or the state the headquarters city is in?
To Blueboar, JP Morgan Chase and Capital 1 both having a commercial banking side, and a credit card side. I would imagine you owing the credit card side of the bank, had less to do with Visa/Mastercard. Visa/Mc makes money from swiping...
And to Lambiam and possibly others. Before my dad divorced his wife #6, she opened up a cc in their name and started using it behind his back, until cc company sent him a bill, hitting $27,000. So he hired a lawyer where the lawyer sent the cc company a picture of his proof of divorce so he's no longer eligible for the bills after their divorce date, which is 99% if not 100%. So now cc company only goes to her for the $27,000. She is Korean but from South America, where she went back to. This was in 2016. Earlier this year, I spoke with the attorney that sent the cc company the divorce, I asked him if the cc company knew she was back in the U.S., could she be arrested? Lawyer said no. Which means this is still a civil issue. So the idea of withdrawing all the money from your bank when the bank notifies you, is probably still not criminal. Also, there is no such thing as civil lawsuits with individuals. Why? Because when the process server rings your doorbell, you say "Sorry you got the wrong address, don't know anyone with that name that lives here." Or better yet you don't answer the doorbell, and certainly never sign anything. So, owing a cc company $27,000 is still not an arrestable offense.
Thanks for your responses guys. 66.99.15.162 (talk) 14:58, 30 July 2024 (UTC).[reply]
Now, regarding if you owe Bank A $27,000, does Bank B know about it? I will put this out, because someone will interpret this as asking for legal advice, but the story is over. Before my dad died, the wife #6 transferred $42,000 from his Bank C, to Bank D. So, why do I go to, Bank C or Bank D. Turns out to be a mix of both. And she put my dad's name in her Bank D so both their names are in the account, to make it look like he was transferring his own money, from his own bank to his other bank. So, that became legal. So here I am contacting Bank D about it. I didn't ask for a refund. I asked to mark her as a fraudster person. The bank responded that they sent my story to their fraud-team. Then I told the Bank D, that she owes the credit-card side of Bank B, $27,000 in 2016. You know what Bank D responded? Bank D responded "that has nothing to do with us." But I reiterated that obviously the credit card side of Bank A considers her a fraudster person, to help back up my story that she is a fraudster, and so, Bank D replied we sent this story to the fraud team again... So, Bank A and D are 5/3rd Bank and Byline Bank. If she owes 5/3rd Bank $27,000, do the other banks know about it? Does 5/3rd Bank not try to tell other banks about it? Well, hence comes privacy laws. The fact that she has a Byline Bank in 2024 makes me think the account opened long before 2016... And ChatGPT says Byline Bank will never e-mail or phone call 5/3rd Bank to confirm if she owes $27,000, that that's not how it works, that the only info they can get is through shared databases. 66.99.15.162 (talk) 15:06, 30 July 2024 (UTC).[reply]
I'm not looking for advice on how to get back the $42,000, but am looking into how banks can know stories of how other people owe other banks money. I'm also looking for any famous cases where Bank B froze your account because they found out you owe Bank A a lot of money. 66.99.15.162 (talk) 15:39, 30 July 2024 (UTC).[reply]

July 30

edit

Digestion before the Fall

edit

Ronald Knox's The Creed in Slow Motion, 1949 ed., is a collection of talks given during the Second World War. On page 68, he says in passing:

I even read a book the other day— not by a Catholic, but by a very intelligent man— which suggested that, before the Fall, man’s will directed his digestion. Think how nice it would be if you could digest your meals at will, like brushing your teeth.

He doesn't mention what book it was. Does anyone have any ideas? Marnanel (talk) 12:58, 30 July 2024 (UTC)[reply]

I believe it would be difficult to be certain. I have to assume that this was between 1920 and 1930. At the end of the 1800s, Nietzsche was popular, which made way for many very similar philosophical works. Nietzshe had a common complaint throughout his work that mankind previously had control over the body, but over time had devolved into a purely reactionary vessel. He specifically mentions digestion in some of his works. I remember one passage that I studied in college in which he claimed that the flood of information fron a daily newspaper muddies the mind and trains people to quickly react, not think, as the information is digested and the body continues this. It quickly digests whatever is put into the mouth withough considering if it should be digested and what the nutrients might be used for. Because he brought this up in multiple books and the same thoughts were expressed by many others at the time, it would be difficult to identify exactly which book Knox read. You could narrow it down to only non-Catholics, but I doubt many people who published Nietzsche-like books were Catholic. 12.116.29.106 (talk) 14:27, 30 July 2024 (UTC)[reply]

July 31

edit

Book publication dates

edit

For a project I'm working on, I need to check the exact publication dates for a bunch of novels etc. Google is my friend, yet often it gives me wildly varying dates for the same book. A case in point: the publication date for Steve Toltz's A Fraction of the Whole is either:

  • 12 Feb 2008, 11 Apr 2008, 1 May 2008, before Jun 2008, 23 Sep 2008, 1 Oct 2008, 6 Oct 2008, 14 May 2009, or possibly others, depending on which hit one chooses to believe. [35]

I understand that editions in different languages, and hardbacks vs. paperbacks can come out at different times. Also, books will be released in foreign countries at varying times, because different publishers will be involved. But in general, these do not seem to explain the variations in publication dates that I'm seeing. What would explain them? -- Jack of Oz [pleasantries] 21:31, 30 July 2024 (UTC)[reply]

A Google search for "A Fraction of the Whole" "first published" gives a large number of mentions of specifically February 12, 2008, apparently by Spiegel & Grau. Amazon states explicitly that this is the first edition.  --Lambiam 23:16, 30 July 2024 (UTC)[reply]
As a book collector and researcher, I would not place much reliance on dates given on Amazon; they often refer to a 'first edition' in a particular country, and are often entered well before publication, which may then be delayed or advanced.
I note that Spiegel & Grau were/are a New York publisher, whereas the book's article gives its (presumably first edition) publisher as Hamish Hamilton, Australia, as one might expect for an Australian author's first novel. The online Science Fiction Encyclopedia (Toltz's work verges on the fantastic/near future) also gives the Australian edition as the first, though (as always) it records only the year, and by policy favours the author's 'home nation' when editions have appeared simultaneously or nearly so in more than one country. {The poster formerly known as 87.81.230.195} 94.2.67.235 (talk) 01:51, 31 July 2024 (UTC)[reply]

Lafcadio Hearn story - One man in a boat

edit

The Algernon Blackwood short story 'The Man who was Milligan' refers to a Lafcadio Hearn story "about a picture of a man in a boat. An observer, watching the picture, had seen the man move. The man actually began to row. Finally, the man rowed right out of the picture and into the place - a temple - where the observer stood." What is the Lafcadio Hearn story? Thank you, DuncanHill (talk) 01:37, 31 July 2024 (UTC)[reply]

Doesn't match the "temple" part, but maybe "The Story of Kwashin Koji"—see the last paragraph of the story here. Deor (talk) 14:41, 31 July 2024 (UTC)[reply]
Oh that must be it, especially with the disappearing never to be seen again. Thank you, DuncanHill (talk) 17:25, 31 July 2024 (UTC)[reply]

Trouble finding reliable sources

edit

I'm trying to add information to Beethoven's legacy section, however I'm having trouble finding reliable sources which give good information. When I use google I run into articles which aren't reliable and when I use google scholar all of the sources are locked behind a paywall. And, yes, I tried looking into the sources section of the wikipedia guides. What can I do? Wikieditor662 (talk) 08:51, 31 July 2024 (UTC)Wikieditor662 (talk) 08:51, 31 July 2024 (UTC)[reply]

@Wikieditor662 If you cannot find reliable sources, there is nothing to do, because Wikipedia articles should be solely based on reliable sources. You might have better luck if you mention this issue on the article talk page, which will be watched by editors more familiar with the subject. Shantavira|feed me 09:10, 31 July 2024 (UTC)[reply]
I'm sure there are reliable sources on the influence of beethoven, I just don't know how to find them. But, sure, I'll ask on the article page... Wikieditor662 (talk) 09:11, 31 July 2024 (UTC)[reply]
Sources don't have to be online (links are nice to have, but not essential). A Public Library of any size (if you can get to one) should have (or could obtain for you via Interlibrary loan) biographies of Beethoven which will likely discuss this topic at length. {The poster formerly known as 87.81.230.195} 94.2.67.235 (talk) 12:27, 31 July 2024 (UTC)[reply]
Thanks, I'll see if I can do that. Also, are there any other ways to get online sources, as they're much easier to use? Wikieditor662 (talk) 12:34, 31 July 2024 (UTC)[reply]
Yes. Have a look at WP:RX. ColinFine (talk) 18:11, 31 July 2024 (UTC)[reply]
A Google Books search often affords no preview or one that is too limited, but you will not run into a paywall. Another searchable repository of books is eBooks and Texts at the Internet Archive. Just searching for Beethoven+legacy turns up possibly useful sources. A source that offers information of interest but is by itself not reliable can serve as a source of inspiration for keywords for a more precisely targeted search. See also Help:Find sources and WP:Advanced source searching.  --Lambiam 21:57, 31 July 2024 (UTC)[reply]

August 1

edit

Language

edit

July 17

edit

"Two English"

edit

An American man and an American woman are "two Americans". This is gender-neutral and unambiguous. An Englishman and an Englishwoman are "two English". However, this is not that well understandable as it is not your first connotation when you hear "two English". What is talked about here? A translation from Spanish "to English"? Is the girlfriend of a Scottish boy "too English" for his grandmother? Is there a better, unmistakable way to address two mixed-gender people from England? --KnightMove (talk) 10:47, 17 July 2024 (UTC)[reply]

Two English people? HiLo48 (talk) 11:23, 17 July 2024 (UTC)[reply]
I think HiLo48 has the best answer. "Two Englanders" is understandable but will leave you sounding like a comedic stereotype. "Two Britons" is correct but less precise. AlmostReadytoFly (talk) 13:00, 17 July 2024 (UTC)[reply]
When not in England, the term "Brits" (albeit in this particular case, as you say, imprecise) is something I've heard commonly used and used myself. Mikenorton (talk) 16:46, 17 July 2024 (UTC)[reply]
I'm a little curious about this. When I hear someone described as a Brit I generally assume they're English; I don't really think of the Scots or the Welsh and certainly not the Northern Irish. Is that the common understanding? --Trovatore (talk) 20:30, 17 July 2024 (UTC)[reply]
I worked in Norway as an ex-pat for over five years and the Brits included Scots and Welsh as far as I can remember, but that's just a personal recollection, no citations available, sorry. Mikenorton (talk) 22:15, 17 July 2024 (UTC)[reply]
In absolute terms, most Brit(on)s are English (84% of the UK population is in England).
Relatively, Northern Ireland is the only constituent country whose residents are more likely to identify as solely British (41%) (or solely Irish at 25%) than solely of their constituent country (21%). AlmostReadytoFly (talk) 09:45, 18 July 2024 (UTC)[reply]
  • I am not aware of a generic & specific term for two people of mixed gender. The word "couple" may be useful, but it does not define two random people. Two Americans, two Britons, two Spaniards (or whatever) can be any combination of genders / sexes. And two Viennese könnten ja ein Paar Würschtel sein. --Cookatoo.ergo.ZooM (talk) 13:15, 17 July 2024 (UTC)[reply]
    Are you drawing a distinction between thigs like " there's a couple of people standing in that corner" and "there's a couple standing in that corner"? Because I would say the first about any 2 people but the second only if they were romantically linked. Except I would also say "there's a couple in that corner" as a response to the question "how many people are in this room?" Again, without regard to their sexes, genders, or relationship statuses.--User:Khajidha (talk) (contributions) 20:39, 17 July 2024 (UTC)[reply]
Occasionally, people use just "two English" as a noun phrase.[36][37][38]  --Lambiam 14:55, 17 July 2024 (UTC)[reply]
I think that would normally be considered non-standard, though. There are some (most?) demonyms which are both adjective and noun (e.g. American/an American, German/a German), some which have different words for each (e.g. Spanish/a Spaniard, Danish/a Dane) and some which are (generally) only used as adjectives (e.g. French, Irish, Chinese). Generally those in the third category need "person" added to make a noun phrase (or "-man"/"-woman" in some cases), and that's where I'd put "English". There seems to be a slight shift towards treating the third category the same as the first category and using the adjective as a noun (e.g. I've seen "a Chinese" instead of "a Chinese person"), which is presumably why there are a few instances of "two English" being used, but it still sounds odd to my (British English) ear. (Admittedly, it does sound a little less odd in the plural. "I saw two English walking down the street" would be odd, but "I saw an English walking down the street" would be downright bizarre.) Proteus (Talk) 15:27, 17 July 2024 (UTC)[reply]
Indeed. Possibly the closest we can come in standard-speak is "The English are a curious race", or "You English are despicable". Not even "some English" works for me.-- Jack of Oz [pleasantries] 15:46, 17 July 2024 (UTC)[reply]
I couldn't find the usage in the first and third links, and the second was definitely with nonstandard grammar.--User:Khajidha (talk) (contributions) 20:31, 17 July 2024 (UTC)[reply]
Of course, here in Australia we also have the useful word "pom", sometimes expressed more fully as "pommy bastard", especially when an Ashes series is underway. Although the word almost always refers to male English people.HiLo48 (talk) 23:47, 17 July 2024 (UTC)[reply]
On my only visit to Australia many decades ago, I was proud to be referred to as a "to and from". Alansplodge (talk) 10:48, 18 July 2024 (UTC)[reply]
Is that rhyming slang? 惑乱 Wakuran (talk) 17:19, 19 July 2024 (UTC)[reply]
Sounds like it, but I've never heard that one. There are expressions used throughout our wide brown land, but then there are some others that have currency only within much smaller communities. The Aussie way is to use such little-known expressions on unsuspecting strangers, whether they be from other parts of the country or overseas, causing the strangers befuddlement. They ask for a translation, and this immediately marks them as "not a local", and hence automatically suspect until such time as they can earn a degree of acceptability. That may take a few minutes, or a few generations, or anywhere in between. Xenophobia works on many levels, not just between nations. -- Jack of Oz [pleasantries] 22:19, 19 July 2024 (UTC)[reply]
"to-and-from n. [rhy. sl. = pom (see POMMIE n.)] (Aus.) a British immigrant to Australia." see Green, Jonathon. "to-and-from". Green’s Dictionary of Slang. DuncanHill (talk) 22:27, 19 July 2024 (UTC)[reply]
It doesn't have so be an immigrant. "Pom" works for visitors too. I believe our King is visiting later this year. He will be called a pom by plenty of Aussies when he's here. — Preceding unsigned comment added by HiLo48 (talkcontribs) 11:54, 20 July 2024 (UTC)[reply]
We Londoners are perfectly fluent in rhyming slang, so there was no befuddlement. Alansplodge (talk) 11:26, 20 July 2024 (UTC)[reply]
I should bleeding coco! Mikenorton (talk) 19:09, 20 July 2024 (UTC) [reply]
We have Terminology of the British Isles and Glossary of names for the British... AnonMoos (talk) 18:10, 20 July 2024 (UTC)[reply]

May someone tell me what language this video are in?

edit

--Trade (talk) 21:11, 17 July 2024 (UTC)[reply]

Based on the framed picture in the background, this is probably Daniela Mercury on the left and her wife Malu Verçosa Mercury on the right speaking Brazilian Portuguese. GalacticShoe (talk) 22:41, 17 July 2024 (UTC)[reply]
The subtitles appear to be in Russian and Arabic, though. Or possibly some other Slavic language in Cyrillic, and Persian. 惑乱 Wakuran (talk) 11:07, 18 July 2024 (UTC)[reply]
The first video has no sound. I listened to the second and can confirm the language is Portuguese. Since I initially found the accent difficult to follow it is undoubtably the Brazilian variety. 2A00:23D0:7C1:5201:8531:C651:42DE:A528 (talk) 11:23, 18 July 2024 (UTC)[reply]
It does indeed have sound. Maybe you'd need to update your browser or something. 惑乱 Wakuran (talk) 12:06, 18 July 2024 (UTC)[reply]
Not really interested in the subtitles. Only the audio Trade (talk) 12:14, 18 July 2024 (UTC)[reply]
Interesting. Didn't even realized the videos featured celebs Trade (talk) 12:16, 18 July 2024 (UTC)[reply]

Drone as a German word

edit

Hey friends, having another puzzle over a wördle.de answer. Yesterday the solution was "DRONE" which has left me baffled. Any help on what the German language means by this word please? All I get from de.wikipedia are proper names. Thanks! 70.67.193.176 (talk) 22:53, 17 July 2024 (UTC)[reply]

German wiktionary lists only an English word under that spelling (without "h"). AnonMoos (talk) 23:59, 17 July 2024 (UTC)[reply]
Exactly, that's why am confused.70.67.193.176 (talk) 02:23, 18 July 2024 (UTC)[reply]
Could the alternative spelling "Drone" have become common in German in recent years? Given how much has been written, discussed, etc. about these unmanned flying machines in the past decade, I wouldn't be surprised. It's now become the standard term in French. Xuxl (talk) 07:56, 18 July 2024 (UTC)[reply]
In my experience, it hasn't. "Drohne" and "drone" are not just different spellings, but also pronounced differently. I guess many German speakers may be more or less familiar with the English term too, but in my perception at least the German term is still the one commonly used, in both meanings of the word (bees and flying devices). Fut.Perf. 08:12, 18 July 2024 (UTC)[reply]
It doesn't appear as a German word on LEO.org either. AlmostReadytoFly (talk) 08:47, 18 July 2024 (UTC)[reply]
Short of a misspelling, the only think I can think of is the English word "drone" (as in "droning sound") that is sometimes used when talking about music. It doesn't seem sufficiently common to count as a loanword, though. In principle, it could also be the plural of a word *Dron, but that doesn't exist either. For a German (not English) word *Drone, the pronunciation would be the same as Drohne — the existing word Krone ("crown") is a perfect rhyme. --Wrongfilter (talk) 12:42, 18 July 2024 (UTC)[reply]
As a native speaker (as is Fut.Perf.), I have never seen the AngloSaxon word "drone" used as a loan word in colloquial German. A typo can be excluded, as Wördle is a constructed linguistic game.
Is it possible, that the solution is NOT "Drone", but a similar German noun / verb / whatever? The OP seems to be Canadian. --Cookatoo.ergo.ZooM (talk) 16:59, 18 July 2024 (UTC)[reply]
We're all native speakers here ;-) I entered "DRONE" into wördle today and it was accepted, which means that it is in the database. I haven't found out how the database was created and what is or is not in it (misspellings seem unlikely but not entirely impossible). Maybe the creator is a Sunn O))) fan and put Drone (Doom) in by hand... --Wrongfilter (talk) 18:32, 18 July 2024 (UTC)[reply]
The unmanned aerial vehicle is supposed to be spelled Drohne in German (like the male bee), but I suspect that the English spelling without the h is common in unedited writing. Even de:Unbemanntes Luftfahrzeug once uses the word Hubschrauberdrone spelled like that. —Mahāgaja · talk 10:16, 24 July 2024 (UTC)[reply]
Not any more. --Wrongfilter (talk) 10:40, 24 July 2024 (UTC)[reply]
Well, I'm typically using "raise" as my starting word in English Wordle. Interestingly I could also use it in wordle.at and I still have no idea what it is supposed to mean in German. I've switched to a more regular German word (Saite) in the meantime. -- 79.91.113.116 (talk) 15:02, 25 July 2024 (UTC)[reply]
Regular plural of Rais, it seems. [39] 惑乱 Wakuran (talk) 19:58, 25 July 2024 (UTC)[reply]
d:Q650316 (Drone music) reports that the article is not present at de.wp, but gives Drone as the German label. Folly Mox (talk) 12:03, 20 July 2024 (UTC)[reply]
As has been noted, there exists a German Wikipedia article for Drone Doom. It is a rather niche genre, though. 惑乱 Wakuran (talk) 14:35, 20 July 2024 (UTC)[reply]
Thank you for all the discussion. It's not the first time this database has been weird, so I guess that's what's going on here. "drone" was indeed the solution, it tells you :) 70.67.193.176 (talk) 01:33, 19 July 2024 (UTC)[reply]
It just might be the german spelling of the Drône (river), as we have no accent circonflexe in German. Lectonar (talk) 11:13, 19 July 2024 (UTC)[reply]
Are other French river names accepted, such as LOIRE and SAONE?  --Lambiam 00:37, 20 July 2024 (UTC)[reply]
LOIRE is accepted, SAONE is not. RHEIN is also accepted. GalacticShoe (talk) 01:11, 20 July 2024 (UTC)[reply]
Note that Drône is an archaic spelling; the current French orthography is Dronne. Since the Saône is a much more significant and better-known river than the Dronne, presence in the Wördle db as the Germanized spelling of archaic DRÔNE is IMO not a satisfactory explanation.  --Lambiam 09:34, 20 July 2024 (UTC)[reply]

July 21

edit

Ur-pun

edit

What is the world's earliest known pun? Any language, not just English. Clarityfiend (talk) 02:39, 21 July 2024 (UTC)[reply]

The Hebrew Bible has "explanations" as to why various people were given their names which are not always etymologically accurate; if such an explanation involves a similar-sounding word, I guess it could be a pun. The episode of Susanna and the Elders added to the Greek version of the book of Daniel has two specific puns on Greek tree names... AnonMoos (talk) 06:32, 21 July 2024 (UTC)[reply]
Earlier: Genesis 4 gives us Cain and Abel § Etymology - "their names may be a direct pun on the roles they take"; perhaps Genesis 2 counts for Adam § Connection to the earth. -- Verbarson  talkedits 19:24, 24 July 2024 (UTC)[reply]
Have you seen Pun#History_and_global_usage? Shantavira|feed me 08:37, 21 July 2024 (UTC)[reply]
I have seen a book titled something like Puns: the foundation of writing. Much of early writing uses homophones for concepts that cannot easily be pictured. —Tamfang (talk) 22:37, 22 July 2024 (UTC)[reply]
Arguably, Chinese characters still do, at least partially. 惑乱 Wakuran (talk) 14:54, 23 July 2024 (UTC)[reply]
Writing hard-to-draw words by substituting a drawing of easy-to-draw words is known as the "Rebus principle" (scan down in article Rebus)... AnonMoos (talk) 19:11, 26 July 2024 (UTC)[reply]

Is backwards English a language?

edit

If all English sentences had their word order perfectly reversed, could this still be a conceivably natural language? Or, would this new Hsilgne language have any unnatural features that would make it unique among other languages? 2600:8800:718D:8D00:D4AB:BD6F:4954:2F7 (talk) 04:35, 21 July 2024 (UTC)[reply]

For almost any order between several connected syntactical parts, such as a verb (V) with a subject (S) and an object (O), or a noun (N) with an adjective (A), there are languages whose grammar prescribes that order, so we have VSO languages, SOV languages, OSV languages, and so on. English is an SVO language, so English reverse would be an OVS language - a rare, but not non-existent type. Likewise, English is an AN language ("blank card"), but there are many NA languages, such as French ("carte blanche"). For more, see Linguistic typology, Word order and Head-directionality parameter. Some languages (e.g. Turkish) do many things in the reverse order of English; for exampe, "I don't know the man who stole your book" becomes "Book-your stealing man-the know-not-I". There is nothing unnatural about the word orders that reverse the word order of English. Therefore it is IMO a reasonable guess that kids growing up immersed in an environment where English reverse is spoken will pick it up just like they would pick up any natural language.  --Lambiam 07:07, 21 July 2024 (UTC)[reply]
OVS OSV isn't unnatural or forbidden, but it's typologically disfavored as the main default word order in a language -- it's the rarest of the six basic word order types by a significant margin... AnonMoos (talk) 07:28, 21 July 2024 (UTC)[reply]
OSV is even rarer, yet Warao kids have no problem learning it naturally, which is the issue.  --Lambiam 07:38, 21 July 2024 (UTC)[reply]
Sorry, I meant "OSV" (I had problems reversing just three letters!). I'm sure that OSV is viable as a language system, but it's very strongly typologically disfavored, as I meant to say. If you round to the nearest percentage, then the occurrence of both SVO and SOV among the world's languages is in the solid double digits, VSO is almost in the double digits, while the closest integer percentage to express the occurrence of OSV languages is 0%. AnonMoos (talk) 13:24, 21 July 2024 (UTC)[reply]
The deuce you say!  Card Zero  (talk) 22:12, 28 July 2024 (UTC)[reply]
A minor aspect that would not be natural: the variation between an and a for the indefinite article, governed in English by whether the onset of the following syllable is zero. "An untied band is not a united band", reversed, should become "Band united a not is band untied an"; however, such a variation, originally a phonological process, being governed by distant phonemes, is not seen in natural languages.  --Lambiam 07:29, 21 July 2024 (UTC)[reply]

Kilometre

edit

Do average English speakers in countries that have metricated almost everything (such as Australia and New Zealand) use word "kilometre" in expressions to indicate an unspecified distance, such as "kilometres away", "a few kilometres", "several kilometres", "thousands of kilometres", in daily lives? --40bus (talk) 19:13, 21 July 2024 (UTC)[reply]

I don't know if Canada qualifies, but the answer here is "mostly no". I mean, you could say it and be perfectly understood, but I'd bet more than half the people you'd meet would use "miles". Kilometers sounds formal and scientific, almost stuffy. Light travels 300,000 kilometers a second, but that town is twenty miles from here. I think younger folks tend to favour the metric more, so perhaps this is changing. Matt Deres (talk) 01:59, 22 July 2024 (UTC)[reply]
Even though there isn't much scientific base for the metre's exact length, anyway. 惑乱 Wakuran (talk) 15:08, 22 July 2024 (UTC)[reply]
Agree. In Canada you'd say about a family relocating to some far-away suburb: "they moved miles away from here", not "kilometres away" unless you want to specify a precise distance, which you would then include in the sentence. Xuxl (talk) 07:32, 22 July 2024 (UTC)[reply]
Yes, "miles away" carries a connotation of "a long way away", that "kilometres away" simply doesn't. Shantavira|feed me 08:57, 22 July 2024 (UTC)[reply]
How about "give him an inch and he'll take an ell"? DuncanHill (talk) 13:04, 22 July 2024 (UTC)[reply]
My international traveler yoga instructor sometimes fashions himself a bit of a linguistic wit, and insists on saying rather than "inch your fingers forward", "millimeter your fingers forward", which is at the very least kinda silly. I stopped saying anything about his dumb jokes a while ago - he's a good teacher and leader, but oy. --jpgordon𝄢𝄆𝄐𝄇 17:42, 27 July 2024 (UTC)[reply]
Is "several kilometres" ever used? Is it common to say "a several-kilometre-long traffic jam". In Finnish is common to say "useita kilometrejä". --40bus (talk) 15:10, 22 July 2024 (UTC)[reply]
Yes, traffic reporters on the radio would say "several kilometres". HiLo48 (talk) 00:07, 23 July 2024 (UTC)[reply]
What syllable do they stress? When I was growing up I learned /kɪlˈɑːmɪtər/ but now I mostly say /ˈkɪloʊˌmiːtər/ (with some of the weak vowels probably devolving into indistinct schwa-ishness in practice). I hear both in the wild but tend to think of the first-syllable stress as "more precise" or "more sciency". I'm willing to allow the possibility that this might be a slight affectation. --Trovatore (talk) 18:56, 27 July 2024 (UTC)[reply]
Sample of one, but I've always favoured stressing the second syllable (not the first and/or third), to avoid the word sounding like a device to meter kilos. I prefer to stress the antipenultimate syllable in names or words that derive (or might derive) from Greek. {The poster formerly known as 87.81.230.195} 94.2.67.235 (talk) 94.2.67.235 (talk) 20:20, 28 July 2024 (UTC)[reply]
Interesting — we have the same goal but the opposite execution. To me the second-syllable stress sounds like "speedometer", which measures speed, and "odometer", which measures, well, od I guess. --Trovatore (talk) 23:33, 30 July 2024 (UTC)[reply]

On a lighter note, see <https://discus.4specs.com/discus/messages/4254/Thanksgiving_day-art_buchwald-8902.pdf>. DOR (ex-HK) (talk) 20:11, 29 July 2024 (UTC)[reply]

July 22

edit

Philomena Cunk's English

edit

The kind of English accent Philomena Cunk uses is obviously not RP. What would you call it? 178.51.74.75 (talk) 01:45, 22 July 2024 (UTC)[reply]

Diane Morgan plays Philomena Cunk and she comes from Bolton, where they speak a type of Lancashire dialect. AlmostReadytoFly (talk) 08:43, 22 July 2024 (UTC)[reply]
Does she use her native accent when playing Philomena Cunk? I don't think I've ever heard her speak out of character. —Mahāgaja · talk 10:02, 24 July 2024 (UTC)[reply]
I'd only seen her as Cunk and in Motherland (where she also has a Bolton accent), but here she is on Late Night with Seth Meyers, explaining that she plays Cunk in her own accent (at about 1m 45s). AlmostReadytoFly (talk) 11:22, 24 July 2024 (UTC)[reply]

Leaving out "to be"

edit

I've noticed that many classic-format jokes, especially as found in joke books, do something odd with the tense: they have a tendency to remove the "to be" portion and I'm not sure what that would be called or what the purpose is. Example: "Two men in a barbershop having their hair cut..." rather than the more standard "Two men are in a barbershop having their hair cut...". Is it just for brevity and flow? The setup to a joke typically needs to be executed quickly - is that all it is? Is there a term for this kind of formation? Matt Deres (talk) 01:55, 22 July 2024 (UTC)[reply]

"Brevity is the soul of wit"? But if you're telling a shaggy dog story, you'd leave in all of these extraneous words. Xuxl (talk) 07:35, 22 July 2024 (UTC)[reply]
Can you give a full sentence exemplifying the phenomenon? There is nothing odd with the tenses in "Two men in a barbershop having their hair cut were startled by a strange sound" - although I'd insert a couple of commas. Also, "Two men in a barbershop having their hair cut" is a cromulent image caption.  --Lambiam 12:37, 22 July 2024 (UTC)[reply]
It's cromulent, but I would consider it marked. Like, someone made the decision to leave out the are. Your point about it being a good image caption is kind of what I'm getting at: image captions can only be evaluated within the context of the image they're captioning: this reads like an image caption, but... there's no image. Maybe I'm hypersensitive to it; I've been studying jokes for several months now and there are other stylistic choices common to jokes that are unusual elsewhere, like leaving out articles: "Man walks into a talent agent..." rather than "A man walks into a talent agent...". What I'm wondering is if there's a more specific term for this kind of thing than ellipsis. Matt Deres (talk) 20:08, 23 July 2024 (UTC)[reply]
Probably not. One style manual I'm familiar with just uses "elliptical style" to describe the omission of articles in assembly directions ("Glue wings to fuselage"), recipes ("Pour batter into 12-inch cake pan"), and the like. Deor (talk) 20:34, 23 July 2024 (UTC)[reply]
I didn't find other terms for this, but I found this grammar page that gives examples of leaving out unstressed words [[[40]]. I've seen standup comedians drop them too, for brevity and to command attention. Modocc (talk) 21:12, 23 July 2024 (UTC)[reply]
Nominative absolute may be relevant in some cases. AnonMoos (talk) 21:46, 23 July 2024 (UTC)[reply]
I think sometimes jokes have their own formulaic grammars. In German, it's very common for jokes use verb-first order, something ordinarily reserved for questions and if-clauses (with the if suppressed). So a joke beginning "A horse goes into a bar" would start in German "Kommt ein Pferd in eine Kneipe", which outside of a joke could only mean "Does a horse go into a bar?" or "If a horse goes into a bar". It is standard German to say "Es kommt ein Pferd in eine Kneipe", but in jokes you leave out the "es" (roughly = dummy "there"). —Mahāgaja · talk 10:09, 24 July 2024 (UTC)[reply]
Doesn't always work, as in the infamous Shakespearian joke intro "Or not". -- Verbarson  talkedits 19:13, 24 July 2024 (UTC)[reply]
It reminds me of the way a scene is described in a play. Play_(theatre)#Terminology has the example "Scene 1. Before the cell of Prospero," but I could imagine also "Scene 2. Man walks into a talent agent," and "Scene 3. Two men in a barbershop having their hair cut." This register (sociolinguistics) is appropriate for a setup (storytelling).  Card Zero  (talk) 22:53, 28 July 2024 (UTC)[reply]
That's the kind of thing I was thinking of. It's unfortunate that our article is shorter than your response. :-) Matt Deres (talk) 16:36, 31 July 2024 (UTC)[reply]

OSV and VSO words orders

edit

A question I was reminded of reading the thread about reverse English: in the OSV and VSO order the predicate group is separated by the subject group? Isn't that a problem? Is the notion of "predicate" with verb and object (and other complements) belonging to it universal? Isn't it more natural that words belonging to the same syntactic group be close together by default? Do OSV and VSO languages have that notion of predicate?

VSO is of course the word order (is "worder" a word?) used in Ancient Hebrew, Classical Arabic, etc.: does the predicate group (if it does exist in those languages) being split have any implication? I've noted that the verbal forms in the past (the non-past is more complicated) have exactly the same order for the suffixed pronouns: verb + subject suffix + object suffix. I've also read somewhere that Ancient Egyptian cycled through various word orders at different stages in its long history. Is that true? Is there a natural way in which word order changes? 178.51.74.75 (talk) 02:01, 22 July 2024 (UTC)[reply]

About "natural", Latin word order mentions the frequent use of hyperbaton:
Thus, Caesar's hae permānsērunt aquae diēs complūrēs, with hae "these" separated from aquae "flood waters", means not "These floods remained for several days" but "This time, the flood waters (unlike the previous ones) remained for several days."
--Error (talk) 09:41, 22 July 2024 (UTC)[reply]
V2 languages (including the Germanic languages other than English) also regularly break up the predicate. The concept of predicate (first definition) isn't used in the traditional grammar of every language and not all modern theories of syntax use the binary distinction of subject vs. predicate. PiusImpavidus (talk) 09:50, 22 July 2024 (UTC)[reply]
Even in English, the predicate may be broken up. "The devil is in the details" is traditionally analyzed as subject "the devil" + predicate "is in the details". By asking, "Is the devil in the details?", the subject is transposed to a position inside the predicate.  --Lambiam 12:25, 22 July 2024 (UTC)[reply]
In modern linguistic theories, what 178.51.74.75 calls the "predicate" is often known as a verb phrase. In the four-word sentence "Who do you love?", there are two separate breaches of the boundaries of the verb phrase. The exact answer to 178.51.74.75's question would depend on the details of specific linguistic theories (and there's probably a lot more data and linguistic discussion about VSO than OSV)... AnonMoos (talk) 17:13, 22 July 2024 (UTC)[reply]

Why is the verb yap considered a TikTok trend?

edit

The practice of referring to senseless chatter as yapping or yapping on (about something) doesn't seem new to me. Of course, the word's entry on the list of Generation Z slang points out that it dates back hundreds of years, but still, its use in its modern sense seems to have been popular for at least a bit longer than some sources might have us believe. Here's a couple of examples I can recall from the top of my head:

Quit yappin' and start workin'!

— Doc Hudson to Lightning McQueen

Is it SHUTYERYAPS?

— Grunkle Stan while competing on Cash Wheel

I doubt that "yapping" would've sounded out of place in a 20th-century Warner Bros. or Hanna-Barbera cartoon. – MrPersonHumanGuy (talk) 17:21, 22 July 2024 (UTC)[reply]

Here it is used in a play from 1953, in the sense of needless talking. Perhaps it is used on TikTok to refer to any speech act.  --Lambiam 18:34, 22 July 2024 (UTC)[reply]
It goes back a thousand years, originally in reference to dogs barking.[41]Baseball Bugs What's up, Doc? carrots19:29, 22 July 2024 (UTC)[reply]
Seems as if it's only attested from the 17th century, though. 惑乱 Wakuran (talk) 16:40, 29 July 2024 (UTC)[reply]

Every generation rejuvenates old customs, mores, folkways, and linguistic usages, and then gives itself credit for having invented them. Gen Z is no different from any other generation in this respect. --Trovatore (talk) 21:17, 22 July 2024 (UTC) [reply]
For what it's worth, as a Gen Z'er, I don't think I've ever actually seen any of us actually claim invention of the term, even if we use it a lot. Maybe I just haven't looked hard enough. GalacticShoe (talk) 16:21, 23 July 2024 (UTC)[reply]
I mean, a trend doesn't technically have to be completely new, I guess, as a Xennial. Robert Crumb popularized the Keep on truckin'... slogan, but he had picked it up from the 1936 single Truckin' My Blues Away by Blind Boy Fuller. 惑乱 Wakuran (talk) 19:52, 23 July 2024 (UTC)[reply]
No, of course not. When I was a teenager in the '80s, the fashionable word for "very good" was awesome, but awesome has had the meaning awe-inspiring since the 17th century. It wasn't a new coinage (like, say, skibidi is), but we still managed to make it "our" word, and it was embarrassing to hear a grownup try to use it the way we did. —Mahāgaja · talk 09:56, 24 July 2024 (UTC)[reply]

Suspend

edit

The verb "suspend" seems to be undergoing a change, particularly in relation to political campaigns. Once, it meant that a campaign was being halted temporarily, but not permanently. Now, we read @ Kamala Harris#2024 presidential campaign:

  • On July 21, 2024, incumbent president and presumptive Democratic nominee Joe Biden suspended his campaign for re-election in 2024 with Harris and endorsed her as the Democratic presidential nominee.

Yet, everything I've heard and read tells me Biden has abandoned his campaign, permanently and irrevocably. It's over. It's not just suspended. I've also seen this language with previous contenders who pulled out. Nobody believed they were just re-grouping or whatever, and intended to mount a reinvigorated campaign in the near future. No, they were withdrawing permanently. It seems to be a purely American phenomenon, as far as I can tell.

Wiktionary:suspend tells me @ Definition 3, that suspend can mean:

  • To discontinue or interrupt a function, task, position, or event, e.g. to suspend a thread of execution in a computer program.

That seems to be the only case where suspension can mean permanent cessation. All the other definitions are about temporary cessation. Computer jargon is often the most impenetrable rubbish, with expressions meaning things that are completely at odds with their natural meanings (eg. client). So, why is this computer jargon now being trotted out for use in political contexts? Is it that words that don't mean what they seem to mean are an obvious and natural fit for politics? </cynicism>-- Jack of Oz [pleasantries] 21:40, 22 July 2024 (UTC)[reply]

I think this relates to some obscure aspect of American campaign-finance law. If you actually officially drop out, it triggers some sort of legal requirement or complication that "suspending" the campaign does not. So campaigns are almost never officially terminated, always suspended. I don't remember the details but I'm sure someone else can help you find them, or this might be enough of a clue by itself. --Trovatore (talk) 22:03, 22 July 2024 (UTC)[reply]
Mpls in 73! —Tamfang (talk) 22:42, 22 July 2024 (UTC)[reply]
I'm hoping to attend, but I will be 119. --Orange Mike | Talk 23:31, 28 July 2024 (UTC)[reply]
This reminds me of a question I asked once on some refdesk; I think it was about Ruth Ellis. The article went into some detail about the possibility of a "reprieve", ultimately not granted.
Now to me a "reprieve" is a temporary suspension of the sentence, essentially the same thing as a "stay". But it was explained to me that the authorities had no option to actually commute her sentence, but they could have stayed it indefinitely, which is the sort of reprieve that was intended in the article. --Trovatore (talk) 23:08, 22 July 2024 (UTC)[reply]
In US presidential campaigns in recent years, especially during primary season, it has become customary for a candidate to say they are "suspending" their campaign when it's obvious they have no chance of winning. There could be legalistic nuances to that, as Trovatore indicates. In the case of Biden, he's not resigning as president (at least not yet), and if Harris suddenly decides against running, Biden could still say, "Here I am!" ←Baseball Bugs What's up, Doc? carrots00:14, 23 July 2024 (UTC)[reply]

Here's an Atlantic article that deals with the subject. It's a "gift" article that should be de-paywalled for the next couple weeks (sorry future readers of the archive).
article
Key quote:
Financially, the benefits of a suspension are even more important. "By not officially terminating his campaign, a candidate can continue to raise money to retire debt," ABC News' Domenico Montarao explained after Herman Cain similarly announced the suspension of his campaign in December. "But a candidate would not be ALLOWED to terminate unless they paid off their obligations and debts."
I think this bit about fundraising to retire debt is the nuance I was thinking about. --Trovatore (talk) 01:02, 23 July 2024 (UTC)[reply]
Ah, I see. It's a legal fiction. Enough said. Thanks. -- Jack of Oz [pleasantries] 07:38, 23 July 2024 (UTC)[reply]
Technically, I guess the campaign isn't dropped, as much as revamped from Biden-Harris to Harris-Vice (incumbent). 惑乱 Wakuran (talk) 14:59, 23 July 2024 (UTC)[reply]
No, those are two different campaigns. When fundraising, the precise wording is legally important, and in many jurisdictions it's often illegal to divert donations given for cause A to cause B, even if A is defunct and they are linked in some way. I actually served on a Crown Court jury (in the UK) where this was a background factor in the case. {The poster formerly known as 87.81.230.195}
94.2.67.235 (talk) 17:46, 24 July 2024 (UTC)[reply]
As I understood it, picking Harris allowed the Democrats to retain most of their collected donations, but I didn't understand all of the details. 惑乱 Wakuran (talk) 18:49, 24 July 2024 (UTC)[reply]
What does "retire debt" mean? DuncanHill (talk) 23:39, 28 July 2024 (UTC)[reply]
Just a fancy way of saying pay it off. (The weak implication is it's a big amount.) Clarityfiend (talk) 08:08, 29 July 2024 (UTC)[reply]
Ah, thanks, repay. DuncanHill (talk) 17:25, 29 July 2024 (UTC)[reply]

July 24

edit

Which + <noun>...? vs. What + <noun>... ?

edit

I'm looking for questions of the type "Which/What + <noun>...?", so that changing the interrogative word by the other interrogative word may sound non-native. 2A06:C701:7B31:C100:7D63:C50F:C3A5:9744 (talk) 12:47, 24 July 2024 (UTC)[reply]

Which/what car do you drive? Which/what kind of food do you like to eat?
"If we’re presented with an infinite number or an undetermined number, then “what” is always the choice. However, if the choices are narrowed down to a more specific selection, we can use “which.”" AlmostReadytoFly (talk) 14:06, 24 July 2024 (UTC)[reply]
So, can "WHAT" always be the default, for a non-native speaker unaware of that rule? 2A06:C701:7B31:C100:7D63:C50F:C3A5:9744 (talk) 14:17, 24 July 2024 (UTC)[reply]
Yes, the same site I linked above advises "It’s worth noting that “what” can be used in place of “which” it’s just more informal if you were to do so." AlmostReadytoFly (talk) 14:24, 24 July 2024 (UTC)[reply]
Thanks. 2A06:C701:7B31:C100:7D63:C50F:C3A5:9744 (talk) 14:34, 24 July 2024 (UTC)[reply]
If you have told someone there are only two options, chicken soup and tomato soup, it sounds IMO a bit peculiar if you then ask, "What soup shall I get for you?". At least to me, "Which soup shall I get for you?" sounds better.  --Lambiam 17:03, 24 July 2024 (UTC)[reply]
Phrases like "which one" and "which answer" often follow multiple choices too. For example, "Our sides are A, B, C and D, which one would you like?" is more natural than substituting "what one" and it would appear that Google's Ngram viewer confirms this. [42] Quoting the above citation: "We may already be presented with a list of potential answers, and we’re just asking someone to clarify which one applies to them." Of course, "what" is definitely used when asking open-ended questions such as:"We have A,B, C and more, what would you like?", although I think it's natural to ask "Which one would you like?" even then... Sentence order matters, compare: "What dressing would you like? We have..." to "We have... Which one would you like?". Modocc (talk) 17:32, 24 July 2024 (UTC)[reply]

Are there apps or any software, that can identify the native language of a speaker currently speaking in a non-native language (e.g. English)?

edit

2A06:C701:7B31:C100:7D63:C50F:C3A5:9744 (talk) 13:18, 24 July 2024 (UTC)[reply]

Google Translate does this. If it's not sure, such as when the words could come from multiple languages, it tells you which languages, along with the translations. Matt Deres (talk) 14:11, 24 July 2024 (UTC)[reply]
I suspect Google-Translate can't do this, unless the text is written. If it's spoken using microphone, Google-Translate can't identify the language, even when spoken by a native speaker.
Anyway, I was asking about identifying a person's native language, when they are currently speaking in a non-native language (e.g. English), rather than in their native language we want to identify. 2A06:C701:7B31:C100:7D63:C50F:C3A5:9744 (talk) 14:30, 24 July 2024 (UTC)[reply]
So you're asking if there's software that can identify someone's accent, if they're speaking English with a French/Italian/German/Swedish/Danish/Finnish/Norwegian/etc/etc/etc accent? That sounds like a big ask. AlmostReadytoFly (talk) 14:44, 24 July 2024 (UTC)[reply]
Yep. Maybe AI can help? 2A06:C701:7B31:C100:7D63:C50F:C3A5:9744 (talk) 14:52, 24 July 2024 (UTC)[reply]
Even bigger than one thinks...for example, people in France or Germany do speak with at least a little hint of their local accents (as do English speakers...Mancunian, Liverpudlian etc.)...irl, you rarely will encounter these stereotypical "accent speakers" (Maurice Chevalier) which are propagated by media. So you would need an even finer mesh to be able to differentiate between Parisian, Bavarian, or Berlinish accents/sociolects. Even AI might be a little overwhelmed by that. Lectonar (talk) 15:01, 24 July 2024 (UTC)[reply]
I would say that in the last ten years or so, genuine French and German people on British TV have taken to using increasingly comic accents. There's an alleged Frenchman called "Fred Serieuse" or somesuch who sounds like Inspector Clouseau on a very bad day. DuncanHill (talk) 23:56, 24 July 2024 (UTC)[reply]
I guess AI may overcome it, by listening to millions of speeches already known (or already defined) to be spoken in a given "typical" accent, say a "British" accent defined in advance to be a "typical British" accent. Thus AI may build a list of typical features of that typical accent. The same may be done for every other typical accent defined in advance, e.g. a "typical Mid-west American" accent, and so forth. After building a list of say 200 accents defined in advance to be "typical", the next step AI should take, is to identify - how close to a given typical accent - our own accent is. All of that may help identify in what accent we speak English, whether our accent is British or American or French or Chinese or Swahili or whatever. HOTmag (talk) 15:13, 24 July 2024 (UTC)[reply]
Estimates of the number of UK regional accents vary between 40 and 56. I think you'd need a lot more than 200. Alansplodge (talk) 18:56, 24 July 2024 (UTC)[reply]
The number of "main" accents depends on our choice: For example we can define a list of eight "main" typical accents: Chinese, Hindi, Spanish, French, Arabic, Bengali, Portuguese, Russian. All depends on what we define in advance to be "main". Then, AI should compare our own accent with this list only, and the output should be something like: "The typical accent closest to your own accent is Chinese", even if I'm a native English speaker, because English is not on the specific selected list defined in advance. In my previous response, I suggested a list of 200 accents defined in advance to be: "[main] typical accents". HOTmag (talk) 23:52, 24 July 2024 (UTC)[reply]
There are local accents too, right? I mean, many people can within their own region pinpoint the exact village a speaker comes from. One village sounds significant different from the next, just three kilometres away. PiusImpavidus (talk) 11:23, 25 July 2024 (UTC)[reply]
You can statistically analyse the speech:
  • How is each phoneme pronounced exactly? Part of this is accent, part is voice, part is random variation, so there's a limit to the amount of information you can extract from that.
  • What's prosody like? To what extend is it stress timed, syllable timed or mora timed? There's a continuum in that. Does the speech appear tonal? We can assign numbers to that.
  • How are words chosen? For example, English often has pairs of synonyms, one from Germanic, one from Romance, and the speaker may have a preference for one of them.
  • Look at syntax. Not only the errors, but also the ratio of alternative constructions both grammatical in the language. For example, English allows preposition + pronoun (in which) and a pronominal adverb (wherein) as alternatives, with the former far more common. If the speaker prefers the latter, chances are he's Dutch (or a lawyer).
Also, the question is about finding the native language of the speaker, not the accent. A person from Amsterdam speaking English has a different accent than one from Antwerp speaking English, although their native languages are the same.
So, you have to record enough speech to get statistics with sufficiently small error bars, collect similar statistics for a large number of accents, each belonging to some native language, then you can compute a score for how well the speech fits a particular native language. You first have to find the accent, then the language and there are more accents, so you need more statistics.
It's just statistics, no intelligence needed, artificial or otherwise (some intelligence may be required for tagging the recordings). But then, AI is just a huge pile of statistics with software calculating correlations. Collecting the statistics may be a challenge and I doubt a suitable corpus is available to make this work well. PiusImpavidus (talk) 11:18, 25 July 2024 (UTC)[reply]

I think the short answer is no. There is no currently functional app to detect accents. It could exist in the future, but even if an AI could utilize the existence of millions of youtube videos there would still be a lot of arbitrary positives or errors, since in the end of the day each human has a unique way of speaking and speech changes over time. Now there would be significant interest for such an app for forensic investigations for Immigration authorities, who has a lot of problems with identifying 'true' geographic origins of individuals. See [43], Language analysis for the determination of origin. --Soman (talk) 10:36, 25 July 2024 (UTC)[reply]

using past perfect verbs with dates in the same sentence

edit

Should past perfect verbs be used in a sentence beginning with the year that a past date happened? I was copyediting this article's history section and found the use of "had" repetitive. When looking into when and when not to use them, I was stuck on the appropriateness of using past perfect in a sentence like "By the end of the financial year in 1874, 1,100 pounds had been spent in construction, of an estimated total of 13,200 pounds." Thank you for your help! Decsok (talk) 18:46, 24 July 2024 (UTC)[reply]

Entirely appropriate. According to Pluperfect (aka past perfect) the tense relates 'to an action that occurred prior to an aforementioned time in the past'. You have a textbook example. Starting from a specific past moment (EoFY 1874), it then refers to one or more preceding events (x pounds had been spent). Alternatively, it could have said 'In FY 1874, x pounds were spent', thus not splitting the spending events from the end of financial year. -- Verbarson  talkedits 19:02, 24 July 2024 (UTC)[reply]
Thank you! Decsok (talk) 19:38, 24 July 2024 (UTC)[reply]
If money had been spent on the same project in earlier financial years, the alternative formulation is not equivalent. It could be that 300 pounds were spent in construction in FY 1873, and a further 800 pounds in FY 1874. Then, by the end of FY 1873, 300 pounds had been spent in construction, which, by the end of FY 1874, had grown to a whopping 1,100 pounds, but in FY 1874 only 800 pounds were spent.  --Lambiam 00:24, 25 July 2024 (UTC)[reply]

July 29

edit

Tais everywhere!

edit

Looking at the names of four Taiwanese cities, Taipei (臺北), Taichung (臺中), Tainan (臺南) and Taitung (臺東), it can be seen that they share the same first character (臺 Tai, as in 臺灣 Taiwan) while the second one indicates their positions in relation to the island of Taiwan (north, center, south and east respectively). What I'm asking is why the "positional" character is the second one and not the first? I'm not a speaker of Chinese but, from what I know about it, I would be expecting a scheme such as: North-Tai, Center-Tai, South-Tai and East-Tai, not the opposite. Can someone explain this situation to me? Are there other examples like this? (I'm erasing this second question because evidently it may sound like I'm asking about any language while I'm talking specifically about Chinese.) Thank you! 79.35.53.87 (talk) 00:06, 29 July 2024 (UTC)[reply]

Paris-Nord, Paris-Sud, Paris-Est.  --Lambiam 11:59, 29 July 2024 (UTC)[reply]
Word order is not the same in all languages.DOR (ex-HK) (talk) 20:09, 29 July 2024 (UTC)[reply]
Chinese has Beijing and Nanjing, but also Hubei and Hunan. Is there a systematic to these differences or is the order free? The translations suggest that direction in the first position indicates an adjective characterising the second part (northern and southern capital), whereas second position represents a direction with respect to the first part (to the north and south of the lake; in the north/south/centre of Taiwan?). I've forgotten too much Chinese... --Wrongfilter (talk) 20:43, 29 July 2024 (UTC)[reply]
There's also Hainan and Nanhai. both made up of Nan/南 and Hai/海。 I think it's just names. I have no experience of Taiwan but in China it's common to form Chinese names of two or three characters with little regard for grammatical meaning/word order. I am thinking of personal names and company names, but it surely applies also to place names which will still be being devised today. And who knows, in a hundred or a thousand years maybe a village being named today will be the capital of a future state.--2A04:4A43:901F:FE56:7C29:7255:3786:BFF3 (talk) 06:46, 30 July 2024 (UTC)[reply]

An Egyptian "god of the extended arm"

edit

On pg 2 of Budge's Book of the Dead lexicon, where is the reference? "Au-a", the "god of the extended arm." I'd like to see that page of papyrus. It sounds like a famous Exodus-related phrase. Deuteronomy 4:34 and 5:15 "with a mighty hand and outstretched arm." Temerarius (talk) 03:01, 29 July 2024 (UTC)[reply]

July 30

edit

What is this hieroglyph?

edit

What's the doughy, ugly glyph on the upper right and lower left? https://i.postimg.cc/KzWSv8fr/IMG-3692.jpg [1] Temerarius (talk) 19:51, 30 July 2024 (UTC) Temerarius (talk) 19:51, 30 July 2024 (UTC)[reply]


References

  1. ^ Pinch, Geraldine (2010-03-01). Magic in Ancient Egypt. Austin: University of Texas Press. p. 19. ISBN 0-292-72262-1.

Is there a generic term for "speaker or signer" yet?

edit

It is uncontroversial both that signed language is truly its own modality distinct from speech, and also that much linguistics work limits itself to the latter and will be up-front about that. That's fine in most cases—they are very different modes, so it's hard to write about both!—but it struck me just now that every interdisciplinary work I've read has to go out of its way to repeat "speaker or signer" if desiring to explicitly indicate both spoken and signed language. What would we choose if we had to invent one? Utterer? Seaker? Spigner? Remsense 20:00, 30 July 2024 (UTC)[reply]

"Communicator"? Maybe still not clear outside of context... 惑乱 Wakuran (talk) 22:39, 30 July 2024 (UTC)[reply]
Another term that does not depend on the modality of the communication channel is sender. Yet another modality-independent term is messager, but sender has the advantage of having a modality-independent term for the counterpart, namely receiver.
 --Lambiam 22:43, 30 July 2024 (UTC)[reply]
Sender seems particularly suited for general interdisciplinary linguistics discussion, thanks!. Remsense 22:46, 30 July 2024 (UTC)[reply]

What are these cuneiform signs?

edit

This may be a follow-up on my question of Ishtar spellings. Is dingir "Ur" here, and the glyphs to the side are Istaran? Or is the dingir "ra" in Istaran?

https://i.postimg.cc/9X119kHK/istaran.png

Temerarius (talk) 20:05, 30 July 2024 (UTC)[reply]

The eight-pointed asterisk is a version of the "star" or "heaven" sign (see article An (cuneiform)), and likely is a determinative sign indicating that a deity name follows, without itself having any pronunciation in this context (that's what the superscript "d" means). The signs which actually write the name itself seem to be loosely sketched in. This image has a rather low relevant information content for its 1.9 megabyte file size. I'm not sure why you're seeking the Akkadian goddess name "Ishtar" in a Sumerian context. I have a scan of a page from a 19th century book which lists Akkadian deity names in 1st millennium B.C. Assyrian versions of cuneiform signs, and it gives several alternative spellings for Ishtar, none of which look anything like the vague rectangles in the 1.9 megabyte image. I'm not sure of the source or where I downloaded it from (in 2006), so can't upload it here... AnonMoos (talk) 23:31, 30 July 2024 (UTC)[reply]
Because I want to know every spelling, all attestations of Asherah, Ishtar, Astarte, and anything that can be confused for them. I'd love to see that page of yours if you can think of anywhere to upload it. Temerarius (talk) 01:21, 31 July 2024 (UTC)[reply]
I'll do a reverse Google image search later today to see if I can track down its source. I don't know anything about image hosting services (have never used one as uploader). Also, the name Asherah has nothing to do with the others: in Hebrew and similar languages it begins with an Aleph or voiceless glottal stop, while `Ashtaroth (singular `Ashtoreth) begins with an `Ayin or voiced pharyngeal consonant. In the older non-Eastern Semitic languages, these are two completely separate consonants, as distinct as "p" and "k", though this can be obscured by the confusable apostrophes often used in Latin alphabet transcriptions. I doubt that the two goddess names were confused in Hebrew... AnonMoos (talk) 12:03, 31 July 2024 (UTC)[reply]
Was Asherah an actual name, or a title that could be applied to female consorts of Caananite gods in general? (I'm sure I've seen a passage translated as ("X and his Asherah.")
Either way, did it perhaps derive (in changed form) from an older name of a particular goddess in Mesopotamian (or another) mythology? {The poster formerly known as 87.81.230.195} 94.2.67.235 (talk) 00:36, 1 August 2024 (UTC)[reply]
Ishakshar? DuncanHill (talk) 00:42, 1 August 2024 (UTC)[reply]

July 31

edit

Argelasse

edit

Provence, it has been said, "smells of argelasse and wild lavender and broom...". Lavender and broom I know, but what is argelasse? Thank you, DuncanHill (talk) 00:18, 31 July 2024 (UTC)[reply]

Websearching only finds it as an Italian surname, bar the one story on Wikisource you've doubtless seen. I suspect it's a misprint, but I haven't been able to figure out what for: Angelica seems to fit the context, but is too different.
The same spelling appears in the story in the collection on Project Gutenberg, but that is Wikisource's source, so no help. {The poster formerly known as 87.81.230.195} 94.2.67.235 (talk) 01:06, 31 July 2024 (UTC)[reply]
Possibly Artemisia (plant). 41.23.55.195 (talk) 05:04, 31 July 2024 (UTC)[reply]
I'd go for fr:Ulex parviflorus, "known in Languedoc and Provence under the names of argelàs, argelat or argeràs". However, that name is also given to fr:Genêt épineux (the article gives the occitan form arjelàs). --Wrongfilter (talk) 05:49, 31 July 2024 (UTC)[reply]
According to French Wiktionary the plural of Occitan argelàs is argelasses. They go with the genêt épineux meaning. --Antiquary (talk) 09:34, 31 July 2024 (UTC)[reply]
This seems to be the answer, especially as épineux is described as noted for its scent, whereas the Ulexes are (in my experience) less obviously scented.
Curious that Onions chose to use a name presumably unfamiliar to most English readers, when he could have used broom with reasonable accuracy, though perhaps I underestimate the Victorians' grasp of botany and Provencal. {The poster formerly known as 87.81.230.195} 94.2.67.235 (talk) 12:14, 31 July 2024 (UTC)[reply]
"smells of broom, wild lavender and broom" would be rather an odd thing to say. "smells of gorse, wild lavender and broom" seems to me to be far more likely. The use and status of Provencal is alluded to in the story. Gorse blossom smells of coconut. DuncanHill (talk) 17:19, 31 July 2024 (UTC)[reply]
Yes, my mistake. I meant gorse, but had become confused by all the related plants I was looking up. And I did read the story. (The poster formerly known as 87.81.230.195} 94.2.67.235 (talk) 00:29, 1 August 2024 (UTC)[reply]

Why do people use "IRL" even when simply "RL" is more grammatically correct? 120.148.140.163 (talk) 09:30, 31 July 2024 (UTC)[reply]

"In real life" is grammatically fine. ←Baseball Bugs What's up, Doc? carrots10:34, 31 July 2024 (UTC)[reply]
IRL is an established acronym, so a phrasing like "IRL Situation" would likely be more understod than "RL Situation". 惑乱 Wakuran (talk) 11:42, 31 July 2024 (UTC)[reply]
See also RAS syndrome for a related situation. Matt Deres (talk) 16:37, 31 July 2024 (UTC)[reply]

Blue Skies

edit

I have just received a couple of emails from a former MP which he signs "yours faithfully and Blue Skies". What does "Blue Skies" mean in this context? Thank you, DuncanHill (talk) 20:14, 31 July 2024 (UTC)[reply]

It originated in the world of aviation, and means best wishes, have a great day full of clear weather yada yada.
Be thankful they didn't mention green fields or low-hanging fruit. -- Jack of Oz [pleasantries] 20:56, 31 July 2024 (UTC)[reply]

August 1

edit

Entertainment

edit

July 17

edit

Who is the female dancer in the video of New Order's "Tutti Frutti"?

edit

In the video of New Order's "Tutti Frutti" there's a young woman playing a dancer who appears throughout the video and is a central character in it. She's obviously the main dancer. When there's dancing the camera is centered on her, although there's usually a few other female dancers around her. She has long brown hair. Despite her importance in the video I could find nothing about her at the usual places (Imdb, etc.) Various AI assistants I've asked return nothing. Do you know of other sources for information about this video? Or can you recognize the woman? For example she's at 1m08-1m13, 1m42-2m09, etc. 178.51.74.75 (talk) 00:36, 17 July 2024 (UTC)[reply]

This site names one Katja Škofic. (Google Translate: The group New Order recorded the video for their single Tutti Frutti in the premises of TV Slovenija, in which, in addition to the Italian actor Ricky Tognazzi, the Slovenian dancer and actress Katja Škofic appears.)--Wrongfilter (talk) 15:45, 17 July 2024 (UTC)[reply]

Billboard's charts

edit

What are the links to Billboard's Digital Song Sales Chart and Billboard's Country Digital Song Sales Chart for the first week of October 2023? Thanks. Bob K31416 (talk) 15:50, 17 July 2024 (UTC)[reply]

Digital songs for first week of October 2023 and Country songs. Billboard lets you pick the week you want to look at and is free access. Matt Deres (talk) 15:28, 19 July 2024 (UTC)[reply]
Very helpful. I was looking for a cover of "I Won't Back Down". Using your links as a starting point, I found it for the week of Oct 14, 2023 for Digital Song Sales at #10 [44] but not for Country Digital Song Sales (#6 was the supposed place). Bob K31416 (talk) 18:05, 19 July 2024 (UTC)[reply]
For country, I think I found what I was looking for. It looks like the country link in your message is for "Hot Country Songs", which apparently is different from "Country Digital Song Sales" (see [45] p. 8). At the latter is the "I Won't Back Down" cover at #6. Bob K31416 (talk) 06:45, 20 July 2024 (UTC)[reply]

July 19

edit

draft declined

edit

I have drafted an article named Last drop that has been declined. Please help me. Pratap Keshari Das (talk) 18:07, 19 July 2024 (UTC)[reply]

Are you talking about Draft:Last Drop? ←Baseball Bugs What's up, Doc? carrots21:38, 19 July 2024 (UTC)[reply]
@Pratap555: If you don't understand the explanation in the decline notice, you should click the "Ask us a question" blue link in the draft decline message, to ask at the AfC help desk. RudolfRed (talk) 01:13, 20 July 2024 (UTC)[reply]
The implied question is, "Can you please help me find sources to establish the notability of this film"? (In particular, sources with significant coverage of the topic, not mere mentions, but per Wikipedia:Notability (films) a major award would also do.)  --Lambiam 10:18, 20 July 2024 (UTC)[reply]
There are also various problems with the verbiage in the article. ←Baseball Bugs What's up, Doc? carrots18:09, 20 July 2024 (UTC)[reply]
As long as the text is understandable and does not violate policy, this should not an issue for reviewers deciding whether to accept or decline a submitted draft. But you're welcome to improve it as you see fit.  --Lambiam 20:37, 20 July 2024 (UTC)[reply]
"...ingeniously uses a simple idea—a leaking tap – to deliver a profound message about water conservation. Through its minimalist approach." That sounds like it was lifted from a movie review somewhere. ←Baseball Bugs What's up, Doc? carrots23:37, 20 July 2024 (UTC)[reply]
Can you prove this? If not, it is skirting very close to calling the editor either incapable of writing a cohesive, comprehensible sentence or a flat-out plagiarist.Maineartists (talk) 12:25, 21 July 2024 (UTC)[reply]
I'm just saying what it sounds like. And as noted below, IMDB is being used as a source for the text, which is not kosher. ←Baseball Bugs What's up, Doc? carrots13:49, 21 July 2024 (UTC)[reply]
"Last Drop" ingeniously uses a simple premise - a leaking tap - to deliver a profound message about water conservation. Reviews: Last Drop - IMDb Alansplodge (talk) 11:33, 21 July 2024 (UTC)[reply]
So, not only very adjacent to plagiarism, but also favourably judgemental and therefore not NPOV: "ingeniously", "profound". {The poster formerly known as 87.81.230.195} 94.2.67.235 (talk) 17:42, 21 July 2024 (UTC)[reply]
The reviews read like they were written by ChatGPT.  --Lambiam 20:43, 21 July 2024 (UTC)[reply]

July 21

edit

For the Sake of Appearance

edit

Do any of you guys know any information about the 1973 tv series titled For the Sake of Appearance. I'm wanting to find information about the show because I'm creating a draft for it and I want to know if the show has survived so can any of you send me the links here. Matthew John Drummond (talk) 12:52, 21 July 2024 (UTC)[reply]

Genome listing Nanonic (talk) 12:58, 21 July 2024 (UTC)[reply]
Has the show survived or is it now lost. I need to know for this Wikipedia page. Matthew John Drummond (talk) 05:19, 22 July 2024 (UTC)[reply]

Older child actors playing younger characters

edit

Why do most older child actors play younger characters? For instance; 8 or 9-year-olds as 6 or 7-year-olds, 13 or 14-year-olds as 11 or 12-year-olds and 15 or 16-year-olds as 12 or 13-year-olds. 86.129.82.17 (talk) 19:57, 21 July 2024 (UTC)[reply]

Who says they do? ←Baseball Bugs What's up, Doc? carrots22:16, 21 July 2024 (UTC)[reply]
There are legal limits to the work children can do: maximum hours per day and per week, dangerous environments, night-time work. Those rules get less strict as the child gets older, making it easier for film makers to work with older children. Furthermore, they become better actors as they get older. So it's best to work with the oldest available child that can believably play the age asked for by the script. PiusImpavidus (talk) 08:15, 22 July 2024 (UTC)[reply]
Which doesn't explain how Jack Benny, who had then been 39 for several years, played an Oxford undergrad in Charley's Aunt. —Tamfang (talk) 00:49, 1 August 2024 (UTC)[reply]
Suspension of disbelief is a requirement to enjoy most movies. I was very confused when I first saw Grease that a thirty-year-old woman was still in high school. It remains to be seen what AI will be able to do with the apparent age of actors. Shantavira|feed me 12:40, 22 July 2024 (UTC)[reply]
Who needs (human) actors when we have AI?  --Lambiam 14:19, 23 July 2024 (UTC)[reply]

July 22

edit

The Reprieve

edit

Has the 1972 short film titled The Reprieve survived or is it now lost. The film was produced by National Film and Television School. Matthew John Drummond (talk) 05:31, 22 July 2024 (UTC)[reply]

@Matthew John Drummond In that case I suggest you ask the National Film and Television School whatever your question is. Shantavira|feed me 08:54, 22 July 2024 (UTC)[reply]
Our Article includes their website address. By the way, have you been utilising the resources of the BFI National Archive regarding these sorts of enquiries? {The poster formerly known as 87.81.230.195} 94.2.67.235 (talk) 11:59, 22 July 2024 (UTC)[reply]
I checked the BFI National Archive and it's not shown on the results. Matthew John Drummond (talk) 17:55, 22 July 2024 (UTC)[reply]
Where can I ask the National Film and Television School my question. Matthew John Drummond (talk) 17:58, 22 July 2024 (UTC)[reply]
NFTS, and scroll down to Get in Touch. -- Jack of Oz [pleasantries] 18:13, 22 July 2024 (UTC)[reply]

Trial

edit

Has the 1971 British tv series titled Trial has this tv show survived or is it now lost. The show had a total of 13 episodes. Matthew John Drummond (talk) 18:15, 22 July 2024 (UTC)[reply]

Was it shown live or was it on film or tape? And I'm curious to know where you're finding these things! ←Baseball Bugs What's up, Doc? carrots20:26, 22 July 2024 (UTC)[reply]
It was broadcast on television. Matthew John Drummond (talk) 04:10, 23 July 2024 (UTC)[reply]
That much is clear. But if it was broadcast "live", it might never have been recorded. ←Baseball Bugs What's up, Doc? carrots08:24, 23 July 2024 (UTC)[reply]
I don't know about the UK, but in those days, magnetic tapes for recording television programs were very expensive and often re-used by broadcasters to record a different program after an initial airing or two. In Canada, we've lost tapings of a whole hosts of popular shows from the 1960s and early 1970s that would now be considered classics, as they featured well-known actors, but that were too expensive to preserve at the time. It's even more likely to have happened for made-for-tv movies that were not expected to have much of a shelf life to begin with. Xuxl (talk) 12:30, 23 July 2024 (UTC)[reply]
It did indeed happen a lot in the UK in that era. See Lost television broadcast#United Kingdom, and Doctor Who missing episodes for a high profile example of a programme that was affected and ways some were recovered. As noted in those articles, it wasn't just tapes that were recorded over but film made of the shows were often destroyed for lack of space. The general article does mention more general recovery efforts (I'm assuming from below this was a BBC programme) but for a programme we don't even have an article, I'm not sure how much effort would have been made to find the lost content. Although possibly there efforts are fairly widespread to cover all lost content of theirs, especially since if copies to do exist somewhere they're likely to be lost over time. Nil Einne (talk) 13:50, 23 July 2024 (UTC)[reply]
Getting old episodes is hampered by a common belief that the BBC sued someone for having old archives that were supposed to be wiped or returned long ago. 12.116.29.106 (talk) 14:07, 23 July 2024 (UTC)[reply]
I noticed that Trial (TV series) redirected to an unrelated show, but the lists of BBC shows linked to that page for the 1971 series. So, I broke the redirect. If you hurry you can see that it is an independent page for the moment. I'm sure someone will revert my change within the next 30 seconds. 12.116.29.106 (talk) 12:25, 23 July 2024 (UTC)[reply]
Yes. It was reverted. I gave up on attempting to do much on Wikipedia long ago as any effort I made was reverted with one click. 75.136.148.8 (talk) 11:31, 24 July 2024 (UTC)[reply]
The solution to that is to make your efforts in accordance with Wikipedia's policies. If you add something, cite it to a reliable source. If you remove something, give your policy-based reason in the edit summary. {The poster formerly known as 87.81.230.195} 94.2.67.235 (talk) 12:57, 24 July 2024 (UTC)[reply]
The problem is that no matter how much research, explanation, and work you do, it only takes one click to revert it without a reason or explanation. In this case, the Trial article is linked to from pages referring to the 1971 television series. But, someone (who knows who, who knows why) thinks it should link to a completely unrelated television series. That cannot be undone. No amount of research, explanation, discussions, or voting can fix it. Try to split it out to two article... click, revert. Have multiple threads agreeing to make it two articles... click, revert. It is simply too easy to revert without giving a reason. 75.136.148.8 (talk) 14:58, 24 July 2024 (UTC)[reply]
No research was in evidence for the page, which is why it was reverted by user JalenFolf just today. ←Baseball Bugs What's up, Doc? carrots18:52, 24 July 2024 (UTC)[reply]
An unsourced statement in the Trivia section on IMDb says, "All episodes of this series are still intact and still all exist to this day.".[46]  --Lambiam 14:15, 23 July 2024 (UTC)[reply]
Trying to watch them is the problem. Matthew John Drummond (talk) 15:42, 23 July 2024 (UTC)[reply]
TV Brain indicates most of the episodes are missing - three survive. https://www.tvbrain.info/tv-archive?showname=the+trial&type=lostshow TrogWoolley (talk) 08:28, 24 July 2024 (UTC)[reply]


July 25

edit

It Happened Like This

edit

Has the 1962-1963 tv series titled It Happened Like This survived or is it now lost. Matthew John Drummond (talk) 02:59, 25 July 2024 (UTC)[reply]

TV Brain indicates it is lost https://www.tvbrain.info/tv-archive?showname=It+happened+like+this&type=lostshow TrogWoolley (talk) 05:09, 25 July 2024 (UTC)[reply]

The Renegade

edit

Has the 1961 tv play titled The Renegade survived or is it now lost. Matthew John Drummond (talk) 03:06, 25 July 2024 (UTC)[reply]

TV Brain indicates it is lost https://www.tvbrain.info/tv-archive?showname=The+renegade&type=lostshow TrogWoolley (talk) 05:14, 25 July 2024 (UTC)[reply]

Crying Down the Lane

edit

Has the 1962 tv Mini series titled Crying Down the Lane survived or is it now lost. Matthew John Drummond (talk) 10:19, 25 July 2024 (UTC)[reply]

Have you checked in that TVBrain website that's been pointed out to you? ←Baseball Bugs What's up, Doc? carrots13:02, 25 July 2024 (UTC)[reply]
Not everything on there is true because bfi collections have that episodes from some tv shows are available to watch on the bfi collections but on tv brain some episodes of some shows say that there lost when they actually aren’t for example the 1965-1966 tv show titled Blackmail shows the episode titled The Set Up is missing when on bfi it shows that it may exist in the bfi archive there’s also some other episodes from the tv show Blackmail that say there lost on tv brain when there actually available to watch at the bfi archive. Matthew John Drummond (talk) 16:02, 25 July 2024 (UTC)[reply]
It sounds like you've got a better handle on this than anyone else here. ←Baseball Bugs What's up, Doc? carrots03:00, 26 July 2024 (UTC)[reply]
According to this article, the BBC usually reused video tape as it cost £120 pounds per hour (about £2,750 today). So if it's not showing up on Google searches (I couldn't find anything either), then it's probably lost. Alansplodge (talk) 12:02, 26 July 2024 (UTC)[reply]
If you can't find anything about Crying Down the Lane for 1962 can you find out if this also applies to the 1973 tv series For the Sake of Appearance. Matthew John Drummond (talk) 19:12, 26 July 2024 (UTC)[reply]

July 26

edit
edit

Tencent owns 100% of Riot. Even if Honor of Kings is a blatant ripoff of League of Legends, Riot has no right to sue Tencent. I don't understand why Tencent was afraid of Riot? They later changed their game to Arena of Valor to release in the West due to Riot's copyright complaint. 68.187.65.220 (talk) 23:27, 26 July 2024 (UTC)[reply]

See Wikipedia:Reference desk/Archives/Entertainment/2022 November 20 § Arena of Valor.  --Lambiam 02:42, 27 July 2024 (UTC)[reply]

July 27

edit

Victor and Hugo: Bunglers in Crime

edit

Which episodes of the 1991-1992 tv show Victor and Hugo: Bunglers in Crime where released on audio cassettes tapes. Matthew John Drummond (talk) 01:26, 27 July 2024 (UTC)[reply]

July 29

edit

Eisenhower's farewell address

edit

Why was Eisenhower's farewell address taken in very low quality black and white video? He was the first U.S. president on color TV on May 22, 1958! -- Toytoy (talk) 06:06, 29 July 2024 (UTC)[reply]

You say "very low quality" black and white, but it was probably the standard quality at the time; color (see Professional video camera) was at that date still new, and (I'm fairly sure) significantly poorer in general definition than B&W. {The poster formerly known as 87.81.230.195} 94.2.67.235 (talk) 14:25, 29 July 2024 (UTC)[reply]

Everyday Readers

edit

Is there anywhere I can watch the 1998 film Everyday Readers starring Peter Sallis. The only website that I've found out about this movie is on the BFI. 2A00:23C8:9DEE:900:FD51:C289:48F0:AA3 (talk) 09:40, 29 July 2024 (UTC)[reply]

The Brazilian soccer player Sócrates had a medical degree

edit

The Brazilian soccer player Sócrates had a medical degree. His article mentions that and gives a source. But he's not in the List of athletes with advanced degrees. Just letting you know. I'd add him myself but I'm not sure how to add the source of the first article to the second article. 178.51.2.117 (talk) 18:18, 29 July 2024 (UTC)[reply]

The article says he has a bachelor's degree, which is not an advanced degree according to the definition on that list page ("PhDs and other degrees at that academic level"). So maybe that's why. Although saying that, the definition of what level a medical degree is seems complicated according to Doctor of Medicine. --Viennese Waltz 20:05, 29 July 2024 (UTC)[reply]

Baseball: error in a perfect game

edit

Error (baseball) presents an unusual situation:

If a batted ball were hit on the fly into foul territory, with the batting team having no runners on base, and a fielder misplayed such ball for an error, it is possible for a team on the winning side of a perfect game to commit at least one error, yet still qualify as a perfect game.

Whether at the major league level or elsewhere, is there any record of a perfect game in which the winning team committed such an error? Nyttend (talk) 23:14, 29 July 2024 (UTC)[reply]

July 30

edit
edit

Dear All A friend of mine is working on his own game which features kobolds (small lizard-creatures who worship dragons, like the ones from "Dungeons & Dragons"). I know that kobolds are originally from European folklore. I did some researcha and I've found the story "Der Kobold", from "Die deutschen Volkssagen" by Friedrich Ranke, published in Munich in 1924, which states that small, dragon-like kobolds did exist in early medieval folklore. So my question is this: are the lizard-like kobolds copyright protected or can they be freely used since they are a part of old European folklore? Thank you very much for your replies! 85.4.154.199 (talk) 22:14, 30 July 2024 (UTC)[reply]

This depends, at least partially, on the country where your friend is located. Your IP address geolocates to Switzerland; is that where your friend is located? Generally, concepts centuries old are not protected by copyright, but some countries have special legal protection for what they consider cultural heritage. You or your friend should talk with a copyright lawyer. Nyttend (talk) 22:54, 30 July 2024 (UTC)[reply]

You are correct, I'm from Switzerland! My friend is also from Switzerland, but he currently lives in the USA (he's a student at a university in Boston, Massachusetts).--85.4.154.199 (talk) 22:59, 30 July 2024 (UTC)[reply]

The general idea of small mischievous reptilian creatures cannot be copyrighted, but any specific artistic form in a published work is in principle protected by copyright. Compare talking dogs: you can publish a comic strip about a talking dog just fine, but if your dog looks much like Goofy, you can expect problems.  --Lambiam 23:30, 30 July 2024 (UTC)[reply]
Friedrich Ranke died in 1950, so copyright on his work has expired. The copyright term in Germany is life of the author plus 70 years. You're free to borrow from him whatever you like. I think copyright on Goofy hasn't expired yet in the US, but is expected to do so at the end of 2027 (publication+95 years). You can begin writing stories featuring him for publication in four years, at the risk of the US increasing copyright term again. PiusImpavidus (talk) 08:37, 31 July 2024 (UTC)[reply]


August 1

edit

Miscellaneous

edit


July 16

edit

AI abundance vs. AI servitude

edit

The following discussion is closed. Please do not modify it. Subsequent comments should be made on the appropriate discussion page. No further edits should be made to this discussion.


Just to be clear, I am not asking a question about a prediction, I am asking a question about the finer points (data, evidence) that supports the techno-libertarian idea behind the theory that Elon Musk promotes, for example, when he says "AI would eventually replace all jobs on Earth, making employment optional and transforming jobs into hobbies as AI and robots would provide all necessary goods and services." My question relates to the bolded text. Why is this sunny outcome even considered likely, when the historical reality shows that a darker, dystopian Elysium-like outcome is far more likely to occur, where most of humanity is forced into dire poverty amidst deteriorating environmental and social conditions, while the wealthy who benefit from AI escape to their own walled gardens and new societies free from the teeming masses? Again, I’m not asking about a prediction about the future, I’m asking on what basis are we supposed to accept the idea that AI will benefit humanity? The technological innovation of agriculture clearly didn’t benefit the masses of humanity, and likely enslaved the great majority of them in some form or another. Why will AI be any different? Viriditas (talk) 02:46, 16 July 2024 (UTC)[reply]

I want to add something else. I’ve recently been working on several articles about the history of pineapple. One of the most interesting things I discovered within this 500 year time frame, was that the best tasting pineapple varieties were for the most part extirpated. It turns out that there is an inverse relationship between taste and commercial viability (canning size, preservation, etc). In other words, what we know as commercial pineapple in the modern era is representative of the worst tasting pineapple cultivars, but those also happened to be the easiest to grow, produce, and distribute, hence the reason they were chosen and the others were discarded and disappeared (Side note, this may be true for all fruit varieties, I don’t know, but there was a recent article that implied the same holds true for commercial strawberries and blueberries). Why would this kind of thing not also happen with AI, such that the most beneficial AI tailored to help humanity progress to a post-scarcity society would be weeded out to serve the interests of resource extraction and scarcity instead? Viriditas (talk) 03:05, 16 July 2024 (UTC)[reply]
The current pineapple you find in stores is in no way the worst tasting pineapple. There are many varieties that taste far worse. Some taste like chewing on wet grass. Similarly, the avacado you can purchase in a store is nowhere close to the worst tasting avacado. Both of these are not "easy" to cultivate. Pineapple is very difficult to cultivate and requires a lot of land and labor, which is why you don't see pineapple farms everywhere. Avacados are even harder to cultivate. It is possible that there is a relative to both the pineapple and avacado that tastes better, but implying that we only cultivate the worst tasting fruits to increase profits is not justified. Another example is the tomato. A much easier to cultivate tomato was developed, but it tasted terrible. So, it was abandoned. As for the overall claim that AI will create a few rich and many poor, there were few rich and many poor in the beginning of recorded history. They had oral traditions talking about the few rich and many poor long before recorded history. Throughout all of history, humans have created nations with a few rich and many poor. Why wouldn't the future be a world of a few rich and many poor? That isn't the result of AI. That is the result of humans. A few rich exploit the many poor until conditions are so bad that the poor overthrow the rich and replace them with a new group of few rich. Alternately, a few rich in one area get their many poor to fight with the many poor of another area so they can take away from another minority of rich. If humans were studied like we study all other animals, this would be labeled normal human behavior. 12.116.29.106 (talk) 11:16, 16 July 2024 (UTC)[reply]
My source was Johanna Lausen-Higgins of Royal Botanic Garden Edinburgh. She's notable for her hands on work on the Lost Gardens of Heligan. She's a horticulturist and garden historian who studied at Bristol University. In the event that I misrepresented her, and it sounds like I did, here's a transcription of her 2020 lecture:

So the first classification that was done was by [Donald Monroe?] in 1835, and he lists around 52 different cultivars, many of which are thought to be lost now. And, this modern cultivar, iconically named [51MD?], really shows why many of these cultivars are lost now. [You've] got all the attributes of easier handling: you've got a smooth edge to the leaf, but also if you look at the outline of the fruit, it's basically been bred to fit neatly into a tin can, [so] cutting machines could cut equal slices with minimal loss...but the cultivars that were particularly favored in the 18th and 19th centuries, [are] very different. [You] see this strongly tapering outline to the fruit. And in the case of Sugarloaf and Queen...they also have very strongly outward protecting fruitlets. Which again is something that is not favored in the canning industry. And again, I can tell you the flavors are so incredible in these different old cultivars. So Abacaxi, so "Black Prince" now, a lost cultivar, is probably an Abacaxi type, actually has white flesh and really unusual, subtle flavors. You can still read in modern treatises on the pineapple, that Smooth Cayenne...this is the one that dominates the trade, has by far the poorest flavour. It's got the highest acidity and also possibly the highest amount of bromelain. Whereas Queen or Sugarloaf, which were particularly favoured as well...the aroma and the flavours are extraordinary.

Thanks. Viriditas (talk) 23:13, 16 July 2024 (UTC)[reply]
Throughout all of history, humans have created nations with a few rich and many poor. Why wouldn't the future be a world of a few rich and many poor...If humans were studied like we study all other animals, this would be labeled normal human behavior.
Forgive me, but this sounds identical to an appeal to tradition. It's also the same argument abolitionists were met with when they opposed slavery. They were told that slavery was natural, and it was normal, and even that god approved of it. They were also told that they were going against the natural order of things in their opposition to it. Viriditas (talk) 23:17, 16 July 2024 (UTC)[reply]
Example: Thomas Roderick Dew (1802–1846) professor, public intellectual, president of The College of William & Mary (1836-1846). Best known for his pro-slavery advocacy based on his belief that blacks were racially inferior, "defending slavery based on race as consistent with the natural order". Viriditas (talk) 01:23, 17 July 2024 (UTC)[reply]
I do not see it as an appeal to tradition. It is refuting the precedent. Your claim appears to be that because of AI, the future will change so that there will be a minority of rich people and a majority of poor people. That is refuted by stating that there has always been a minority of rich people and a majority of poor people. Therefore, the precent that the result is because of AI is invalid. You can make any claim you want. Because (whomever gets elected in November) the future will have a rich minority and poor majority. Because the Simpsons was signed for another season, the future will have a rich minority and a poor majority. Because Beyonce went into country music, the future will have a rich minority and a poor majority. etc... It isn't an appeal to tradition. It is a statement of history which should lead you to refine your claim. Because of AI, how will the rich minority and poor majority change? Will the minority become smaller? Will the gap widen, which is already does every generation? How much AI is required to make the change you are discussing? 75.136.148.8 (talk) 15:03, 17 July 2024 (UTC)[reply]
I don't think its refuted at all. You're ignoring the rise of economic inequality over the last several centuries. What I think you are doing is ignoring recent history. And this is, in fact, what all the discussion about AI focuses on.[47][48] So, I just find your comment a bit odd. Your comments are also highly reminiscent of all the discussions I've had with right-libertarians who refuse to accept there's even a problem and see the nation state and democracy as the true threat. Most of these types of people don't accept the concept of wealth disparity or income inequality and think it should be ignored. Viriditas (talk) 21:57, 17 July 2024 (UTC)[reply]
IMO bringing in this fruit analogy does not help to clarify the central issue. The best tasting varieties were not "extirpated" and are mostly still available – only not in your local supermarket. Growing and transporting produce requires resources, which have a limited capacity. Capacity is unlikely to be a major limiting factor in deploying beneficial AI. If AI and robots provide all necessary goods and services, no one will have an income to buy them. Can one expect the profit-driven owners of the means of production to make them available to all for free? Why should they do that? They'll be happy when they themselves are provided with all necessary goods and services and have no incentive for extending this to the rest of us.  --Lambiam 12:13, 16 July 2024 (UTC)[reply]
The example I was aiming for with the fruit analogy was to try and show that the agricultural preference for viable, commercial fruit that led one to select a certain variety for its qualities related to growing, packaging, shipping, and shelf stability, are comparably the same kind of commerical qualities we might expect with human-driven, artificial selection in AI development, leading to something like an algorithmic bias favoring poor outcomes for humanity, much as the flavor and palatability was selected against with the commerical dominance of Smooth Cayenne. According to Wikipedia "Smooth Cayenne is now the dominant cultivar in world production." Viriditas (talk) 23:45, 16 July 2024 (UTC)[reply]
I agree that historical precedents suggest rather strongly and convincingly that the most powerful people are not likely to give up their relatively privileged position voluntarily and will even resort to brutal measures to stay on top. Total control over the use of AI will make it much easier for the rulers of the world to remain the most powerful. They will need us no longer, so something drastic is necessary to save humanity from getting stuck in an Elysium-like future, one I'm afraid Matt Damon will be of little help getting us out of.  --Lambiam 11:49, 16 July 2024 (UTC)[reply]
I guess my followup question is this: why do Musk and others in the tech community keep repeating this line? Are they merely hopeful, technological utopians, or are they deliberately lying? Hacker News has thousands upon thousands of comments by people in the tech industry insisting that AI will make jobs a thing of the past and everyone will have leisure time to pursue their own hobbies. The thing is, I'm familiar with the older literature. People have been saying this for a little over a century. It never happened, but what did happen was the complete opposite: human productivity was expected to increase just as industrialization maximized output, resulting in less leisure time than in the past. In fact, the conventional wisdom now is that feudal serfs had more leisure time 500 years ago than modern workers do today. So are people deliberately lying about AI or are they just delusional? Finally, if you're familiar with Musk, then you know his position is that humanity needs to merge with the machine as a cyborg. That's literally his answer, I'm not making this up. Why am I the only person who finds this unacceptable? If you're the least bit familiar with science fiction, the evil scientist who somehow convinces the public to become cyborgs always ends up removing their individuality and exerting complete control over them. Surely, someone else has pointed this out? Viriditas (talk) 00:01, 17 July 2024 (UTC)[reply]
Just to add a bit to the cyborg argument: my understanding as to why Musk and others make this argument comes down to this: 1) They believe that humanity has evolved or gone as far as it can go without being threatened with extinction by machine intelligence 2) They believe that one way to insure survival into the foreseeable future is to compromise by becoming part-machine and merging with it as a kind of cyborg 3) This idea almost seems to contradict their assertion that we won’t need to work and everybody will have access to abundant resources 4) Newer data indicates that AI consumes far too many resources and energy requirements that makes it a direct threat to human existence. 5) See 1. Is this a self-fulfilling prophecy? Viriditas (talk) 03:53, 17 July 2024 (UTC)[reply]

Hah, i recall an earlier question of yours concerning a passage in Consider Phlebas, did you continue past the cannibalism of the islanders living in a natural state to the comic passage of Horza delighting in his destruction of the shuttle AI?

I am going to question your understanding of history. If you believe that the technological innovation of agriculture clearly didn’t benefit the masses of humanity then i wonder what kind of metrics you are using to evaluate the human condition? History has been a very long path out of darkness, towards greater standards of living and more liberal societies. While i think you are correct to take a skeptical look at the AI hype, i don't think you have a historical argument for imagining a future where most of humanity is forced into dire poverty. Many live in dire poverty now, and poverty is widespread, but along with technological innovation there has been a substantial positive trend:

The chart shows that almost 10% of the world's population live in extreme poverty. It also tells us that two hundred years ago, the same was true for almost 80% of the world’s population. In 1820, only a small elite enjoyed higher standards of living, while the vast majority of people lived in conditions that we call extreme poverty today. Since then, the share of extremely poor people fell continuously. More and more world regions industrialized and achieved economic growth which made it possible to lift more people out of poverty: In 1950 about half the world were living in extreme poverty; in 1990, it was still more than a third. By 2019 the share of the world population in extreme poverty has fallen below 10%.

If you have a basis for your fears i'm not sure how it comes from "historical reality". fiveby(zero) 01:40, 17 July 2024 (UTC)[reply]

I think you misunderstood what you read. I was citing the famous passage from Harari, which I assumed everyone was familiar with by now considering how much it has been quoted. His argument is that we didn’t so much as master agriculture as it mastered us and turned us into slaves. Harari argues that we were the ones domesticated by the plants, which changed our lives from one of leisure to one of toil. Keep in mind, Harari is intentionally turning the conventional narrative on its head. He’s arguing that the agricultural revolution was not as great as we make it out to be. It destroyed our bodies with labor, it eliminated our leisure time, it gave us a poor diet, it was less economically secure than hunting and gathering, and if the monoculture was threatened or the climate changed, it killed millions of peasants. It offered less security due to the need to protect possessions and provisions. “Since we enjoy affluence and security, and since our affluence and security are built on foundations laid by the Agricultural Revolution, we assume that the Agricultural Revolution was a wonderful improvement. Yet it is wrong to judge thousands of years of history from the perspective of today. A much more representative viewpoint is that of a three-year-old girl dying from malnutrition in first-century China because her father's crops have failed. Would she say 'I am dying from malnutrition, but in 2,000 years, people will have plenty to eat and live in big air-conditioned houses, so my suffering is a worthwhile sacrifice'?...Rather than heralding a new era of easy living, the Agricultural Revolution left farmers with lives generally more difficult and less satisfying than those of foragers. Hunter-gatherers spent their time in more stimulating and varied ways, and were less in danger of starvation and disease. The Agricultural Revolution certainly enlarged the sum total of food at the disposal of humankind, but the extra food did not translate into a better diet or more leisure. Rather, it translated into population explosions and pampered elites. The average farmer worked harder than the average forager, and got a worse diet in return. The Agricultural Revolution was history's biggest fraud." Viriditas (talk) 02:01, 17 July 2024 (UTC)[reply]
From another POV, see Changes in the Land: Indians, Colonists and the Ecology of New England (1983), which nicely illustrates the clash of civilizations in terms of agriculture. The colonists didn’t want to even try to understand how and why the indigenous people refused to settle down and stay put growing food on farms, aghast that they would even incorporate "lean times" into their worldview as an acceptable practice and normalized part of their life. Why don’t you just store food so you don’t have to go hungry, they would ask? The author investigates this question, finding that the nomadic, always on the move practice could have serious ecological benefits for the land, providing a kind of harmonious resiliency when things went well. Of course, when they didn’t, the risk of starving was very real. The book presents a very real look at an alternative way of life to agricultural farming in one place, perhaps a kind of living that has been entirely lost to history. And in spite of the ever present risks and dangers, there is a sense of a kind of special freedom and leisure that we no longer are aware of, one that has been lost to time. Viriditas (talk) 02:27, 17 July 2024 (UTC)[reply]
Hah, i recall an earlier question of yours concerning a passage in Consider Phlebas, did you continue past the cannibalism of the islanders living in a natural state to the comic passage of Horza delighting in his destruction of the shuttle AI?
Yes, I made it all the way to the eighth book, Matter, which I have in front of me. I threw it against the wall after getting so depressed by the events in it. I made it halfway through. I do plan on picking it up again so I can finish up with Surface Detail and The Hydrogen Sonata. One of the things I don't like is how Banks constantly reuses the same words and imagery. One of the things I do like, is how he manages to combine very serious drama, violence, and humor all in a single chapter. That's quite an achievement, and I can't quite recall another author successfully mixing all those elements together before. Viriditas (talk) 03:05, 17 July 2024 (UTC)[reply]
By positioning the upcoming robot revolution in a general setting of revolutionary cultural transitions, its unique character gets obscured. Read this article by Noah Smith: "Drones will cause an upheaval of society like we haven’t seen in 700 years", until its last sentence, "the age of freedom and dignity and equality that much of humanity now enjoys may turn out to have been a bizarre, temporary aberration."  --Lambiam 08:29, 17 July 2024 (UTC)[reply]
Thank you. I think it's pretty obvious where this is going. This 2020 opinion piece from scholars around the world, "Do Democracy and Capitalism Really Need Each Other?" indicates to me, based on the trends that we are seeing, that AI will be used to eliminate democracy once and for all. It's also interesting to note how the philosophical impetus for cryptocurrency fits into all of this. Crypto was intended by anti-democractic libertarians to be used to bankrupt the state, paving the way for right-wing billionaires to take over and use AI to create a new world where they aren't taxed and where the general public works for them on corporate slave plantations (company towns) with no regulatory framework, no guarantee of human rights, and no public infrastructure for healthcare, safe water, food, or air, in an economy based on servicing the wealthy and powerful. Viriditas (talk) 21:56, 17 July 2024 (UTC)[reply]
The discussion above is closed. Please do not modify it. Subsequent comments should be made on the appropriate discussion page. No further edits should be made to this discussion.

July 17

edit

Little curiosity about 2016 Us election

edit

The following discussion is closed. Please do not modify it. Subsequent comments should be made on the appropriate discussion page. No further edits should be made to this discussion.


If Hillary Clinton had won the Electoral College, in 2016, would there have been in reverse roles, faithless electors who would have prevented her election? Thanks. 2.35.188.164 (talk) 19:16, 17 July 2024 (UTC)[reply]
There is no way one can answer this question about a counterfactual hypothetical situation. There is no known reason to assume that some of the hypothetical pledged electors would have been faithless.  --Lambiam 19:51, 17 July 2024 (UTC)[reply]
Please read Faithless elector which describes several such Clinton electors in the 2016 presidential election. These were, in effect, protest votes cast when it was clear that Trump had won. Personally, I doubt those protest votes would have been cast had Clinton won the Electoral College, but this is speculation about a hypothetical. Cullen328 (talk) 19:56, 17 July 2024 (UTC)[reply]
One can speculate about this counterfactual hypothetical, but there is no known reason to assume that some of the pledged electors in this hypothetical situation would have been faithless – unless Clinton had won by a landslide, but then any faithless electors would not have prevented her election. However, there are no known facts that imply it is impossible that many would have voted for Faith Spotted Eagle. Therefore there is no way one can answer this question.  --Lambiam 06:24, 18 July 2024 (UTC)[reply]
This is very complicated because the electors are ruled by state law, not federal law. The Supreme Court (Jan 2020) affirmed that electors fall completely under state law. Since then, states have taken more and more action to stop and punish faithless electors, including proposing laws to invalidate and replace a faithless elector's vote. In the end, it is state law, so any complete answer would require a discussion of how each state would be handled, along with the changes in the laws from year to year. 75.136.148.8 (talk) 20:30, 17 July 2024 (UTC)[reply]
An exhaustive discussion of how each potential case would have been handled in each of the several states, informed by the changes in their laws from day to day, will not be of help in answering he question.  --Lambiam 06:30, 18 July 2024 (UTC)[reply]
That's because, as you keep pointing out, nobody can ever say what would have happened if some event that didn't happen had happened. The closest we could ever get is reporting the opinion of some commentator about what would/might have happened. But that's just their opinion; no-one can say whether it would actually have happened that way or not. And that is why we do not entertain questions that call for hypothesis, speculation or debate. And that is why I'm closing this now. -- Jack of Oz [pleasantries] 07:26, 18 July 2024 (UTC)[reply]
The discussion above is closed. Please do not modify it. Subsequent comments should be made on the appropriate discussion page. No further edits should be made to this discussion.

July 18

edit

Lake Lats and Longs

edit

Is there an accepted, standard location when finding latitude and longitude for ponds and lakes? Or, is wherever the pond lake first encounter? It is probably changed with GPS and Sats. DMc75771 (talk) 19:47, 18 July 2024 (UTC)[reply]

If there were one, I believe it would be the same for any type of geographic feature occupying an area with a somewhat defined shape, such as an island, a swamp, a salt flat, and so on. A plausible candidate, if one doesn't want to single out a specific feature of the area, is its geographical centre. As stated in a United States Geological Survey document quoted in our article, "There is no generally accepted definition of geographic center, and no completely satisfactory method for determining it." For most purposes, the centroid of the area will usually be satisfactory in practice. I suppose one will want the location to fall inside the area, but if the area is not even roughly convex, for example C-shaped, its centroid may fall outside the area.  --Lambiam 09:04, 19 July 2024 (UTC)[reply]
Agreed; the United States Geological Survey used "locations of lake centers in latitude and longitude" in its Digital Data Base of Lakes on the North Slope, Alaska (1986) p. 1. Alansplodge (talk) 11:18, 19 July 2024 (UTC)[reply]
Surprisingly hard to find a definitive statement from a major cartographic agency, but the Natural History Museum, London guidelines for recording the location of species says "It is best to use the geographic centre (the centroid/midpoint of both the latitude and longitude extremes) for the coordinates of named places". [49] Alansplodge (talk) 11:48, 19 July 2024 (UTC)[reply]
This definition means that – in theory – two disjoint lakes can share their centres; look here.  --Lambiam 23:50, 19 July 2024 (UTC)[reply]
Your long and curved lake is better suited to the advice for rivers: "Make a straight line from the mouth of the river to the head of the river. Calculate the centre of this line, and place the coordinates closest to the centre of the line on the river itself". Alansplodge (talk) 11:21, 20 July 2024 (UTC)[reply]
Another twist here is that for feature geometries polygons can have holes, and water body features are often modeled with multipoloygons, that is a set of polygons each of which can have zero or more holes. So even for a convex polygon there is no guarantee the centroid or the center of the bounding box are inside the feature. fiveby(zero) 02:03, 20 July 2024 (UTC)[reply]
From Centroid § Properties: "The geometric centroid of a convex object always lies in the object." It is not hard to show that this also holds for the centre of its bounding box.  --Lambiam 09:11, 20 July 2024 (UTC)[reply]
A convex polygon with holes is not a convex set, but if there are holes i guess it's not a simple polygon and you can't really call it convex. fiveby(zero) 13:42, 20 July 2024 (UTC)[reply]
I'm not sure that there's an actual requirement for perfect mathematical exactitude; if the co-ordinates indicate a point somewhere in the middle of a particular lake, then the job is done. Alansplodge (talk) 11:36, 20 July 2024 (UTC)[reply]
As I wrote, "I suppose one will want the location to fall inside the area", but if one is writing a program for assigning locations to geological features of a known extent, one needs some more precise definition. For example, if the centroid is not inside the area, the algorithm could select the location inside the area as far away as possible from its boundary, and if there are several such locations (for example for the C-shaped lake in my illustration), one as close as possible to the centroid.  --Lambiam 13:17, 20 July 2024 (UTC)[reply]
Some rounding to avoid false precision may be appropriate. Giving the position of Lake Superior to arcsecond precision may be excessive, as degree precision is good enough. PiusImpavidus (talk) 08:57, 21 July 2024 (UTC)[reply]
It may be worthwhile to point to WikiProject Geographical coordinates's guidelines on the matter. Elsewhere on that page one finds "For geographical features with an area, such as lakes, reservoirs, and islands, use a point reasonably in the center of the feature. (Remember not to specify too much precision; see Precision guidelines below.)" Deor (talk) 13:55, 21 July 2024 (UTC)[reply]

July 19

edit

Getting cash directly from the Internet

edit

I got to thinking, would it be possible in some way to get cash directly from the Internet, by using one's bank account credentials but without using a debit card or going to a bank office to ask for cash in person? You would log in to your bank over the Internet, withdraw money, and have the money somehow physically appear in front of you in cash.

MikroBitti magazine published an article of a "home banknote printer" in the early 1990s as I recall. The article announced a new kind of home printer that would print cash. "Of course, the printer won't print cash out of thin air", the magazine said, "you have to go out and buy licences for it." But this was an April Fool's joke. Would such a machine actually work, by withdrawing money from your bank account over the Internet and then printing it out in cash? JIP | Talk 20:32, 19 July 2024 (UTC)[reply]

There's a term for printing your own money: Counterfeiting. ←Baseball Bugs What's up, Doc? carrots21:40, 19 July 2024 (UTC)[reply]
Counterfeiting is not printing your own money, it is printing someone else's money. I can invent my own money and print it right now. As long as it is not a copy of an existing bank note, copying of which is indeed punishable by law in most places.
Personal checks, store credit coupons, lunch vouchers, local currencies and many others are scrip that promise to deliver goods and services, and are issuded by non-govermental entities.
We could invent a form of cryptocurrencies that can be printed out. Takes a bit of thiking though: how do you prevent them from being copied. Someone out there must be clever enough to figure it out, if not has alreadly done so. 85.76.35.183 (talk) 13:59, 21 July 2024 (UTC)[reply]
You can certainly print your own checks. That's routinely done in business. That's not the same thing as printing actual money. And printing "your own money" is not printing actual money either. ←Baseball Bugs What's up, Doc? carrots18:05, 21 July 2024 (UTC)[reply]
It's also worth pointing out that none of those things are legal tender. No one is required to accept checks nor bitcoins as payment. And note that the OP said "cash". None of those things qualify. ←Baseball Bugs What's up, Doc? carrots22:22, 21 July 2024 (UTC)[reply]
Bitcoin was briefly legal tender in the Central African Republic. It is legal tender now in El Salvador.[50]  --Lambiam 11:54, 22 July 2024 (UTC)[reply]
If a country wants to abandon its monetary system in favor of "vaporcash", they could do so. ←Baseball Bugs What's up, Doc? carrots19:51, 22 July 2024 (UTC)[reply]
I'm not promoting it; I just felt it was also worth pointing out that your statement "none of those things are legal tender" is not entirely accurate. Also, something that is generally accepted as payment for goods and services and repayment of debts, becomes, by virtue of that quality, "actual" money.  --Lambiam 22:16, 22 July 2024 (UTC)[reply]
Sure, a business can accept various forms of payment, including checks, credit cards, and even crypto if they want to. But that doesn't make it "legal tender" except for a country which has decided to allow bitcoins as such, for whatever reason. ←Baseball Bugs What's up, Doc? carrots23:54, 22 July 2024 (UTC)[reply]
DigiCash had a cryptographic protocol that would allow this. The "cash" was, as for bitcoin, a digital identifier, which could be represented as a conventional string such as 2ABA28C066F74CC8710ED3B22798255E, but for the purpose of home-printed money should be machine readable, as for example a QR code. As with any form of digital money, there is a potential problem of someone attempting to spend the same digital string twice. One solution is that a central system keeps a list of the unspent digital identifiers issued, validates a digital identifier when the money is used for a payment and at the same time removes it from the list. The DigiCash solution was decentralized; a malicious user could in theory spend a digital identifier twice, but this would reveal their identity. Otherwise, the user would remain untraceable.  --Lambiam 22:33, 19 July 2024 (UTC)[reply]

July 20

edit

Professional diving water jets?

edit

during a diving competition that is aquatic diving. What is the purpose of the water jets that spray into the pool? Brad (talk) 16:30, 20 July 2024 (UTC)[reply]

The Fédération Internationale de Natation (FINA) requires[51] surface agitation to provide a visual cue that helps divers judge when to enter their rotation as they dive into the water. Philvoids (talk) 16:50, 20 July 2024 (UTC)[reply]
thank you for your prompt reply! Now I just wonder why that didn't show up in the first place and I tried to Google it. Thanks again

Farmer market "slogan"

edit

Why do some farmers market stands say "We grow what we sell"? What does that mean, anyway? TWOrantulaTM (enter the web) 21:48, 20 July 2024 (UTC)[reply]

Because a lot of market stalls don't make what they sell. They buy their products from wholesalers. Nanonic (talk) 21:59, 20 July 2024 (UTC)[reply]
So it means that they themselves are the farmers who grew the produce you find at their stand.  --Lambiam 05:58, 21 July 2024 (UTC)[reply]
I used to live in a town with a large farmer's market. The local news investigated and found that almost all of the food for sale was purchased from a wholesaler who distributed food grown in other states and countries. The concept is that a farmer's market supports local farmers. But, over time, this market was overrun by people who did nto have a farm and simply resold food they purchased in bulk without the overhead of running a grocery store. Those who did grow their own food were characterized as lawyers who grew a few tomatos on the porch and sold them at a loss just to get tax writeoffs for having farmland on their property. I am certain that this practice is far more widespread than one city. So, people hear that the farmer's market doesn't actually have any local farmers and there is no incentive to go. It is necessary to advertise that the people are local and the food was grown locally. But, are they honest? Why not purchase vegetables off a plane from Brazil and sell it with a big sign that says "I grew this locally!" (Will anyone actually go through the trouble to sue for false advertisint? Will be be very hard for a lawyer to argue that "grew locally" includes "purchased from a shipment from another country"?) This exact same issue comes up with "farm to table" restaurants. They claim to primarily use ingredients from local farms. Do they? How do you know? What farms? What does "local" include? 75.136.148.8 (talk) 18:50, 22 July 2024 (UTC)[reply]
Perhaps local means local group. I'd be very surprised to see anything coming from further away, but not sure that I'd be willing to eat it.-Gadfium (talk) 20:46, 22 July 2024 (UTC)[reply]
The advertising admits "We supplement the bags with fruit and veg from further afield when local produce is not available."[52]. Here the advertising is "Strict rules apply so you can be certain that everything sold at the market is being sold by the farmer who grew it."[53] 2A00:23D0:E91:1101:C919:B9E0:1479:94A5 (talk) 12:36, 23 July 2024 (UTC)[reply]
Don't forget that time dilation will keep the goods fresh for longer than you might think. —Tamfang (talk) 00:37, 25 July 2024 (UTC)[reply]

July 22

edit

Biking

edit

During a hot summer in California, is it better to bike in the early morning or the evening? TWOrantulaTM (enter the web) 05:23, 22 July 2024 (UTC)[reply]

Define "better". ←Baseball Bugs What's up, Doc? carrots07:35, 22 July 2024 (UTC)[reply]
It's generally cooler in the early morning than in the early evening. Shantavira|feed me 08:59, 22 July 2024 (UTC)[reply]
☝🏽 that and traffic is usually lighter too, plus early morning cardio has health benefits. Folly Mox (talk) 10:58, 22 July 2024 (UTC)[reply]

Airplane Engineers' Wages in the Late 1950s

edit

Paul A. Suhler, From Rainbow to Gusto: Stealth and the Design of the Lockheed Blackbird (2009), p. 95:

Using a figure of $10 per hour, one week would cost $16,000. He (Kelly Johnson) added a profit of $500 per week and concluded, “For $225,000 can go full steam for 3 months” (Johnson, C. L., Archangel project design notebook, Lockheed ADP, Burbank, CA, entry for 26 April 1958, pg. 1).

It cost $16,000/week for a 40-man team to operate at full speed in 1958. I think this cost means wages + office costs (housing, utilities, many other things). How much money could an engineer make in the late 1950s? -- Toytoy (talk) 05:49, 22 July 2024 (UTC)[reply]

The 1960 census report (published in 1962) here shows "craftsmen, operatives, and kind" had a median income of $6,200. It also shows that salaries increased a great deal between 1950 and 1960, even when adjusted for 1960 dollars. 75.136.148.8 (talk) 14:34, 22 July 2024 (UTC)[reply]
edit

Hello everybody. At one time, Palestinian security forces used the former fishing trawler Chandalahe for military purposes; Israel damaged it in 2002 and since then it has been abandoned off the coast of Gaza. I wanted to know what his current fate is? Vyacheslav84 (talk) 15:33, 22 July 2024 (UTC)[reply]

Google has no results for that name except your query. You might try Wikipedia talk:WikiProject Ships? Alansplodge (talk) 15:19, 23 July 2024 (UTC)[reply]
Ok, thanks. Vyacheslav84 (talk) 23:23, 23 July 2024 (UTC)[reply]
Is this the transliteration of the name in Arabic?  --Lambiam 21:57, 23 July 2024 (UTC)[reply]
I do not know. Vyacheslav84 (talk) 23:23, 23 July 2024 (UTC)[reply]
Do you have any accessible source mentioning this name?  --Lambiam 09:35, 24 July 2024 (UTC)[reply]
Here the ship is named "Chindallah", and said to have been destroyed, although it may have been only "badly damaged".[54] "Shindallah" (شيندالله?) looks like an Arabic name.  --Lambiam 10:05, 24 July 2024 (UTC)[reply]

July 23

edit

Joseph Redford's autistic pride flag

edit
 

I found this in the Autism article, and here's what's wrong with it:

  • It appears to have been upscaled from a super low resolution.
  • JPEG artifacts are noticable, especially upon magnification.
  • There's a white dot in the infinity symbol and another white dot nearby in the golden region.

The Reference Desk may be a funny choice of place for me to bring up a shoddy image file, but this isn't the first time I had done that before. – MrPersonHumanGuy (talk) 02:01, 23 July 2024 (UTC)[reply]

Is that flag copyrighted? ←Baseball Bugs What's up, Doc? carrots03:36, 23 July 2024 (UTC)[reply]
The uploader, Intervex, claims it isn't eligible for copyright because of its simplicity, but I don't know if that's true. After all, Seattle's flag is copyrighted. – MrPersonHumanGuy (talk) 10:45, 23 July 2024 (UTC)[reply]
[Edit Conflict] I'm not an expert on copyright, but as a former professional editor have more than a layperson's grasp of it. In my judgement, this design is too complex to be ineligible for copyright and the licencing claim (which is on slightly different grounds anyway, go read) is mistaken.
It would not be surprising if the designer, Joseph Redford, had released it under a suitable CC licence, but we have no evidence of that, as it's not stated in the linked source.
However, flags are problematic, since they're meant to be publicly seen, analagous to heraldic coats of arms, which anyone writing about the arms' holder (for example) can depict without permission (a particular depiction may be copyright, but the point of arms, and usually flags, is that their essential details can be defined in words and redrawn from them). {The poster formerly known as 87.81.230.195} 94.2.67.235 (talk) 11:13, 23 July 2024 (UTC)[reply]
@MrPersonHumanGuy Do you have a reference desk question? If you wish to propose an improvement to any article the place to do so is the Talk page of that article, but the caption does make it clear that that is an old version. Shantavira|feed me 08:54, 23 July 2024 (UTC)[reply]
I have uploaded a photograph of the actual historic flag on display at People's History Museum during the 2023 "Nothing About Us Without Us" exhibition. If it is not used in the Autism article, it will eventually be deleted.  --Lambiam 11:09, 23 July 2024 (UTC)[reply]
Which one is upside down, the image embedded or the photo linked in the previous comment? 12.116.29.106 (talk) 15:20, 23 July 2024 (UTC)[reply]
The photo appears to show the flag rotated through 180º. Normally the hoist should be on the left. AlmostReadytoFly (talk) 15:49, 23 July 2024 (UTC)[reply]
File:Autistic Pride flag – Joseph Redford.png is about too be deleted. It was copied from https://phm.org.uk/wp-content/uploads/2023/06/Fabric-of-Protest-Get-Ready-June-2023.pdf, page 2.  --Lambiam 02:12, 27 July 2024 (UTC)[reply]

July 24

edit

Looking for one book

edit

Looking for one book. I only remember the beginning of the piece. Some guy found a derelict computer, sat down at it and started doing something, and then he saw a man with a gun walk up to the desk, they looked at each other in silence for a while, then the guy mechanically pressed the Enter button and the man shot him back. The work was read in the 1990s or very early 2000s. The piece appeared no later than the 1990s (probably earlier). I also remember that the guy was doing something enthusiastically on the computer: at first he typed without looking at the screen, but the message on the computer monitor made him do his work more slowly and carefully. The phrases went something like this. The message on the computer screen made him work more carefully. Behind the desk stood a man with a gun in his hand. The guy had never seen a real gun, except in the movies, but he knew immediately what it was. The guy's hand dropped mechanically to the Enter button, and the same second the black muzzle of the gun burst into flames, ending his life. Vyacheslav84 (talk) 08:27, 24 July 2024 (UTC)[reply]

Just to be clear, was this a novel, occupying a full printed volume, or a shorter work, perhaps the first story in a collection or anthology? {The poster formerly known as 87.81.230.195} 94.2.67.235 (talk) 12:50, 24 July 2024 (UTC)[reply]
I don't remember. --Vyacheslav84 (talk) 13:21, 24 July 2024 (UTC)[reply]

What is the name of this fallacy?

edit

What is the name of this fallacy?
1-A perfect man has the characteristics A, B and C.
2-Womans dont like a man with all the characteristics A, B and C.
3-So womans dont like a perfect man.

The problem here is that being perfect is about the result, if a woman dont like man X, he is not perfect, no matter what characteristics he has.
The fallacy implies having the characteristics A, B and C makes you perfect man, and then since this "A, B, C characteristics" man is not wanted by a girl, girls dont like perfect man.177.207.110.196 (talk) 14:14, 24 July 2024 (UTC)[reply]

See Fallacy of composition, Faulty generalization and Stereotyping. Modocc (talk) 14:56, 24 July 2024 (UTC)[reply]
The English sentences are somewhat ambiguous. "A man" can mean "some man" or "every man". Take the following syllogism:
  1. Every perfect man has all the characteristics A, B and C.
  2. There is no man with all the characteristics A, B and C who is liked by women.
  3. Therefore, there is no perfect man who is liked by women.
In this form, it is IMO a valid syllogism.  --Lambiam 16:45, 24 July 2024 (UTC)[reply]
I agree the wording is ambiguous. I interpreted it the same as you at first. But they added an additional condition below that; "if a woman dont like man X, he is not perfect", which means there is an additional statement: "2.5. Every perfect man is liked by women". The result of 1, 2, and 2.5 is "3A. Therefore, there are no perfect men". This, too, is logically consistent, and not a fallacy. Maybe the reason the original set of statements seems like a fallacy is that 1, 2 (and 2.5) are not really consistent with real life. Floquenbeam (talk) 21:40, 24 July 2024 (UTC)[reply]
I read "The problem here is that being perfect is about the result, if a woman dont like man X, he is not perfectt, no matter what characteristics he has." not as an additional statement to the syllogism, but as a rebuttal of it, implying that their first statement meant "some man" is "A perfect man...". Modocc (talk) 22:11, 24 July 2024 (UTC)[reply]
I find it difficult to interpret the sentence explaining the perceived problem with the syllogism, especally the part "being perfect is about the result" – the result of what, and why is that problematic? The best I can come up with for the rest of the sentence is that there is an additional premise:
4. A man who is not liked by some woman is not perfect.
Taking being liked by women to be a yes-or-no predicate, combining this prenise with the earlier conclusion 3 allows us to deduce that no man is perfect.  --Lambiam 22:44, 24 July 2024 (UTC)[reply]
The OP also wrote "The fallacy implies having the characteristics A, B and C makes you perfect man..." which confirms that not all men with these characteristics are perfect, only some are, which is why its conclusion is a hasty generalization. Modocc (talk) 22:48, 24 July 2024 (UTC)[reply]
The fallacy would include the assumption that A, B and C are sufficient to define perfection. ←Baseball Bugs What's up, Doc? carrots00:04, 25 July 2024 (UTC)[reply]

The OP's syllogism comes as sad news to any perfect mans hoping for admiration from womans but ordinary literate men and women need not feel affected. Philvoids (talk) 21:10, 24 July 2024 (UTC)[reply]

Don't be a jerk to people whose first language isn't English. Floquenbeam (talk) 21:29, 24 July 2024 (UTC)[reply]

I will explain myself better, when talking about a perfect man, the word perfect is about the results, a perfect man A if he wanted a woman B would be able to make her want to date him and not want to leave him and would love staying with him, all that with 100% certainty.
The fallacy comes from the fact that the person saying all that, is implying that having characteristics A, B and C makes you perfect, what makes someone perfect is the result and not the characteristics he has (unless some set of characteristics leads to him ALWAYS having the result and this is not the case with characteristics A, B and C alone). After seeing that a girl didnt wanted a guy with characteristics A, B and C (that he implied it meant the guy was perfect), he says that because of that it means girls wont like a perfect man.
Another example, lets imagine that assuming perfect play you can always wins chess as white, an related example would be.
1-A perfect chess player has the characteristics A, B and C.
2-A player with those characteristics playing as white lost the game.
3-This means being a perfect player doenst imply that you will win as white.
Again a perfect chess player is one that always win, he implies that some characteristics make some player perfect and assume something based at what happened with a player that has his own definition of perfect player.177.207.110.147 (talk) 00:50, 25 July 2024 (UTC)[reply]

For this to be a fallacy the conclusion 3 is either false or not implied, because being a perfect player means they win. Check. Then 2 someone with characteristics A, B and C loses. Check. The loser shares these characteristics with either all or some perfect players. Check. So 1 and 2 are not sufficient to conclude 3. Thus its a faulty generalization. Correct? Modocc (talk) 01:20, 25 July 2024 (UTC)[reply]
"For this to be a fallacy the conclusion 3 is either false or not implied". 3 is false.
"The loser shares these characteristics with either all or some perfect players. " They dont share those characteristics with perfect players , the person saying thing 1 implied those characteristics make someone perfect, this came out of his own mind, one characteristic that would make someone perfect is to have the endgame tablebase up to 32 pieces (max amount of pieces you can have at chess) and also never misclick/mouse slip when playing chess (so the move he want to make and the move he make is the same) and also be able to move really fast (needed if they are playing fast time controls)177.207.110.147 (talk) 01:49, 25 July 2024 (UTC)[reply]
It can be a valid argument even if the 1st premise is false and the conclusion is wrong. See False premise. Modocc (talk) 02:06, 25 July 2024 (UTC)[reply]
With a false premise the argument is unsound. Modocc (talk) 02:43, 25 July 2024 (UTC)[reply]
It's a Fallacy of presumption. Modocc (talk) 03:23, 25 July 2024 (UTC)[reply]
The objection to the conclusion relies on a premise not given in the syllogism: that a perfect chess player always wins as white (or that the perfect man is liked by women). Since that's not a premise given in the syllogism, the conclusion is perfectly valid. --Avocado (talk) 00:52, 27 July 2024 (UTC)[reply]
Perfectly valid sure but unsound, nevertheless, when its premise(s) are known to be false. Thus, it is an informal fallacy, and the name of the fallacy is what the OP requested and it's listed in the article as a fallacy of presumption. It's a fallacy that frequently occurs with paradigm shifts. Modocc (talk) 01:05, 27 July 2024 (UTC)[reply]

July 25

edit

Personal communications service mentions that:

A personal communications service (PCS) is set of communications capabilities that provide a combination of terminal mobility, personal mobility, and service profile management.

Does it means that Personal communications service (NANP) are mobile numbers because of providing mobility? I cannot find the mention of ‘mobile numbers’ in NANP or List of North American Numbering Plan area codes. 49.182.135.140 (talk) 01:24, 25 July 2024 (UTC)[reply]

Most mobile numbers on the NANP, in the sense of telephone numbers assigned to mobile cellular devices, do not have a geography-based area code; although the first three digits of such numbers are commonly referred to as their "area code", this is a bit of a misnomer then. In the USA these mobile numbers have a 5XX "area code". However, according to North American Numbering Plan § Non-geographic services, the converse is not always true: personal communications service 5XX numbers can also be for fixed devices (not using a cellular network).  --Lambiam 11:59, 25 July 2024 (UTC)[reply]

Punjabi and Bengali instruction language university

edit

Which universities in Punjab, India and Pakistan are known to give instructions or lectures in Punjabi and which universities in West Bengal are known to give instructions in Bengali? I was under the impression that all universities in Punjab, India; Punjab, Pakistan and West Bengal in India give only in Hindi and English. Please do enlighten. Donmust90 Donmust90 (talk) 16:46, 25 July 2024 (UTC)[reply]

July 26

edit

Name for type of interchange

edit

At coordinates 34.79749785424678, -82.37485296822788, there is an interchange. On both sides of the highway, there is a short side roadway that has multiple on and offramps. I've been looking at interchange designs, but I can't find an example of this "multi-interchange" with a proper name. I would like a reference that includes this interchange design. 75.136.148.8 (talk) 16:04, 26 July 2024 (UTC)[reply]

That looks like a collector-distributor interchange. —C.Fred (talk) 16:06, 26 July 2024 (UTC)[reply]
Thanks. That looks correct. 75.136.148.8 (talk) 16:32, 26 July 2024 (UTC)[reply]

July 27

edit

Extraterrestrial orbiters update?

edit

What's the latest data telling us about Jupiter from Juno and about Venus from Akatsuki? Have we learned anything new and interesting lately? Viriditas (talk) 01:05, 27 July 2024 (UTC)[reply]

Juno news is still being updated here. Akatsuki news hasn't had updates, but the old articles are still getting minor updates here. 75.136.148.8 (talk) 11:29, 29 July 2024 (UTC)[reply]

Boom box batteries

edit

In Do the Right Thing, one of the characters needs 20 D batteries for his boombox. What sort of boombox would require 20 D batteries, or was that simply a fictional exaggeration? How many and what kind of batteries would one expect to use for a boombox of the size shown in the movie? 2601:18A:C500:E830:526A:B17D:E5EF:4ACD (talk) 04:25, 27 July 2024 (UTC)[reply]

[55] Modocc (talk) 05:27, 27 July 2024 (UTC)[reply]
So, 10 D batteries then. 2601:18A:C500:E830:526A:B17D:E5EF:4ACD (talk) 05:41, 27 July 2024 (UTC)[reply]
My understanding is you would need 20 at a time because the 10 wouldn't last long enough. Viriditas (talk) 08:52, 27 July 2024 (UTC)[reply]
You've got to fight the power consumption. Clarityfiend (talk) 11:06, 28 July 2024 (UTC)[reply]

July 28

edit

Multiplication memorization

edit

I remember back in elementary school, one of my classmates said he memorized multiplication tables by assigning a gender and color to each equation. For example, he would say eight times nine was male and purple and five times eight was female and brown. Is there a name for this phenomenon, and does this actually work? TWOrantulaTM (enter the web) 02:49, 28 July 2024 (UTC)[reply]

Grapheme–color synesthesia.-Gadfium (talk) 04:38, 28 July 2024 (UTC)[reply]
Synesthesia is likely a natural in-born perception while the OP's classmate voluntarily cultivates an unusual (two-dimensional) form of Mnemonic. These can aid memory because we more easily remember spatial, personal, surprising, physical, sexual, humorous and otherwise "relatable" information than more abstract or impersonal forms of information. Philvoids (talk) 13:46, 28 July 2024 (UTC)[reply]

July 30

edit

National Identity perceived

edit

Hello. Small curiosity; I don't know if a census was taken at the time or something, but when England won the World Cup in 1966, what was the perceived national identity in the country that year, (England alone); did people describe themselves as more English or more British? Thank you very much. 2.45.43.119 (talk) 12:51, 30 July 2024 (UTC)[reply]

Identifying as English or British is not exclusive of one another. A person can authentically identify as both. It is based on need. Your IP looks up to the United States, so in a American sense, a person can identify as Cajun, Louisianian, and American all at the same time, with none of those identities taking away from another identity or being more of an identity than any other identity. It is based on need. If asked for identity while in New Orleans, the person would likely claim to be Cajun. If asked for an identity at the Delta hub in Atlanta, the person might identify as Louisianian. If asked for identity at the Olymics in France, the person might identify as American. Similarly, a person might identify as English while in Britain, but as British while outside Britain, and even go further to identify as a Londoner if in Manchester. 12.116.29.106 (talk) 17:16, 30 July 2024 (UTC)[reply]
The English/British nationality issue is complex and evolving, but my impression (I was 8 in 1966) was that English and British were used interchangeably by English people in the 1960s. Note that there was no question about ethnicity in the UK census until the 1991 United Kingdom census and then "White British" was the only option for "indigenous" people (for want of a better term).
For the current situation, see National identity, England and Wales: Census 2021 Alansplodge (talk) 18:16, 30 July 2024 (UTC)[reply]
@Alansplodge: I'm only a couple of years older than you, but certainly we were taught the difference between British and English. Possibly a north/south or London/rest of the UK issue? We lived in Yorkshire and I'm of mixed Yorkshire/Scottish parentage.
One can have been taught the distinction and yet use the terms interchangeably in informal speech.  --Lambiam 08:35, 31 July 2024 (UTC)[reply]


August 1

edit